You are on page 1of 143

GENERAL MEDICINE

PAPER 1

SMAHRT NOTES
................................................................................................3

..........................................................................................................................................................3
...........................................................................................................................................................1
......................................................................................................................................................10

.........................................................................................................................................13

........................................................................................................................................................13
.........................................................................................................................................................15
......................................................................................................................................................21

...............................................................................................................................23

........................................................................................................................................................23
.........................................................................................................................................................27
......................................................................................................................................................33

.................................................................................................35

........................................................................................................................................................35
.........................................................................................................................................................44
......................................................................................................................................................57

...........................................................................................................60

........................................................................................................................................................60
.........................................................................................................................................................70
......................................................................................................................................................81

..............................................................................................................84

........................................................................................................................................................84
.........................................................................................................................................................91
....................................................................................................................................................100

...................................................102

......................................................................................................................................................102
.......................................................................................................................................................103
....................................................................................................................................................105
..........................................................................107

.......................................................................................................................................................107

............................................................................................................................109

......................................................................................................................................................109
.......................................................................................................................................................113
....................................................................................................................................................119

.............................................................................123

......................................................................................................................................................123
...........................................................................................................................................................4
........................................................................................................................................................9
1. What is peptic ulcer? Write about Peptic acid disease – causes, clinical features & Mx [17, 12, 11]
a. H. Pylori Infection (incl treatment) [21, 05, 03]
b. Rapid urease test [21]
c. Sweat chloride test [17]
d. Mention 4 Proton Pump Inhibitors with dosage [14, 12]
e. Mention the classes of drugs used in the treatment of Peptic Ulcer [13]
f. Treatment of Recurrent Duodenal Ulcers [03]
Ans.
 A peptic ulcer is a break in the epithelial cells of either the stomach or the duodenum penetrating
down to the muscularis mucosa
Causes: Helicobacter pylori is a slow-growing, spiral, Gram-negative, flagellate, urease producing
bacterium, which plays a major role in gastritis and peptic ulcer disease
Clinical features of peptic ulcer disease:
 Recurrent, burning epigastric pain

Gastric Ulcer Duodenal Ulcer


Incidence Less Common More Common {2-3 times}
Site Along Lesser Curvature 1st Part of Duodenum
A/w H. Pylori Less Common More Common
Acid Levels Normal High

 Food aggravates the Pain Pain is worse when patient is hungry


Pain
 Anorexia and weight loss occur Night Pain
Nausea &
 More Common Less Common
Vomiting
Blood  Hematemesis more common Melena more common
Weight Loss  Present Absent
Course  Less chances of remission More chances of Remission
Malignancy  High potential Less potential
Diagnosis of Helicobacter pylori infection
Non-invasive methods
Serological tests - detect the particular IgG antibodies
13
C – Urea breath test: It can be used as a screening test.
Stool antigen test – A specific immunoassay, using monoclonal antibodies for the qualitative
detection of H. pylori antigen, is now widely available.
Invasive methods (endoscopy)
Biopsy urease test (Campylobacter--like organism – CLO test): Antral biopsies + urea + phenol red ➔ If H.
pylori is present, the urease enzyme of the bacteria splits the urea to release ammonia, which causes a rapid colour change
(yellow to ).
Histology of Giemsa-stained sections of gastric mucosa obtained at endoscopy.
Culture of gastric biopsies and in vitro sensitivities to antibiotics can be tested – in refractory cases.
All gastric ulcers must be biopsied to exclude an underlying malignancy
Complications of peptic ulcer disease
Haemorrhage & Perforation – DUs perforate more commonly than GUs.
Gastric outlet obstruction – The obstruction – may be pre-pyloric, pyloric or duodenal – occurs
either because of the surrounding oedema (with pain) or because the healing of an ulcer has been
followed by scarring (no pain)
Management
Eradication therapy: Eradication regimens comprise 2 antibiotics & 1 PPI. Examples are:
 Omeprazole 20 mg + clarithromycin 500 mg & amoxicillin 1 g – all twice daily for 1-2 weeks.
 Omeprazole 20 mg + clarithromycin 500 mg & metronidazole 400 mg – all BD for 1-2 weeks.
Two--week treatments the eradication rates
In areas of clarithromycin resistance – Use Quadruple therapy: Bismuth chelates (120 mg 4
times daily), metronidazole (400 mg 3 times daily), tetracycline (500 mg 4 times daily) and a
PPI (20–40 mg BD) for 14 days is used.
General measures
Stop smoking as it slows mucosal healing.
Patients with gastric ulcers should be routinely re--endoscoped at 6 weeks to confirm mucosal
healing and exclude an underlying gastric cancer.
Stop the ingestion of NSAIDs or use one with low side-effects.
Surgical treatment – used only for complications.
In the past, two types of operation were performed: a partial gastrectomy or a vagotomy. In the
latter, either a truncal vagotomy with a pyloroplasty or gastro--jejunostomy was performed, or
highly selective vagotomy or proximal gastric vagotomy, which did not require a bypass
procedure.
Long--term complications of surgery include: Recurrent ulcer, Dumping, Diarrhoea & Nutritional
complications (iron deficiency, folate deficiency, vitamin B12 deficiency due to intrinsic factor
deficiency & weight loss).
Other H. pylori-associated diseases: Gastric adenocarcinoma & Gastric B--cell lymphoma (MALT)
----------------------------------------------------------------------------------------------------------------------------------------
2. Discuss the causes, pathogenesis; C/F, inv. & Mx of chronic diarrheal diseases in children [03, 2000]
a. Management of persistent diarrhoea [04]
b. Oral Rehydration therapy [05, 01]; Preparation of oral rehydration solution at home [97]
Ans.
CHRONIC
DIARRHEA is defined
as

. The term
chronic diarrhea is not synonymous
with persistent diarrhea.

ETIOLOGY: 👉🏻
CLINICAL FEATURES:
Onset of diarrhea after intake of cow or buffalo milk or formula
Rectal bleeding; Anemia; Failure to thrive
Response to milk withdrawal
History of meconium ileus
Family history of allergy or atopy
Predominant or associated lower respiratory tract Infections
Signs: Clubbing; Non-pitting pedal edema (lymphedema); Recurrent anasarca; Hypoalbuminemia &
hypoproteinaemia
High sweat chloride (>60 mEq/L)
INVESTIGATIONS:
 Faecal calprotectin  Lactose challenge
 Faecal elastase  Secretin – Cholecystokinin stimulation test
 Ileocolonoscopy with biopsies  Small-bowel biopsy
 24-hour stool fat estimation  Small-bowel MRI
 Ultrasound, CT and MRCP Abdomen  Barium follow-through
TREATMENT:
 Fluid and electrolyte Mx – ORS & Intravenous rehydration
 Antimotility agents – Loperamide, diphenoxylate
 Antisecretory agents – Racecadotril
 Antispasmodic agents – Dicyclomine, hyoscine
 Antibiotics – Ciprofloxacin, Metronidazole

:
The principles of management are:
 Correction of dehydration, electrolytes and hypoglycaemia
 Evaluate for infections using investigations (hemogram, blood and urine culture) and their Mx.
 Nutritional therapy.
Most of patients with persistent diarrhea can be treated on outpatient basis.
Patients in need of hospital admission are those with
Age <4 months and not breastfed
Presence of dehydration
SAM – Severe Acute Malnutrition
Presence or suspicion of systemic infection.

The aim of oral fluid therapy is to prevent dehydration & reduce mortality
Oral fluid therapy is based on the observation that glucose given orally ↑the intestinal absorption
of salt and water, and is capable of correcting the electrolyte and water deficit.
Indications of ORS: To treat acute diarrhoeas due to
all aetiologies, in all age groups, and in all countries.
Composition of ORS:
Inclusion of trisodium citrate in place of sodium bicarbonate made
the product more stable and it resulted in less stool output
especially in high-output diarrhoea as in cholera, probably because of direct effect of trisodium citrate in increasing
intestinal absorption of sodium and water
If the WHO mixture of salts is not available, then: a
simple mixture consisting of table salt (1 level
teaspoon) and sugar (6 level teaspoon) dissolved in 1
litre of drinking water may be safely used until the
proper mixture is obtained.
The earlier the treatment is instituted the better it is
for the patient
Advantages of ORS: The introduction of oral
rehydration fluid has not only reduced the cost of treatment, but also made possible treatment of
patients in their own homes by primary health workers or relatives of patient
----------------------------------------------------------------------------------------------------------------------------------------

1) Inflammatory bowel disease [21, 12]


a. Ulcerative Colitis {incl Complications} [18, 16, 15, 08]
Ans.
Inflammatory bowel disease is an immune mediated chronic intestinal inflammation which occurs
due to dysregulation of innate immunity
It is of two types – Crohn’s Disease & Ulcerative Colitis

Crohn’s Disease
Gender M=F M>F
Genes HLA DR 103
NOD2, IRGM
involves {use of α-methyl dopa risk}

Smoking Increases risk for Crohn’s disease Protects against ulcerative colitis
1) Inflammation involves the entire
1) Inflammation is Superficial with only
wall of GIT – Transmural
mucosal & submucosal ulcers
inflammation with Deep knife-like
2) No skip lesions – Continuous
Pathogenesis ulcers
involvement
2) Skip lesions
3) Superficial crypt abscess
3) Deep granulomas
4) TH2 cells are involved
4) TH1 cells are involved
Terminal ileum (MC site); but it can UC Begins in rectum & can extend
Sites proximally upto Caecum; remainder of GIT
affect any area of GIT
is unaffected
Left lower quadrant pain (rectum) with
Clinical Right lower quadrant pain (ileum) with
bloody diarrhea
presentation non-bloody diarrhea
{Ileocolitis with rectal sparing} Proctitis (rectum)
 Many ulcers along the bowel axis  Pseudopolyps
Endoscopic
 Cobblestone mucosa  Loss of haustra
findings
 Creeping fat &  No stricture formation
Imaging String sign; Rose thorn appearance Lead pipe sign; mucosal granularity
• Malabsorption with nutritional deficiency
• Fistula formation Toxic megacolon, hemorrhage &
Complications
• Calcium oxalate nephrolithiasis Carcinoma of Colon
• Carcinoma of Colon
 Eye – conjunctivitis, episcleritis, Uveitis
 Skin – Erythema Nodosum, Pyoderma Gangrenosurn {EN & PG}
Extraintestinal
 Hepatobiliary system – liver abscess, primary sclerosing cholangitis
manifestations
 Arthritis (peripheral joints, ankylosing spondylitis, sacroiliitis, migratory polyarthritis)
 p—ANCA vasculitis
1. Blood Tests – ESR & CRP is raised; anemia & hypoalbuminemia due to loss from
intestine
Investigations 2. Stool Examination – to exclude infective causes of colitis
3. Imaging: USG; Barium Meal; CT Abdomen; Radionuclide scans
4. Colonoscopy & Colonic biopsy
 Conservative: Avoid cigarette smoking; Control diarrhea with loperamide
 5 ASA {Amino-salicylic acid} – Ex: Sulfasalazine, Olsasalazine, Mesalamine etc.
Medical  Glucocorticoids: Prednisone 40 – 60 mg/day
Management  Immunosuppressive agents – Azathioprine, 6-MP, MTX
 Nutritional therapies
 Anti TNF antibody – Infliximab, adalimumab
Surgical Indicated only in presence of strictures
Total proctocolectomy with ileostomy
Management and obstruction
----------------------------------------------------------------------------------------------------------------------------------------
2) Barrett's Oesophagus [21]
Ans. Barrett’s oesophagus is a pre-malignant condition, in which the normal squamous lining of the
lower oesophagus is replaced by columnar mucosa that may contain areas of intestinal metaplasia
• Also known as CLO – Columnar lined Oesophagus
• Etiology:
- Age & Sex: > 50s; M>F
- Risk factor: Chronic GERD
• Pathogenesis: It is an adaptive response to chronic gastro-oesophageal reflux {GERD}
 Metaplastic columnar epithelium develops because it is more resistant to acid pepsin damage
than squamous epithelium
 Endoscopically, Barrett’s epithelium can be seen as continuous sheet, red velvety appearance
and finger like projections into esophagus
 In long standing cases, it progresses to dysplasia  Adenocarcinoma
• Classification:
» : metaplasia ≥ 3cm
» metaplasia < 3 cm
• Investigations:
 Endoscopy – use with indigo carmine spray to detect intestinal metaplasia and dysplasia
 Biopsy – use Alcian blue stain – to detect mucin-containing goblet cells – Hallmark of Barret’s
esophagus
• Complications: Dysplasia; Esophageal adenocarcinoma
• Treatment:
» Endoscopic surveillance for 2-3 years followed by esophagectomy of the involved region.
» Radiofrequency ablation and photodynamic therapy
----------------------------------------------------------------------------------------------------------------------------------------
3) Irritable Bowel Syndrome {incl Treatment} [18, 17, 15, 14, 09]
Ans.
Irritable bowel syndrome (IBS) is characterised by recurrent abdominal pain in association
with abnormal defecation in the absence of a structural abnormality of the gut.
:
Hereditary and environmental factors
Abnormal gastrointestinal motility – Ex:  gastrocolic reflex, altered gastric emptying,  small
bowel contractions &  small intestinal transit
Visceral hypersensitivity & Microscopic inflammation
Psychological disturbances – anxiety, depression, phobias and somatization.
Excess caffeine intake.

Pain – Colicky or cramping in nature, felt in the lower abdomen and relieved by defecation.
Abdominal bloating worsens throughout the day –  belching & flatulence.
Most patients alternate between episodes of diarrhoea and constipation
A/w - weight loss, sleep
disturbances, dyspepsia,
heartburn, nausea & vomiting
Symptoms worsens by stress.
:

- faecal
calprotectin
– for pts. > 50
years to rule out carcinoma
colon
Hydrogen Breath Test – to rule
out malabsorption
– for
prominent dyspepsia

: is clinical &
can be made by Rome
III criteria 👉🏻
: 👉🏻
----------------------------------------------------------------------------------------------------------------------------------------
4) Gluten sensitive enteropathy & it’s Dx [18]
Ans.
Gluten sensitive enteropathy, aka Coeliac disease, is an inflammatory disorder of the small bowel
occurring in genetically susceptible individuals.

Immune-mediated damage of small bowel villi occurs due to gluten exposure


Gluten is present in wheat, barley, rye & oats.
Gluten protein {contain gliadin toxin}  Once
absorbed, gliadin is deamidated by tissue
transglutaminase (tTG)  Deamidated
gliadin is presented by APCs via MHC class II
 TH cells mediate tissue damage  cause
Inflammatory damage to the intestinal
mucosa
a/w HLA-DQ2 & DQ8 {in 90% cases};  incidence
of IgA deficiency is seen in celiac disease

Children classically present with abdominal distension, diarrhea, and failure to thrive after weaning
on to gluten-containing foods
Adults classically present with chronic diarrhea and bloating
Symptoms of Malabsorption  weight loss, growth retardation, features of vitamin and mineral
deficiencies, anaemia, osteoporosis {impaired iron, folate & calcium absorption}
Small, herpes—like vesicles may arise on skin (dermatitis herpetiformis) – due to IgA deposition at
the tips of dermal papillae; resolves with gluten-free diet.

– reveals flattening of villi, hyperplasia of crypts &  intraepithelial


lymphocytes.
:
 IgAtTG {Immunoglobulin-A anti-tissue transglutaminase} antibody detection by ELISA
 Antiendomysial IgA antibodies are highly sensitive and specific for celiac disease
 Antigliadin IgA and IgG antibodies are sensitive but not specific.
of proteins, carbohydrate, fat and vitamins

Conservative Management:
 Lifelong gluten-free diet – Wheat, rye, barley & oats should be excluded from the diet.
 Lactose-free diet is also recommended
 Any deficient vitamins and minerals should be replaced.
 90% of patients on gluten-free diet experience symptomatic improvement within 2 weeks.
 Lymphoma should be ruled out in refractory sprue.
Medical Management – Steroids may be of help in refractory sprue

Rash (dermatitis herpetiformis)


Neurologic disorders (myopathy, epilepsy); Psychiatric disorders (depression, paranoia)
Reproductive disorders (infertility, spontaneous abortion).
Small bowel carcinoma and T-cell lymphoma are late complications in refractory sprue
Food allergy; Tropical sprue; Crohn disease; Autoimmune enteropathy;
Prolonged post-enteritis syndrome; Common variable immunodeficiency
----------------------------------------------------------------------------------------------------------------------------------------
5) Zollinger Ellison Syndrome [15, 14, 01]
Ans.
 ZES is a rare disorder characterised by the triad of gastric acid hypersecretion, severe peptic
ulceration, and a neuro-endocrine tumour of the pancreas or duodenum (‘Gastrinoma’)
 Age: it is most common between 30 and 50 years of age
 Site: Found within the "Gastrinoma triangle" bounded by porta hepatis, neck of the pancreas, and
3rd part of duodenum.
:
Symptoms of gastroesophageal reflux, Abdominal pain – due to peptic ulcers.
Multiple Ulcers – can extend even upto jejunum.
Pancreatic enzyme inactivation by acid  maldigestion & malabsorption
Diarrhea and weight loss occur in one-third of patients due to direct intestinal mucosal injury
:
Upper GI endoscopy: to look for hypertrophy of gastric folds & duodenal ulcerations.
 Fasting serum gastrin concentration (> 150 pg/ml)
Measurement of gastric pH
Secretin stimulation test: - IV secretin produces  in serum gastrin in patients with Gastrinoma –
useful to distinguish ZES from other causes of hypergastrinemia.
Imaging studies – Endoscopic ultrasonography (EUS); CT and MRI scans – to locate the site of
primary tumor and to identify metastases.
Somatostatin receptor scintigraphy (SRS) with single photon emission computed tomography
(SPECT) can identify the site of Gastrinoma.
Serum PTH, prolactin, LH, FSH & GH levels should be obtained to exclude MEN 1
:
Medical Management:
Oral PPIs (omeprazole, esomeprazole, rabeprazole, pantoprazole) – to  acid secretion.
Somatostatin analogues
Other measure: Interferon alpha; Cytotoxic chemotherapy; Hepatic arterial chemoembolization.
Surgical Management – Localized disease is treated with surgical resection.
----------------------------------------------------------------------------------------------------------------------------------------
6) Reflux Oesophagitis [13, 2000]
a. Gastro oesophageal reflux [07, 04]
Ans.
Reflux Oesophagitis occurs mainly due to ↓ function of LES causing regurgitation of gastric contents
into oesophagus.
:
Pregnancy, obesity, ascites and weight lifting acts –
 intra-abdominal pressure
Alcohol, smoking, chocolate, coffee, large fatty meals
-  LES tone
Drugs that relax the smooth muscle –
Anticholinergic, β-adrenergic drugs, Nitrates & CCBs.
After treatment for achalasia – reflux increases
Sliding Hiatus hernia, scleroderma – predisposes to
reflux
:
Patient is often overweight with c/o substernal pain, heartburn - provoked by bending,
straining or lying down.
Water brash – salivation due to reflex salivary gland stimulation as acid enters the gullet
Odynophagia or dysphagia is present
Mucosal erosions may produce bleeding, hematemesis and anaemia.
Some patients are woken at night by choking as refluxed fluid irritates the larynx  hoarseness
Reflux into the pharynx, larynx & tracheobronchial tree  chronic cough, bronchoconstriction,
pharyngitis, laryngitis, bronchitis, or pneumonia
:
• in the head-down position can detect movement of barium
from stomach to esophagus suggesting reflux
• is used to confirm damage to esophagus or to rule out other alternate pathology
• can tell whether the symptoms are due to acid reflux
• is the gold standard test for identifying reflux
• Resting and stress to rule out angina
:
Conservative Mx: Weight reduction; Elevation of the
head of bed at night; Avoiding food at least 3 h before
bedtime; Avoid smoking, alcohol, caffeine, chocolates,
mints & carbonated drinks
Antacids – MgSO4 + Al(OH)3
Drugs that ↑ tone of LES {Prokinetic Drugs}, e.g.,
metoclopramide; mosapride, cinitapride & domperidone
H2 Blockers, e.g., cimetidine and ranitidine.
Oral PPIs (omeprazole, esomeprazole, rabeprazole,
pantoprazole) – to  acid secretion
Antireflux surgery, e.g., Modified Nissen’s
fundoplication.

Oesophagitis, oesophageal mucosal erosion, haemorrhage & Benign oesophageal stricture.


Barrett’s oesophagus.
: Aspiration pneumonia, Chronic cough, Asthma & Bronchiectasis.

 Posterior laryngitis ➔ pain in throat, hoarseness and repeated throat clearing.


 Contact ulcers and granulomas.
 Posterior glottic stenosis.
 Carcinoma larynx.
– Otitis media with effusion.
– Globus hystericus; Iron deficiency anemia due to chronic blood loss from esophageal ulcers
----------------------------------------------------------------------------------------------------------------------------------------
7) Constipation [04]
Ans.
Constipation is defined as infrequent stools (less than 3 times in a week), hard stools, excessive
straining, or a sense of incomplete evacuation.
ETIOLOGY: female sex, older age etc. 👉🏻
CLINICAL FEATURES:
Excessive straining, or prolonged defecation time;
sensation of incomplete defecation
Manual manoeuvring required to defecate
Lumpy or hard stools
Pain on defecation, and rectal bleeding
Presence of vomiting, inability to pass flatus and pain
abdomen – indicates intestinal obstruction.
INVESTIGATIONS
: Serum calcium, blood sugar, thyroid &
parathyroid function tests, if clinically indicated.
– Look for occult blood to r/o Ca colon.
– to r/o local anorectal abnormalities

- measured by performing an abdominal radiograph after ingestion of


radiopaque markers. Retention of >20% of the marker indicates prolonged transit.
– used to assess pelvic floor dysfunction & anorectal disorders.
TREATMENT
:  fluid intake, high fiber diet & physical activity.
:
ulk laxatives – Ex: psyllium (ispaghula), methylcellulose, and calcium polycarbophil.
smotic laxatives – Ex: magnesium sulfate, lactulose, polyethylene glycol, sorbitol & glycerine
mollients (stool softeners) Ex: Docusate sodium and Liquid paraffin.
timulant laxatives – Ex: castor oil, bisacodyl and senna – They  intestinal motility & secretion of water
into the bowel.
rokinetic agents – Ex: Metoclopramide and mosapride – They  intestinal motility.
nemas – they act within 5-15 min and are given rectally.
ewer therapies for constipation:
▪ Prucalopride - a prokinetic agent -  colonic motility &  transit time
▪ Osmotic agents – Lubiprostone and Linaclotide –  intestinal fluid secretion by acting on the
intestinal mucosa.
▪ Naloxegol and methylnaltrexone are useful in opioid induced constipation.
----------------------------------------------------------------------------------------------------------------------------------------
8) Differential diagnosis of abdominal pain in children [03]
Ans.
----------------------------------------------------------------------------------------------------------------------------------------
9) Causes and treatment of upper gastric intestinal bleeding [03, 02]
Ans.
Haematemesis is red with clots when bleeding is rapid and profuse, or black (‘coffee grounds’) when
bleeding is less severe
:
:
Syncope; Melaena & in chronic
cases symptoms of anaemia can
be seen
:
Gain IV access with one
large-bore cannula
Basic investigations: CBP,
Urea, LFT, electrolytes, PT
etc.
Determine Blatchford score
– A score of 2 or less is associated with a good prognosis.
IV PPIs.
Urgent endoscopy with cauterisation– for patients not responding to PPIs. Actively bleeding
varices can be treated with sclerosant injection or rubber band application to the bleeding varix.
Other options: Intra-arterial Embolization or Vasopressin; TIPS {Transvenous intrahepatic Portosystemic Shunts}
Packed red blood cells & Aggressive fluid replacement with normal saline or Ringer lactate
Surgical Management – Laparotomy – for perforated ulcers.
----------------------------------------------------------------------------------------------------------------------------------------
10) Congenital hypertrophic pyloric stenosis [02]
Ans.
Hypertrophic pyloric stenosis is the most common surgical disorder of the GIT in infants.
Etiology: The pylorus is thickened and elongated with narrowing of its lumen due to hypertrophy of
the circular muscle fibres of pylorus. The disorder is 4-6 times more common in boys than girls
Clinical Features:
 Infants present with non-bilious vomiting that gradually increases in frequency and severity to
become projectile in nature.
 Recurrent and persistent vomiting  dehydration, malnutrition and hypochloremic alkalosis
 A/w Constipation.
 Signs:
• A vigorous peristaltic wave can be seen to move from left hypochondrium to umbilicus
especially after feeds (as the stomach muscles contract forcibly to overcome the obstruction).
• A firm olive-shaped mass is palpable in the mid-epigastrium, especially after feeds.
Investigation:
USG abdomen – it reveals muscle thickness of 4 mm and pylorus length of 16 mm – USG is 100%
sensitive and nearly 90% specific in the diagnosis.
Upper GI barium study & Upper GI endoscopy can also be done
Treatment – Surgery – is performed.
----------------------------------------------------------------------------------------------------------------------------------------
11) Hirschsprung’s disease [2000]
Ans. Hirschsprung’s disease refers to congenital absence of ganglion cells in the large intestine leading
to constipation and colonic dilatation (megacolon)
Associated with Down syndrome
Mode of inheritance: Autosomal dominant
Genetic Alteration: RET proto-oncogene (also involved in MEN-2 & MEN-3).
Pathogenesis: loss-of-function mutations  dysregulation of enteric neurogenesis  absence of
ganglion cells in rectum and/or sigmoid colon  internal anal sphincter fails to relax
Clinical Features:
Constipation, abdominal distension immediately after birth; {some cases present late with Chronic constipation}
Fail to pass meconium in first 48 hours of life.
Failure to thrive
Episodes of enterocolitis (loose stools with blood and mucus).
Signs of distal obstruction – vomiting, bloating, inability to pass gas, crampy abdominal pain etc.
PR examination – it reveals empty rectum & a gush of liquid stools on withdrawal of finger.
Investigations:
- reveals small rectum & colonic dilatation
– to confirm the absence of ganglion cells
– to detect acetylcholinesterase
- reveals failure of the rectum to relax with balloon distension
Treatment – :
 Resect the aganglionic bowel & anastomosis of normally innervated ganglionic bowel close to
the anal margin.
 Some cases present late  Do a colonostomy to relieve the obstruction  Dilated hypertrophied
proximal bowel to return to normal.
----------------------------------------------------------------------------------------------------------------------------------------
1. Plummer Vinson's syndrome [19, 17]
Ans.
Plummer–Vinson (Patterson– Brown–Kelly) Syndrome:
This occurs due to atrophy of the mucous membrane of the alimentary tract
Clinical features: dysphagia, iron-deficiency anaemia, glossitis, angular stomatitis, koilonychia
(spooning of nails) and achlorhydria.
Predominantly, it affects females past 40 years.
Barium swallow shows a web in the postcricoid region and the same can be seen on
oesophagoscopy. It is due to subepithelial fibrosis in this region.
Complications: It predisposes to the development of carcinoma in the tongue, buccal mucosa,
pharynx, oesophagus and the stomach.
Treatment
To correct anaemia by oral/parenteral iron: Treat associated B12 & B6 deficiency
Dilatation of the webbed area by oesophageal bougies.
----------------------------------------------------------------------------------------------------------------------------------------
2. Colonoscopy [19, 16]
Ans.
Colonoscopy allows good visualization of the whole colon
and terminal ileum.
Indications:
▪ Bleeding per rectum, resistant anaemia.
▪ To take biopsies from different parts of the bowel.
▪ To identify synchronous growths, ulcerative colitis.
▪ To remove polyps.
▪ When barium enema shows irregularity.
▪ For therapy – colonoscopic polypectomy,
dilatation of stricture colon, fulguration etc.
Procedure:
Complications: Perforation occurs in 1:1000
examinations but this is higher (up to 2%) after polypectomy and endoscopic mucosal resection.
----------------------------------------------------------------------------------------------------------------------------------------
3. Mention '2' contraindications & '2' complications of upper gastrointestinal endoscopy [14]
Ans.
Contraindications of Upper GI endoscopy – coagulopathies; patient in shock; uncooperative
patient; inadequately trained personnel etc.
Complications of Upper GI endoscopy – Bleeding, perforation, aspiration, airway obstruction,
cardiopulmonary complications & infection etc.
----------------------------------------------------------------------------------------------------------------------------------------
4. Aetiology of tropical sprue [13]
Ans.
Tropical sprue refers to damage of small bowel villi due to an unknown organism resulting in
malabsorption
Similar to celiac disease except:
a) Occurs in tropical regions (e.g., Caribbean)
b) Arises after infectious diarrhea and responds to antibiotics
c) Damage is most prominent in jejunum & ileum (secondary vitamin B12 or folate deficiency may ensue);
duodenum is less commonly involved.
Management: Leave the sprue area; Take Folic acid (5 mg daily) & Vit B12
 Fluids and electrolytes for dehydration
 Abx {usually tetracycline 1 g daily for up to 6 months}
----------------------------------------------------------------------------------------------------------------------------------------
5. Prokinetic drugs [08]
Ans.
These are drugs which promote GI transit and speed gastric emptying by enhancing coordinated
propulsive motility
: Metoclopramide, Domperidone, mosapride etc.
:
In constipation
To alleviate symptoms a/w gastric stasis in patients with post-op gastroparesis
To stimulate gastric emptying during GI radiological procedures
GERD: Provide symptomatic Relief
As an antiemetic: used in disease-associated vomiting, Cancer chemotherapy-induced vomiting,
Vomiting due to radiation sickness etc. {Domperidone is a preferred antiemetic in children – as it has no EPS}
:
 Metoclopramide - Extrapyramidal symptoms {EPS} are common
 Domperidone - Dryness of mouth, diarrhoea, skin rashes, galactorrhea & menstrual irregularities
----------------------------------------------------------------------------------------------------------------------------------------
6. Oesophageal stricture [07]
Ans.
The strictures (narrowing of esophageal lumen) occur when muscular coat of oesophagus is damaged
AETIOLOGY: The common causes are:
1. Burns due to corrosive substances or hot fluids.
2. Trauma to oesophageal wall due to impacted foreign bodies or instrumentation or external injuries.
3. Ulcerations due to reflux oesophagitis.
4. Ulcerations due to diphtheria or typhoid.
5. Sites of surgical anastomosis.
6. Congenital, usually in the lower third.
CLINICAL FEATURES & DIAGNOSIS
• Dysphagia, first to solids and then to liquids, is the common complaint.
• When obstruction is complete, regurgitation & cough may occur.
• Patient is malnourished.
• Barium swallow establishes the diagnosis. Oesophagoscopy is required to exclude malignancy.
TREATMENT
It should be done under direct vision through oesophagoscope.
It helps to feed the patients and give rest to the inflamed area above the strictures. After
a few days, when inflammation subsides, lumen become visible & prograde dilatation can be restored.
of strictured segment & reconstruction of food passage
----------------------------------------------------------------------------------------------------------------------------------------
7. Management of acute variceal bleeding [08]
a. Drug treatment of oesophageal varices [07]
Ans.

Management of GI haemorrhage due to


oesophageal varices

 Introduce β-blocker as secondary prophylaxis


after the “bleeding stops”
 If bleeding does not stop even after
endoscopic banding  then repeat endoscopic banding
Balloon tamponade Emergency TIPSS {Trans-jugular
intrahepatic portosystemic stent shunt}
 Balloon tamponade – This technique employs a Sengstaken–
Blakemore tube, which consists of two balloons that exert
pressure in the fundus of the stomach and in the lower
oesophagus 👉🏻
1) Barbiturate Poisoning – C/F, DDx & Mx [21, 19, 12, 09, 03]
Ans.
Barbiturates are non-selective CNS depressants.
: Barbiturates potentiate the inhibitory effects of GABA
:
 CNS: Barbiturates depress all areas of the CNS  drowsiness,
confusion, hallucinations, etc.
 CVS: hypotension
 Respiration: respiratory depression
 Renal: renal shutdown
 Others: megaloblastic anemia, bullous eruptions, convulsions, coma and death.
intoxication by other substances with sedative effects, such as benzodiazepines,
anticonvulsants (carbamazepine), alcohols (ethanol, ethylene glycol, methanol), opioids, carbon
monoxide, sleep aids,

1. Maintain airway, breathing and circulation.


2. Maintain electrolyte balance.
3. Endotracheal intubation to protect the airway in unconscious patients
4. Gastric lavage.
5. Alkaline diuresis— IV NaHCO3 (1 mEq/kg) alkalinizes urine. Barbiturates are weakly acidic drugs. In alkaline
urine, barbiturates exist in ionized form, so they are not reabsorbed while passing through renal tubules & are rapidly excreted in urine.
6. Haemodialysis and haemoperfusion (via activated charcoal) are highly effective & employed in
severe cases
----------------------------------------------------------------------------------------------------------------------------------------
2) Organophosphorous poisoning – MOA, C/F, Dx, complications & Mx [21, 19, 18, 15, 11, 08]
a. Cholinesterase re activators [17]
b. Drug therapy of organophosphorus compound poisoning [13]
Ans.
: OP compounds {attach to esteratic site of ChE & form covalent bonds} irreversibly inhibit
cholinesterases and cause accumulation of ACh at muscarinic and nicotinic sites. All organophosphorus (OP)
compounds except echothiophate have no therapeutic applications. Echothiophate is rarely used in resistant cases of glaucoma.

Muscarinic effects: Profuse sweating, salivation, irritation of eyes, lacrimation, tracheobronchial


secretions, bronchospasm, vomiting, abdominal cramp, miosis, bradycardia, hypotension, cardiac
arrhythmias, involuntary urination & defecation.
Nicotinic effects: Twitching, fasciculations, muscle weakness & paralysis (due to prolonged depolarization).
Central effects: Headache, restlessness, confusion, convulsions, coma & death are usually due to
resp. failure.
. OP poisoning can be diagnosed by:
 History of exposure.
 Characteristic signs and symptoms.
 ↓cholinesterase activity in blood.
:
:
1. Remove the Contaminated clothes; wash skin with soap & water.
2. Gastric lavage should be continued till the returning fluid is clear
3. Airway should be maintained
4. If necessary, Artificial respiration should be given
5. Supportive measures—maintain BP, hydration, Diazepam should be used cautiously by slow IV
inj. to control convulsions.
:
1. is the 1st Drug to be given in OP poisoning -
 It competitively blocks the muscarinic effects of OP compounds (competitive antagonism).
 Inject Atropine 2mg IV stat & should be repeated every 5-10 min doubling the dose, if required,
till the patient is fully atropinized (fully dilated & non-reactive pupils, tachycardia, etc.).
2. Atropine is not effective for reversal of neuromuscular paralysis.
Neuromuscular transmission can be improved by giving
cholinesterase reactivators such as pralidoxime, obidoxime, etc.
MOA: OP compounds inactivate ChE by phosphorylating esteratic
site of the enzyme. Oximes bind with high affinity to anionic site,
dephosphorylate the enzyme, and reactivate it. Early
administration of oximes is necessary before the phosphorylated enzyme undergoes aging (loses
alkyl groups) and becomes resistant to reactivation.
Pralidoxime is administered IV slowly in a dose of 1-2 g.
Oximes are ineffective as an antidote to carbamate anti-ChEs (physostigmine, neostigmine,
carbaryl, propoxur) in which case the anionic site of the enzyme is not free to provide
attachment to it. It is rather contraindicated in carbamate poisoning, because not only it does
not reactivate carbamylated enzyme, it has weak anti-ChE activity of its own.
:
Repeated exposure to OP compounds results in polyneuritis and demyelination after a latent
period of days to weeks.
Sensory disturbances occur first followed by muscle weakness, tenderness & ↓ tendon reflexes—
LMN paralysis.
In the 2nd phase, spasticity and UMN paralysis gradually supervenes. Recovery may take years.
there is no specific treatment.
----------------------------------------------------------------------------------------------------------------------------------------
3) Lead Poisoning – C/F & Mx [18, 12, 10]
a. Lead Neuropathy [05]
Ans. Lead is a cumulative poison; It inhibits sulfhydryl group of enzymes, necessary for heme synthesis

Workers in industries using lead, like hair-dye, paints. glass blowers, toys. plumbing, batteries
Automobile exhaust – MCC in cities
Food and water contaminated with lead
Children playing with toys (licking)
Absorption of sindur (vermilion) in women

Oliguria, red colored urine (coproporphyrin).


Lead Neuropathy – seen in chronic lead toxicity – wrist and foot drop {radial nerve & common
fibular nerve} – {write from ortho}
Burtonian line (lead line) – Blue line seen at the junction of gums and teeth

Diagnosis: History, Clinical features, Serum Lead > 0.1 mg/100 mL; Urinary Lead > 0.25 mg/100 mL;
Blood picture (anemia, basophilic stippling etc.); X-ray shows lead lines; Urine shows level of porphyrins
Treatment:
Stomach wash, demulcents and emetics
Calcium gluconate 10%, 10 ml IV
BAL, penicillamine, succimer is safe
Symptomatic – morphine for pain, antiemetics for vomiting
----------------------------------------------------------------------------------------------------------------------------------------
4) Cyanide Poisoning – C/F & Mx [14]
Ans.
Cyanide is the most rapidly acting poison and hence used by terrorists to commit suicide
:
Cyanide inhibits cytochrome oxidase and carbonic anhydrase enzyme  interferes with the cellular
respiration
There is formation of cyanmethemoglobin and the blood is bright red in color. Oxygen will be
available in the blood, but the tissues are unable to utilize oxygen, since cytochrome oxidase is
necessary for uptake of oxygen by the tissues
Fatal dose: 50 to 60 mg of acid; 200 to 300 mg of salt; concentration of 1:500 in air
Fatal period: Immediate by inhalation; half an hour with salts
:
CNS: Headache, vertigo, perspiration, anxiety, confusion, drowsiness, convulsions, coma and death.
GIT: Bitter burning taste, numbness, nausea and rarely vomiting.
RS: Smell of bitter almonds in breath, tachypnea, later slowing of respiration.
CVS: Tachycardia, hypotension and collapse.
Skin: Perspiration and subcutaneous bullae; pupils are dilated and there is acidosis.
– the aim is to reverse cyanide-cytochrome combination.
for 15 to 30 minutes - repeat, if necessary,
0.3 mg in 10 mL distilled water by slow IV injection  form methaemoglobin which competes
with cytochrome oxidase for cyanide ions, thus protecting cytochrome oxidase.
25 g as 15% solution by slow IV over a period of 3 minutes; it converts cyanide into nontoxic
thiocyanate. All these are repeated when necessary.
4 to 5 g by slow IV as infusion  form nontoxic cyanocobalamin, which is excreted in urine.
acts by chelating cyanide to form harmless compound and is excreted in urine.
Gastric lavage using 1:5000 potassium permanganate.
Ventilation with 100% oxygen.
----------------------------------------------------------------------------------------------------------------------------------------

1. Paracetamol Overdose/ poisoning [21, 16, 11, 04]


a. Treatment of Salicylate overdose [19]
Ans.
:
:
High doses of PCM cause depletion of GSH levels
Acute overdosage mainly causes hepatotoxicity—
symptoms are nausea, vomiting, diarrhoea,
abdominal pain, hypoglycaemia, hypotension,
hypoprothrombinaemia, coma, etc.
Alcoholics & premature infants are more prone to
hepatotoxicity
Death is usually due to hepatic necrosis.
:
» Antidote: replenishes the glutathione stores of the liver and
protects liver cells.
» Activated charcoal is administered to decrease the absorption of paracetamol from the gut
» Haemodialysis may be required in
cases with acute renal failure

:
Clinical Features:
Treatment – it is mainly symptomatic
Alkalinisation of urine in salicylate
poisoning – administer sod.
Bicarbonate
Supplemental glucose {100 ml of 50%
dextrose} – to overcome
neuroglycopenia.
Hemodialysis – in severe cases
----------------------------------------------------------------------------------------------------------------------------------------
2. Treatment of snake bite [21, 13, 12, 09]
a. Anti-Snake Venom [16]
b. Treatment of vasculotoxic snake bite [11]
c. Poisonous snake bite-clinical features [05]
d. Renal manifestations in Snake bite [04]
e. Types of Snake Venoms [03]
Ans.

:
1. Neurotoxic – Elapidae {cobras, mamba, kraits} – causes muscle paralysis
2. Haemotoxic – Viperidae {Russel’s viper, Pit viper, Saw-scaled viper} – causes hemolysis [vasculotoxic]
3. Myotoxic – Hydrophidae {Sea snakes} – causes muscle pain & myoglobinuria
– seen in all snake bites  fright, fear, anxiety, cold clammy skin, rapid breathing,
shock & rarely death
Local effects Systemic Effects
▪ Salivation, vomiting, difficulty in swallowing &
Features of Burning pain,
speaking;
Neurotoxic – redness, swelling &
▪ Paralysis of lower limb;
Elapidae regional lymphangitis
▪ Death is due to rep. muscle paralysis
 Salivation, vomiting,
Intense pain; oozing
 Petechial hemorrhages on mucous membranes of organs;
Features of of blood; Blisters
 Bleeding from nose, ears etc. Hemoptysis;
Haemotoxic – filled with serosanguinous
 Intracranial hemorrhages; subconjunctival hemorrhages
Viperidae fluid; Local necrosis;
 Hematuria, renal failure
& regional lymphangitis
 Hypotension, shock & death
Features of ▪ Nausea, vomiting;
Mild pain &
Myotoxic – ▪ Pain, stiffness, weakness & flaccid paralysis of skeletal muscle;
swelling
Hydrophidae ▪ Myoglobinuria, renal failure, shock & death
: it neutralizes the toxic effects of snake venom. It is of two types:
a) Monovalent – effective against one specific snake
b) Polyvalent – effective against cobra, krait, Russel’s viper & saw-scaled viper
:
:
Alleviate anxiety & fear {MCC of death is fear}
Wash the wound with soap and water / KMNO4
Immobilize the limb – to prevent spread of venom
Sutherland wrap is applied to occlude veins & lymphatics {except in viper bite to avoid local necrosis}
:
Secure IV line
Remove Sutherland bandage
Inj. TT, 0.5 mL IM
Inj. { olyvalent nti-snake enom} – to neutralize the toxic effects of snake venom
Dosage: lyophilized powder is diluted in 500 ml normal saline and infused in 1 hour.
Minimal symptoms (only local signs, no systemic signs) – 5 Vials
Moderate (moderate local & systemic signs) – 10 Vials
Severe (severe local & systemic signs) – 15 Vials
If signs of paralysis seen  Inj. Neostigmine 1.5 mg
Inj. Atropine 0.6 mg to counteract muscarinic effects of Neostigmine
If clotting abnormalities seen – Inj. Heparin 1000-5000 units
Antibiotics, Pain killers {PCM} & Surgical debridement of wound
Hemodialysis and other supportive measures
----------------------------------------------------------------------------------------------------------------------------------------
3. General management of poisoned patient [17, 13, 08]
a. Charcoal tablets [06]
Ans.
Hospitalization
Clear Airway.
In comatose patients, there is danger of respiratory obstruction by tongue, secretions and aspiration of
vomitus. Hence, patient should be turned to his left lateral side.
A cuffed endotracheal tube should be inserted and secretions should be aspirated regularly.
Assess Breathing – If there is hypoxaemia, give O2 or mechanical ventilation if there is respiratory insufficiency.
Assess Circulation (HR & BP) and an i.v. (intravenous) line should be maintained.
To prevent further absorption of poison:
a) Inhaled poisons (gases) – Patient should be moved to fresh air.
b) Contact poisons – Contaminated clothes should be removed & the body part should be washed
with soap and water.
c) Ingested poisons –
Gastric lavage within 2-3 h of poisoning {avoid in children, corrosives, convulsant, comatose etc.}  repeat
lavage till the returning fluid is clear  Give activated charcoal. It adsorbs many drugs and poisons
(physical antagonism).
Activated charcoal has a large surface area & is highly porous - bind with poisonous material.
Emetics – Mustard, common salt, syrup ipecac, etc. – to induce vomiting
Laxatives – Magnesium sulphate or citrate can be used orally to promote elimination of the
ingested poison.
To promote elimination of absorbed portion of the drugs:
Diuretics (i.v. mannitol or furosemide)
Alter the pH of urine, e.g., alkalinisation of urine in salicylate poisoning & acidification of urine
in amphetamine poisoning.
Dialysis is used in cases of severe poisoning, e.g., lithium, aspirin, methanol, etc.
Symptomatic treatment: IV diazepam 5-10 mg if there are convulsions and external cooling for
hyperpyrexia.
Maintenance of fluid and electrolyte balance:
Hyponatraemia – treat with i.v. normal saline
Hypernatremia – treat with i.v. furosemide.
Hypokalaemia – treat with potassium chloride, oral or slow i.v. infusion (it has cardiac depressant effect).
For mild hyperkalaemia – use Thiazides or furosemide.
For Severe hyperkalaemia – use 10% calcium gluconate intravenously. is used to treat.
Metabolic acidosis – treat with Intravenous sodium bicarbonate
----------------------------------------------------------------------------------------------------------------------------------------
4. Arsenic poisoning – signs, symptoms & Mx [17]
Ans.
Pure arsenic is non-poisonous; Arsenic SALTS are POISONOUS; It is deposited in hair nails & bones
Used as suicidal, homicidal, abortifacient, cattle poison, aphrodisiac (to increase vigour and vitality) etc.
: It blocks the sulfhydryl enzymes in the cells  Interferes cell metabolism.
Also acts as local irritant.
:
:
Gastroenteric type – garlicy odor in breath, rice watery stools, abdominal pain, hair loss,
circulatory failure & renal failure
Narcotic type: GI symptoms are minimal; there is giddiness, formication, delirium, coma & death.
Skin: Brown colored, pin point pigmentation (melanosis) of the skin – known as "rain drop
appearance” seen over eyelids, temples, neck
Milk and roses complexion – due to vasodilatation
Soles and palms - hyperkeratosis
Nails become brittle, transverse opaque bands (Aldrich Mees lines) appears on finger nails. It
indicates period of supressed growth of nail
Hair loss
Eyes - conjunctivitis, photophobia
G.I inflammation and hepatorenal damage
Features of cardiac, respiratory & renal
abnormalities
:
Gastric lavage
Demulcents like butter milk
Ferric oxide orally convert arsenic into harmless ferric arsenite
Chelating agents:
▪ BAL 3 mg/kg deep IM 4th hourly, for 7-10 days
▪ DMSA (succimer) can be given, it is superior to BAL.
Supportive measures like hemodialysis I.V fluids
----------------------------------------------------------------------------------------------------------------------------------------
5. Chelating agents [16, 12, 09]
a. Mention four chelating agents and their use [13]
Ans. Chelating agents combine with metallic ions and form ring structures that are water-soluble
complexes and are rapidly excreted from the body. These agents are used in heavy metal poisoning.

Dimercaprol (British anti-Lewisite; BAL).

Disodium edetate (Na2EDTA) In hypercalcaemia, as an anticoagulant in vitro


Calcium disodium edetate (CaNa2EDTA). Lead, Zn, Mn, Cu poisoning
Cu, Hg, Zn, lead poisoning
D -Penicillamine Other uses: Wilson’s disease, scleroderma, cystinuria
& rheumatoid arthritis
Desferrioxamine. Iron poisoning
Transfusion siderosis in thalassemia, acute iron
Deferiprone.
poisoning
----------------------------------------------------------------------------------------------------------------------------------------
6. Aluminium phosphate poisoning [12, 07]
a. Aluminium Phosphide Poisoning [18, 15, 07]
Ans.

: When exposed to air and moisture, aluminium phosphide


liberates phosphine which causes multi-organ damage. AlP + 3H2O→
Al(OH)3 + PH3
:
Metallic taste, vomiting, garlicky (or fishy) odour of breath, intense thirst, burning epigastric pain,
and diarrhoea
Respiratory distress with cyanosis, and cold, clammy skin
Massive focal myocardial injury with serum levels of cardiac enzymes
In severe cases, there will be hypotension, cardiac abnormalities, hepatic damage, renal failure,
metabolic acidosis, convulsions & coma
:
Garlicky odour in the breath
Abnormal urinalysis, LFTs & RFTs.
ECG abnormalities: sinus arrhythmia with ST segment depression in lead II, III, and AVF, ST
elevation, atrial fibrillation, T wave inversion in V5-6, sinus arrest, chaotic atrial pacemaker,
complete heart block, bundle branch block, and ventricular premature complexes followed by
ventricular tachycardia
Test for phosphine in the breath and gastric aspirate
:
Though there is intense thirst, do not administer water since whatever aluminium phosphide is still remaining in the
stomach will react with it, releasing phosphine. Gastric lavage is contraindicated.
Administer Activated charcoal
Manage circulatory shock with IV fluids with Dopamine
Manage respiratory distress with 100% humidified oxygen, intubation, and assisted ventilation
Manage metabolic acidosis with sodium bicarbonate (50 mEq/15 min)
Control of convulsions with anticonvulsants (benzodiazepines, barbiturates, etc.).
Magnesium sulfate therapy – can be used for cardiac arrhythmias.
Ranitidine 50 mg IV 8th hourly to counter the severe epigastric pain
----------------------------------------------------------------------------------------------------------------------------------------
7. Complications of copper sulphate poisoning [07]
Ans.
:
Chronic inhalation of copper sulfate spray used as an insecticide in vineyards can cause vineyard
sprayer’s lung disease  histiocytic granulomatous lung. Liver damage is also common
Green hair discolouration – in swimming pool water with algicidal copper chemicals
Chronic copper toxicity is the hallmark of Wilson’s disease, an autosomal recessive genetic disorder
in which there is deficiency of caeruloplasmin.
Discolouration of the peripheral part of the cornea ( ) is pathognomonic
A ‘sunflower-like’ discoloration of the most anterior layers of the lens has also been reported
Skin exposure can produce severe irritation, itching, erythema, dermatitis and eczema
: Serum ceruloplasmin level & Blood copper level
:
 Induced emesis is contraindicated.
 Stomach wash using potassium ferrocyanide – converts copper into cupric ferrocyanide.
 Administration of egg white or milk orally – helps in detoxifying copper.
 Hemodialysis is useful in early stages.
 In Severe cases – chelation with dimercaprol 2.5 mg/kg, 4th hourly; followed by oral penicillamine.
----------------------------------------------------------------------------------------------------------------------------------------
8. Scorpion sting [2000]
Ans.

----------------------------------------------------------------------------------------------------------------------------------------

1. Yellow oleander poisoning [22]


Ans.
All parts of yellow oleander are poisonous
Contains thevetin, thevetoxin, cerberin etc.  Acts directly on the heart
Clinical features:
 Heart – bradycardia, AV block, ectopics & arrhythmias etc.
 Death is due to circulatory failure
 CNS – dilated pupils, giddiness, fainting, muscle weakness, convulsions etc.
 GIT irritation – NVD, burning sensation etc.
Treatment:
Gastric lavage
Symptomatic & supportive treatment:
Atropine for bradycardia
Lignocaine for arrhythmias
Fab is given in severe cases
----------------------------------------------------------------------------------------------------------------------------------------
2. Carbon-monoxide poisoning [14]
Ans.
:
 Binds to Hb  form CO-Hb   oxygen content of blood & tissues. Thus, it acts as a chemical
asphyxiant
 It also causes direct toxic effect on cellular cytochromes and heart cells
: Signs and symptoms are those of hypoxia
Mild poisoning {CO-Hb <30%}: Nausea, vomiting, headache, dyspnea on exertion, flushed skin
(cherry red)
Moderate poisoning {CO-Hb 30 to 40%}: (Above features) + Buzzing in ears, Blurred vision, Severe
throbbing headache, Increasing dyspnea, tachypnea, Drowsiness, hallucinations & cardiovascular
toxicity.
Severe Poisoning {CO-Hb > 40%}: (Above features) + Respiratory paralysis, Myocardial ischemia,
hypotension, Muscle spasms, staggering gait, Skin blisters, Albumin and glucose in urine &
Convulsions, coma and death.
Carbon monoxide automatism: The victim before dying has wild and erratic swinging movements,
which disturbs clothing and surroundings. It gives a false impression of violent struggle and
homicide.
:
Shift the person from source of carbon monoxide to fresh air.
Symptomatic treatment:
Hyperbaric oxygen; 100% oxygen – to correct hypoxia
Mannitol – for headache
Antibiotics to prevent pneumonia
Whole blood transfusion in severe cases
----------------------------------------------------------------------------------------------------------------------------------------
3. Ocular drug toxicity [10]
Ans.

----------------------------------------------------------------------------------------------------------------------------------------
4. Botulinum toxin [05]
Ans.
Botulinum toxin is a highly potent exotoxins produced by Clostridium botulinum.
: Inhibit the release of cholinergic transmission
:
Localized injection of minute quantity of botulinum toxin A (BOTOX) or its haemagglutinin
complex (DYSPORT) is used in the treatment of a number of spastic and other neurological
conditions like blepharospasm, spastic cerebral palsy, strabismus, spasmodic torticollis,
nystagmus, hemifacial spasm, post stroke spasticity, spasmodic dysphonia, axillary
hyperhidrosis, etc. which are due to overactivity of cholinergic nerves.
It is being employed popularly as beauty treatment for removal of age-related facial wrinkles.
: occur due to incorrect injection or overdose  ptosis, diplopia, facial swelling, dry
mouth, dysphagia, dysarthria, muscular weakness and even respiratory paralysis has occurred
1. Rheumatoid Arthritis – Etiopathogenesis, C/F (articular & extra articular manifestations) inv. & Mx
[22, 17, 14, 13, 10, 08, 05]
a. Extra articular features of rheumatoid arthritis (incl 4) [21, 15]
b. Methotrexate: its role in Rheumatoid arthritis [21, 11]
c. Treatment of Rheumatoid Arthritis [19, 12]
d. Diagnostic criteria for rheumatoid arthritis [10, 07, 05]
e. Disease modifying Anti Rheumatoid drugs [08]
f. Eye Manifestations (Eye signs) in Rheumatoid arthritis [06, 03]
Ans.
Rheumatoid Arthritis is a systemic autoimmune disease characterised by inflammatory polyarthritis,
which affects peripheral joints, especially the small joints of the hands & feet

RA might be a manifestation of the response to an infectious agent (Mycoplasma, EBV, CMV,


parvovirus, and rubella virus) in a genetically susceptible host.
Most patients with RA are HLA-DR4 +ve.
Risk factors: Female gender and cigarette smoking

RA is characterized by chronic inflammation, granuloma formation and joint destruction.


Lymphocytes (CD4+ T cells), plasma cells and macrophages infiltrate the synovial membrane 
Swelling and congestion ➔ release of cytokines➔B cells differentiate into plasma cells➔
antibodies against the Fe fragment of IgG which is termed as the rheumatoid factor.
Inflammation of synovium➔ synovial hypertrophy and effusion of synovial fluid into the joint
space  joint swelling.
Inflammatory granulation tissue (pannus) forms  spreads around & damage the articular cartilage
➔ fibrous or bony ankylosis.
Muscles adjacent to inflamed joints atrophy and there may be focal infiltration with lymphocytes.
Subcutaneous nodules consist of a central area of fibrinoid necrosis surrounded by radially
arranged (palisade) mononuclear cells with strands of collagen.
Rheumatoid vasculitis involves medium and small arteries and venules, with infiltration by
lymphocytes.
Age of onset & Sex: 20 - 50 y/o females.
Typical presentation: An acute, symmetrical polyarthritis: Pain & stiffness in multiple joints
(at least 4), particularly in the morning, mark the
beginning of the disease. This may be followed by • MP joints of hand
frank symptoms of articular inflammation. Joints Common • PIP joints of fingers
affected in Rheumatoid arthritis are: 👉🏻 • Wrists, knees, elbows, ankles
The onset may be with fever or visceral Less
Hip joint & TMJ
manifestations like: pneumonitis, rheumatoid common
nodules etc. • Atlanto-axial joint
Uncommon
• Facet joints of cervical spine
Examination – reveals
Ulnar drift of the hand
swollen boggy joints. Hand Boutonniere deformity
Joints of the hand Swan neck deformity
show typical Elbow Flexion deformity
deformities. 👉🏻
Early – flexion deformity
Limited ROM Knee
Late – triple subluxation
There may be fever,
Ankle Equinus deformity
rash & signs
Foot Hallux valgus, Hammer toe
suggestive of systemic
vasculitis.
The rash in rheumatoid arthritis is typically non-pruritic
and maculo-papular on the face, trunk and extremities.
Extra-articular manifestatiosn 👉🏻

Investigations:
The goals of treatment are: (1) Relief of pain, (2)
Reduction of inflammation, (3) Protection of articular
structures, (4) Maintenance of function, and (5) Control
of systemic involvement.
AIDS
Analgesics – to pain – Ex: Acetaminophen (PCM), &
NSAIDs (diclofenac, ibuprofen, aceclofenac)
DMARDs (disease-modifying antirheumatic drugs) – to
 joint damage – Ex: hydroxychloroquine (200-400 mg daily),
methotrexate (7.5-25 mg per week), sulfasalazine,
leflunomide, gold salts etc.
Steroids – to inflammation – Ex: prednisolone, triamcinolone, etc.
Immunosuppressive agents - used as 3rd line drugs – Ex: azathioprine and cyclophosphamide
Biological response modifiers AIRE
 Etanercept (subcut. Inj.)– TNF receptor blocker – used in many forms of inflammatory arthritis
like RA, psoriatic arthritis, and ankylosing spondylitis.
 Infliximab – antibody against TNF - given as IV infusion every six weeks
 Rituximab – anti-CD20 antibody.
 Anakinra – interleukin-I receptor antagonist

Patient education and counselling – Explain about the chronic nature of disease, the need for long-
term therapy, the side effects of drugs & the need for follow-up stressed.
Rest – during acute arthritis and during flare ups.
Exercise – to improve joint mobility, contractures, and muscle atrophy.
Physiotherapy – application of heat or cold to relieve pain or stiffness; ultrasound to tenosynovitis,
passive and active exercises to improve and maintain joint motion range.
– indicated when medical therapy fails
▪ Synovectomy of the wrist or finger tendon sheaths – for pain relief or to prevent tendon rupture.
▪ Osteotomy, arthrodesis or arthroplasties – done in advanced disease.
----------------------------------------------------------------------------------------------------------------------------------------
2. Gout – etiopath, C/F, Dx, complications & Mx [20, 18, 17, 16, 12, 10]
a. Treatment of Acute Gout [21]
b. Clinical features and treatment of acute Gouty Arthritis [19, 13]
c. Tophi [10, 05, 04]
d. Medical management of gout [08]
Ans:
Gout is a Crystal-induced arthritis with monosodium urate
(MSU) within & around joints.
Clinical Features – 3 types of clinical presentation:

Sudden onset of severe pain, redness, and swelling of the


affected joints a/w fever, malaise and even confusion
Usually involves single distal joint – MC is base of great toe
{1st metatarsophalangeal joint}, known as podagra
Other common sites (in order of decreasing frequency) are the ankle, midfoot, knee, small joints of
hands, wrist and elbow
Complete resolution occurs within a few days to several weeks
As the attack subsides, pruritus and desquamation of overlying skin
: After an acute attack some people have a 2nd attack and the
frequency of attacks gradually increases with time joint damage and chronic pain

It is characterized by collections of solid urates in connective tissues


These collections produce irregular painless firm nodules ('tophi') around extensor surfaces of
fingers, hands, forearm, elbows, Achilles tendons and sometimes the helix of the ear.
They may be clinically visible or detected by imaging
Large tophi may ulcerate, discharging white gritty material
Diagnosis of gout:
 History & physical examination
 Synovial fluid analysis – reveals MSU crystals – GOLD
standard
 Serum urate levels – neither sensitive nor specific
DDx: CPPD (pseudogout), rheumatoid arthritis,
osteoarthritis, septic arthritis, psoriatic arthritis etc.
Management of gout: Management should focus on
first dealing with the acute attack and then giving
prophylaxis to lower SUA and prevent further attacks

----------------------------------------------------------------------------------------------------------------------------------------
3. Define bone density. How is it measured? Osteoporosis – causes, C/F, Dx & Mx [20, 18, 17, 11, 15]
Ans.
Bone density: Bone density is a measure of the amount of minerals (mainly calcium and
phosphorous) contained in a certain volume of bone.
Measurement of bone density: , DEXA, Single energy x-ray absorptiometry, Ultrasound

Osteoporosis is characterized by a decrease in the amount of bone.


The rate of bone formation is often normal, whereas the
rate of bone resorption is increased
Osteoporosis is a/w risk of fractures especially the
spine and hip. {Pathological Fractures}

X-rays show osteopenia.


Biochemistry:
Total plasma proteins and plasma albumin may be low
Serum calcium, phosphates and alkaline phosphatase are within normal limits.
: This is a method to quantify osteoporosis.
In this method absorption of photons (emitted from gamma emitting isotopes) by the bone
calcium is measured.
2 types of bone densitometry are available – ultrasound based and X-ray based.
DEXA scan is an X-ray based bone densitometry, and is the gold standard in the quantification of
bone mass.
Neutron activation analysis: In this method, calcium in the bone is activated by neutron bombing,
and its activity is measured.
Bone biopsy

High protein diet: ↑protein intake may ↑ the formation of organic matrix of the bone.
Calcium supplementation & Fluoride supplementation
Androgens they have an anabolic effect on the protein matrix of bone
Estrogens: they halt the progressive loss of bone mass in postmenopausal osteoporosis.
Vitamin D: to increase calcium absorption from the gut.
Calcitonin helps in building up the bone mass and also acts as an analgesic
Alendronate once a day on empty stomach.
Teriparatide Anabolic agent increasing osteoblastic new bone formation.
Antiresorptive agents: Denosumab, Strontium
: This consists of the following:
Weight bearing is a major stimulus to bone formation.
esp. to prevent pathological fractures in a severely osteoporotic spine.
----------------------------------------------------------------------------------------------------------------------------------------

1) Ankylosing spondylitis [22, 13, 08, 04]


a. Two Extra Articular Manifestations of Ankylosing Spondylitis [21]
b. Clinical features of Ankylosis spondylitis [21]
Ans:
Ankylosing spondylitis is a chronic disease characterised by a progressive inflammatory stiffening of
the joints, with a predilection for the joints of the axial skeleton, especially the sacroiliac joints.
Etiopathology
 Genetic Factors: .
 Pathology:
Site: Sacro-iliac joints are the 1st to get involved; followed by the spine from the lumbar region
upwards. The hip, the knee and the manubrio-sternal joints are also involved frequently.
Initially synovitis occurs➔cartilage destruction and bony erosion➔fibrosis➔ fibrous ankylosis
(stiff joint)➔bony ankylosis.
After bony fusion occurs, the pain may subside, leaving the spine permanently stiff (burnt out disease).
Clinical features
It characteristically affects young adults with a peak age of onset between 20 and 30 years.
male: female ratio 3:1
Articular Features:
▪ Insidious onset of and .
▪ As the disease progresses, spine becomes progressively ankylosed, spinal rigidity and secondary
osteoporosis predispose to spinal fracture.
▪ shows restricted spinal mobility in all directions, pain on sacroiliac compression,
diminished chest expansion and stooped posture.
Extra-articular Features:
inflammation in tendon or ligament insertion sites (Achille’s tendon, iliac crest and
greater trochanter)
Eye – Anterior uveitis
Prostatitis (80% men).
CVS – aortic regurgitation, pericarditis
GIT – mucosal ulcers.
Investigations
ESR and CRP
Rheumatoid factor is negative or present in low titre.
Imaging:
• X-ray of sacroiliac joint reveal irregularity and loss of cortical margins, widening of the joint
space, sclerosis, narrowing and fusion.
• X-ray of spine – Bamboo spine
• MRI – shows erosion or narrowing of joint space (early) or Joint fusion (later on)
Genetic testing for HLA B27.
Management:
Non-pharmacological Measures: Education of the patient about the disease, Regular daily back
extension exercises, including a morning ·warm-up' routine, Avoid prolonged periods of inactivity.
Poor bed and chair posture must be avoided.
Drug Therapy: to relieve pain and stiffness
» NSAIDs: A long-acting NSAID at night – Ex: Ibuprofen.
» Antirheumatic drugs {DMARDs}: sulfasalazine, MTX or azathioprine
» Anti-TNF agents: etanercept, infliximab, and adalimumab
» Steroids – local corticosteroid injections in persistent plantar fasciitis and Enthesitis.
Surgery – mainly done for severe hip, knee or shoulder stiffness.
Total hip replacement (total hip arthroplasty)
Cervical fusion for atlantoaxial subluxation, &
Wedge osteotomy for severe flexion deformities of the spine
----------------------------------------------------------------------------------------------------------------------------------------
2) Usefulness of Reticulocyte count [16]
Ans:

Reticulocytes are immature, non-nucleated


RBCs released from bone marrow.
Normal reticulocyte count: 0.5– 2.5% and
in newborn (cord blood) it is 1– 7.0%.

----------------------------------------------------------------------------------------------------------------------------------------
3) Paget's disease of bone & it’s treatment [16, 03]
Ans:
Paget’s Disease of Bone {Osteitis deformans} is a condition characterised by a progressive tendency
for one or more bones to bend, get thickened and spongy.
– Idiopathic; mostly triggered by infections like measles etc.
Due to defect in the osteoclast   irregular bone resorption & bone turnover
: Tibia is most commonly affected – the bone is soft in the initial stages, but becomes dense
and hard later.
: The disease begins after 40 years of age.
Initial stages – asymptomatic
In late stages – misshapen bones can:
▪ Impinge on nerves  pain; dull in nature
▪ Impinge on optic nerve  vision loss
▪ Overgrow  leontiasis (lion facies);
▪ Narrow auditory foramen  hearing loss
▪ Kyphosis, pelvic asymmetry & lower limb
weakness etc.
:
 X-rays show multiple confluent lytic areas with interspersed new bone formation.
 Bone scan shows an increased uptake.
 Serum alkaline phosphatase is elevated.
pathological fracture; Osteosarcoma.
is by calcitonin or bisphosphonate.
----------------------------------------------------------------------------------------------------------------------------------------
4) Osteoarthritis – etiopath & C/F [14, 09]
Ans: Refer Ortho pg no. 72

----------------------------------------------------------------------------------------------------------------------------------------
5) Osteomalacia [12, 2000]
Ans.Osteomalacia is a metabolic bone disease characterised by inadequate mineralisation of bone.
Causes:
Vitamin D deficiency: Deficient intake, Malabsorption, Inadequate sunlight exposure, Loss of
vitamin D binding protein (nephrotic syndrome), Defective 25-hydroxylation (e.g. cirrhosis of liver).
Defective 1-alpha 25chydroxylation (e.g., renal failure), Vitamin D-resistance
Mineralization defects: Abnormal matrix, Chronic renal failure, Inhibitors of mineralization
(fluoride, aluminium, bisphosphonates)
Phosphate deficiency: Decreased intake, Antacids, Impaired renal reabsorption (e.g., Fanconi
syndrome)
Clinical Features
It can be asymptomatic and present radiologically as osteopenia.
Symptoms: diffuse bone pain & proximal muscle weakness.
Investigations
Radiologic findings: Reduced bone density with thinning of the cortex.
Changes in vertebral bodies – the codfish vertebrae.
Looser zones – these are pseudofractures & are the characteristic finding in osteomalacia.
They are usually found at the femoral neck, femoral shaft, and the pubic and ischial rami.
The term "Milkman syndrome" refers to the combination of multiple, bilateral and symmetric
pseudofractures in a patient with osteomalacia.
Patients with vitamin D deficiency have 25-hydroxyvitamin D, hypophosphatemia, & Ca
concentration.
Serum phosphorus level may also be ↓ in hypophosphatemic rickets.
ALP is  in hypophosphatemic osteomalacia and normal or  in hypocalcemic osteomalacia.
Treatment
Underlying cause should be treated.
Correction of hypophosphatemia, hypocalcemia, and vitamin D deficiency.
----------------------------------------------------------------------------------------------------------------------------------------
6) Rickets [04] ➔ refer pg. no. 18 in Paeds Notes
a. Biochemical and Radiological features of Nutritional Rickets [05]
----------------------------------------------------------------------------------------------------------------------------------------
7) Rheumatoid factor [11, 05]
Ans: Rheumatoid factor (RF) is an autoantibody directed against the Fe portion of IgG.
Rheumatoid arthritis, Sjogren's syndrome, Mixed
connective tissue disease, Mixed cryoglobulinemia, SLE, Polymyositis/dermatomyositis
RF is not diagnostic of rheumatoid arthritis {since it’s not specific}.
RF is used as a prognostic marker – high RF indicates poor prognosis & vice versa
Chronic infections {subacute bacterial endocarditis, hepatitis B or C virus
infection), Sarcoidosis, Malignancy, Primary biliary cirrhosis.
RF can be positive in up to 4% of healthy individuals.
----------------------------------------------------------------------------------------------------------------------------------------
8) Polymyositis [10]
Ans: The term polymyositis refers to the idiopathic inflammatory myopathy characterized by the
presence of muscle inflammation (myositis) and weakness. It spares the skin.
Clinical Features – more common in females (2: 1 ratio); between 40 and 60 years of age

» The MC presentation is with symmetrical proximal muscle weakness, usually affecting the lower
limbs first.
 Difficulty in getting up from sitting position and climbing stairs.
 Muscle pain may be present.
 Dyspnea and aspiration (if Respiratory or pharyngeal muscles are involved).
» Systemic features such as fever, weight loss and fatigue may be present.
Diagnosis
Creatine kinase (CK)
Electromyography (EMG) to confirm myopathy and exclude neuropathy.
Muscle biopsy shows features of inflammation, necrosis, and regeneration.
MRI is useful to identify areas of abnormal muscle for biopsy.
CT scans of chest/abdomen/pelvis, and mammography to rule out any underlying malignancy.
Management

➢ Steroids: Oral steroids (e.g., prednisolone 40-60 mg daily) are the mainstay of treatment.
Intravenous steroids (methylprednisolone 1 g daily for 3 days) – if there is respiratory or
pharyngeal weakness.
➢ Immunosuppressive agents – Ex: Azathioprine, MTX, Cyclosporin, cyclophosphamide etc.
➢ General measures: Physiotherapy, avoidance of sunlight, prevention of opportunistic infections, etc.
----------------------------------------------------------------------------------------------------------------------------------------
9) Pyogenic arthritis [05]
Ans. Also known as septic arthritis, infective arthritis or suppurative arthritis.

Predisposing factors: poor hygiene, poor resistance, diabetes etc.


Causative organism: Staphylococcus aureus (MC), Streptococcus Pneumococcus and Gonococcus.
 The organisms reach the joint by one of the following routes:
▪ via blood – from a primary focus of infection like pyoderma, throat infection, septicaemia etc.
▪ Secondary to nearby osteomyelitis – seen in joints with intraarticular metaphysis e.g., the hip,
shoulder etc.
▪ Penetrating wounds, Iatrogenic (intra-articular procedure)
▪ Umbilical cord sepsis in infants can travel to joints.
 Bacteria reaches the joint inflammatory response in the synovium  exudation of fluid within
the joint & destruction of joint cartilage  complete loss of joint movement (ankylosis).
The knee is the commonest joint affected {MONOARTHRITIS}
Typically seen in child – severe throbbing pain, swelling and redness of the affected joint.
The child is unable to use the affected limb
High grade fever and malaise will be there – septic arthritis is an emergency
On examination: The child is generally severely toxic with high temperature and tachycardia.
The affected joint is swollen and held in the position of ease
Palpation reveals warm & tender joint with severe limitation of movements
: Diagnosis is mainly clinical.
Radiological Examination {X-rays} –  joint space & a soft tissue shadow corresponding to the
distended capsule due to swelling of the joint.
Ultrasound – to detect collections in deep joints such as the hip and shoulder
CBP – neutrophils; ESR
Blood culture & Joint aspiration – to identify the causative organism.

Broad-spectrum antibiotics should be started by parenteral route {Ceftriaxone + Cloxacillin}. After


culture report  start the specific Abx for 6 weeks
The joint must be put to rest in a splint or in traction.
In late cases, with extensive destruction of the joints, it is not possible to expect joint movement. In
such cases, after an arthrotomy and extensive debridement of the joint, it is immobilised in the
position of optimum function, so that as the disease heals, ankylosis occurs in that position.
----------------------------------------------------------------------------------------------------------------------------------------
10) Behcet's syndrome [02]
Ans.
Behçet’s disease is a chronic, relapsing, autoimmune disease characterized by triple-symptom complex
of recurrent oral aphthous ulcers, genital ulcers, and uveitis
Etiology – Idiopathic;
HLAB51 is associated with some cases.
a/w Immunologic (including autoimmune) and viral or bacterial triggers
Clinical Features: Behcet’s disease involves multiple systems – MC feature is the presence of recurrent
mucocutaneous ulcers.
Aphthous ulcers in mouth; Genital ulcers;
Skin lesions - ex: erythema nodosum
Ocular disease - Uveitis is the main featuremay lead to blindness if untreated.
Vasculitis; arthritis, meningoencephalitis, epididymitis, intestinal ulcerations, renal, cardiac and
lung involvement.
Treatment
Topical steroids for oral ulcers
Thalidomide - for resistant oral and genital ulceration - but is teratogenic and neurotoxic.
Colchicine is effective for erythema nodosum and arthralgia.
Systemic disease requires oral steroids with other immunosuppressive drugs.
----------------------------------------------------------------------------------------------------------------------------------------
11) Reactive arthritis [02]
Ans. Reactive arthritis refers to an arthritis that is associated with a recent or co-existing extra-articular
infection usually gastrointestinal or genitourinary infections.
Etiology
Genetic predisposition: HLA - B27
Genitourinary infections: Chlamydia, N. gonorrhea
Gastrointestinal infections: Salmonella, Shigella, Campylobacter and Yersinia.
Clinical Features
Reactive arthritis commonly affects young men (sex ratio 15: l) aged 16-35 years.
Presentation is usually acute onset oligoarthritic affecting the large and small joints of the lower
limbs 1 to 3 weeks following sexual exposure or an attack of dysentery.
Symptoms and signs of urethritis or conjunctivitis
Extra-articular features are Achilles’ tendonitis, plantar fasciitis, circinate balanitis, keratoderma
blennorrhagica, nail dystrophy and buccal ulcers.
Systemic features like fever, fatigue and weight loss can occur.
Investigations - ESR & CRP; Normocytic, normochromic anemia.
» Synovial fluid analysis shows features of inflammation such as low viscosity, turbid appearance and
presence of giant macrophages (Reiter's cells).
» Urine culture may show N. gonorrhea or Chlamydia.
» Serologic testing - to detect antibodies against the causative organism
» Rheumatoid factor and ANA are negative.
» Radiographic features: Initially there are no changes.
 With chronic disease, periarticular osteopenia, joint space narrowing and marginal proliferative
erosions may develop.
Management
NSAIDs & Steroids
Antirheumatic drugs: sulfasalazine or methotrexate
Antibiotics are used to treat acute infection. Chlamydia urethritis is treated with doxycycline.
----------------------------------------------------------------------------------------------------------------------------------------
12) Still's disease [2000]
Ans: Juvenile idiopathic arthritis (JIA) is a group of rheumatic diseases that begins at or before age 16.
Systemic JIA (Still’s disease):
This is the least common form.
Here there is arthritis, skin rash and intermittent fever.
It is called adult-onset Still's disease if it occurs after the age of 16.
Clinical Presentations:
 Child with high spiking fevers, rashes leukocytosis, and anemia.
 Rash is macular, pink - MC in the axilla and waist, but may be present anywhere on the body.
 Other features are splenomegaly, hepatomegaly, lymphadenopathy, pericarditis, pleural
effusion, etc.
Treatment: similar to rheumatoid arthritis.
DMARDs: methotrexate, hydroxychloroquine
Biologic agents (e.g., etanercept, anakinra)
NSAIDs are used to control pain and inflammation.
Corticosteroids are useful in severe disease.
----------------------------------------------------------------------------------------------------------------------------------------

1. Hypertrophic Pulmonary Osteopathy [18]


Ans.
Hypertrophic pulmonary osteoarthropathy (HPOA) is a paraneoplastic syndrome characterised by
clubbing and painful swelling of the limbs, periosteal new bone formation and arthralgia/arthritis.
Causes: Bronchial carcinoma and mesothelioma are MCC.
Clinical Presentation:
This is a painful periostitis of the distal tibia,
fibula, radius and ulna.
While most frequently associated with lung
cancer, HPOA can occur with other tumours
Investigations
Bone scans show increased periosteal uptake
before new bone is apparent on X-ray.
X-rays reveal subperiosteal new bone formation.
----------------------------------------------------------------------------------------------------------------------------------------
2. Causes of Sero Negative Arthritis [15]
Ans.
----------------------------------------------------------------------------------------------------------------------------------------
3. Pseudogout [14]
Ans. Pseudogout is a form of arthritis that develops in some people in response to the presence of
calcium pyrophosphate (CPP) crystals.
 Symptoms like those of gout
 X-rays of the knee - reveals Meniscus calcification.
Treatment:
Aspiration of joint effusion quickly reduces pain and may alone be sufficient.
Intra-articular injection of corticosteroid.
Oral NSAIDs and colchicine
----------------------------------------------------------------------------------------------------------------------------------------
4. Nodal Osteoarthritis [09]
Ans. More common in women
1. Chronic Myeloid Leukemia – C/F, Lab inv., Cytogenetic & Chromosomal abnormalities & Mx [22, 20, 18,
16, 08, 05]
a. Peripheral smear In Chronic Myeloid Leukemia [11]
b. Philadelphia chromosome [04, 01]
Ans.
Chronic myeloid leukaemia (CML) is a myeloproliferative stem cell disorder
characterized by the presence of a BCR-ABL fusion gene.

ETIOPATHOGENESIS:
Origin: Pluripotent hematopoietic stem cell
Genetic abnormalities: MC - Reciprocal t (9;22) translocation,
designated as Philadelphia
chromosome
Philadelphia Chromosome - it
contains BCR ABL fusion gene
with tyrosine kinase activity
Pathogenesis: Genetic
aberrations drive the
proliferation of granulocytic &
megakaryocytic progenitors,
and also causes abnormal
release of immature granulocytic forms into the
blood

CLINICAL FEATURES:
Age of onset: 50-60 yrs
The disease has 3 phases: 👉🏻
A stable phase which may last for 3yrs
Increasing anemia &
thrombocytopenia are seen after a period of 3yrs in 50% of patients.
Transformation to an acute leukemia like picture is seen having many blast forms.
In 70% of cases, blasts are of myeloid origin
(myeloid blast crisis) & in the reminder the blasts
are of pre-B cell origin (lymphoid blast crisis).
Pallor; Lymphadenopathy; Hepatosplenomegaly
Sternal tenderness due to bone marrow hyperplasia.
Acute gouty arthritis may be present due to
overproduction of uric acid.
Priapism, respiratory distress, visual blurring, and
altered mental status.

LAB FINDINGS: Normocytic, normochromic anaemia


Peripheral ▪ Anemia, leucocytosis (>105/mm3) with shift to left having few myeloblasts & thrombocytosis.
smear: ▪ Eosinophilia & basophilia are seen
1) Hypercellular marrow with elevated myeloid & megakaryocytic series of cells.
Bone Erythroid series is either unaffected or may show mild suppression.
marrow 2) Sea-blue histiocytes (scattered macrophages with abundant wrinkled green-blue
cytoplasm) are seen with increased deposition of reticulin
Karyotyping: Detects the BCR-ABL fusion gene
TREATMENT:
- 1st line drugs in CML - to inhibit
the tyrosine kinase activity of the BCR/ABL oncogene.
 In chronic phase of CML, the dose of imatinib is 400 mg orally daily. Blood
counts normalize and splenomegaly regresses within 3 months.
 Dasatinib is another agent which is effective in patients not responding to
imatinib.
- It is a protein translation inhibitor - used in
cases resistance and/or intolerance to 2 or more tyrosine kinase inhibitors.
can be used to decrease very high platelet count not responding to imatinib alone
– Suppress the bone marrow and reduce the leukocyte count.
- inhibits the late progenitors.
- If the patients do not
respond to imatinib, this is the 2nd choice of therapy
- Used to control the number of WBCs in emergency situations - Ex: priapism,
pregnant patient with CML etc.
----------------------------------------------------------------------------------------------------------------------------------------
2. Megaloblastic Anaemia – etiopath, C/F, complications, inv. Dx & Mx [22, 19, 18, 14, 13, 08, 03]
Ans.
It is a Nutritional anemia with Vitamin B12 or folic acid deficiency
Pathogenesis: Causes of Vitamin B12 deficiency Causes of folic acid deficiency
Deficiency of vitamin B12 & Poor intake - ▪ Poor intake of vegetables
Folic acid → Impaired DNA Ex: vegetarian diet ▪ Malabsorption due to
synthesis → nuclear Malabsorption due to Coeliac disease or distal ileal
maturation lags behind the pernicious anemia or distal resection
ileal resection ▪ Drugs - MTX; OCPs
cytoplasmic maturation →
demand during pregnancy ▪ demand during pregnancy
megaloblasts (abnormally large
Chronic alcoholism ▪ Chronic alcoholism
nucleated erythroid precursors) &

macrocytes (RBCs are larger than normal) are formed → Undergo intramedullary destruction → Ineffective
erythropoiesis and hemolysis →
Vit. B12 deficiency also causes abnormal myelin degradation leading to neurologic complications
Investigations:
Bone Marrow in Megaloblastic Peripheral Blood in Megaloblastic anemia
anemia Hb & PCV Reduced.
Hypercellular. Red cell ↑ MCV;
Megaloblasts & indices: ↑ MCH & normal or  MCHC
Cellulari Peripheral smear - PANCYTOPENIA
Giant
ty: Macrocytic & oval (egg-shaped)
metamyelocytes can
be seen Lack central pallor
Due to marked RBCs Anisopoikilocytosis
M:E erythroid Evidence of dyserythropoiesis: Basophilic
ratio hyperplasia, M: E stippling, Cabot ring and Howell Jolly bodies
ratio is reversed
WBCs &

BM Iron increased Platelets

Tests for both B12 & Folate Deficiency:


in serum {homocysteine,
bilirubin, iron & ferritin}.
Deoxyuridine suppression test:
POSITIVE {detect deficiency of 5, 10-methylene THF}
Specific Tests for Specific Tests for
B12 deficiency Folate deficiency
▪  Serum B12
▪ MMA in urine   Serum Folate
▪ Schilling test  FIGLU in urine
for B12 absorption
Treatment of Megaloblastic Anemia
Improve dietary habits.
Sources of Folic acid: animal products (liver, kidney) & green leafy vegetables (spinach,
cabbage, yeast, nuts and fruits)
Sources of Vitamin B12: animal products and dairy products.
Oral B12 (2 mg/day) & Folate (5-15 mg/day) supplements {parenteral, if malabsorption is suspected}
Patients on folic acid antagonists such as methotrexate should be given folinic acid daily orally
Packed red cell transfusion - For severe symptomatic anemia (Hb <7 g/dl).
----------------------------------------------------------------------------------------------------------------------------------------
3. Multiple Myeloma – C/F, Lab inv. & Mx
[21, 15, 12, 11]
a. Chemotherapy of multiple myeloma [10]
b. Myeloma protein [04]
Ans.
It is characterized by neoplastic proliferation
of a single clone of plasma cells in the bone
marrow.
CLINICAL FEATURES: It Presents as multiple
tumor masses throughout the skeletal system.
 Resorption of bone ➔ pathologic fractures
& Hypercalcaemia, chronic bone pain and
tenderness & Anemia
 Renal lesions: Light-chain cast of BJ protein
damages renal tubules ➔ Amyloidosis of the AL
type ➔ Nephrotic syndrome
 Hypercalcemia leads to nephrocalcinosis
 Humoral immune deficiency - Predisposes to
recurrent infections
 Features of amyloidosis such as enlarged
tongue, neuropathy, congestive heart failure,
or hepatomegaly may be present
INVESTIGATIONS:
Diagnosis of myeloma requires two of the
following criteria to be fulfilled:
• increased malignant plasma cells in the bone
marrow
• Serum and/or urinary M-protein
• Skeletal lytic lesions
Imaging:
 X-rays of bones show multiple punched out (lytic) lesions commonly seen in the axial skeleton
 Positron emission tomography (PET) scans are useful for staging of myeloma.
Laboratory Findings
 Hb: 
 Peripheral smear: Normocytic normochromic anemia, RBCs show rouleaux
formation due to  Igs
 ESR: High due to high gamma globulin (immunoglobulin) and rouleaux formation
 Bleeding time: Increased
Cellularity: Hypercellular due to > 30 % neoplastic plasma cells
: Erythropoiesis: Diminished and is Normoblastic.
Myelopoiesis & Megakaryopoiesis: Normal.
Serum β2 microglobulin - high values signify poor prognosis.
Hypercalcemia
Renal function tests: Blood urea, serum creatinine and uric acid levels
▪ Monoclonal spikes in 80–90% of cases
on
▪ monoclonal Igs in the blood
Serum and
▪ Light chains or Bence Jones (BJ) proteins in the urine may be seen in 60–80% of cases.
Urine
TREATMENT:
For Asymptomatic patients - keep under observation.
For Symptomatic patients  Start on thalidomide + dexamethasone.
Newer agents are bortezomib and lenalidomide
Localized radiotherapy can reduce bone pain and eradicate the tumor
For Hypercalcemia - Bisphosphonates (pamidronate 90 mg or zoledronic acid 4 mg intravenously
monthly) + Adequate hydration & mobilisation.
Blood transfusion for anemia.
should be considered in young patients
----------------------------------------------------------------------------------------------------------------------------------------
4. Classify hemolytic anemias. Discuss in detail thalassemias [21, 12]
a. Name four Investigations for Thalassemia [22]; Cooley’s anemia-Treatment [22]
b. Thalassemia Minor [15]
c. Beta Thalassemia [01]
Ans. Thalassemia {Cooley’s Anemia} are Inherited disorders
with mutations decreasing the synthesis of α or β globin
chains
They are mainly classified as:
α- Thalassemia syndromes: Impaired synthesis
of α-chains of globin. They can be of the following
types:
1) Four α-gene deletions Hb Bart’s or Hydrops foetalis
2) Three α-gene deletions HbH disease
3) Two or one α-gene deletion α-thalassaemia trait:
4) Silent Carrier
β-Thalassemia syndromes: Impaired synthesis
of β-chains of globin - only HbA contains β-chains
β globin gene is present on chromosome 11
:
 Absence of β-globin chain → absence of synthesis
of HbA →microcytic hypochromic RBC
 Unpaired and excess α-chains aggregate into
insoluble precipitates → bind to and damage the membrane of erythroid precursors →
erythroid precursors fail to mature → Marked erythroid hyperplasia in the bone Marrow.
 Extravascular hemolysis: RBCs with β-chain inclusions are removed by macrophages of spleen.
 γ globin chain synthesis continues even 6 months
after birth and combines with α-globin → ↑↑ HbF.
 Extramedullary hematopoiesis: in liver and spleen
→ hepatosplenomegaly.
 Expansion of marrow may cause erosion of existing
cortical bone & subsequent new bone formation. In
the bones of the face & skull, this gives rise to Hair
on end (“crew-cut”) appearance on x-ray.
:
Age: Infants develop moderate to severe anemia
{severe pallor} 6-9 months after birth.
Icterus - due to iron overload and liver dysfunction
Bone changes: Distortion of skull and facial bones  Thalassemic
facies & malocclusion of the jaw.
Thalassemic (chipmunk face) facies—prominent forehead,
cheekbones & upper jaw; Depressed nasal bridges
Hepatosplenomegaly
Cachexia in severe cases
:
Peripheral Blood Smear
RBCs – Microcytic hypochromic anemia; Many target cells;
Anisopoikilocytosis; Basophilic stippling; Nucleated red cell
precursors (normoblasts)
WBCs: Leukocytosis with mild left shift
Platelets: Normal
Serum haptoglobin - 
X-ray skull shows hair on end appearance
Hb electrophoresis - HbF 👉🏻
HPLC – used for confirmation of Diagnosis
:
Correction of anemia—packed red cell transfusions
Hydroxyurea—15–20 mg/kg /day used to HbF and
reduce need for blood transfusion in other thalassemic states
Future treatment—Gene replacement therapy, intrauterine
BMT
Management of Complications like:
Iron overload—treated with chelation therapy with
Desferrioxamine
Blood transfusion related reactions and infections {Hep B,
Hep C, HIV, malaria, CMV etc.}
Hypersplenism – splenectomy
Osteoporosis; osteopenia; Gall stones
Extramedullary hematopoiesis
Psychological complications
Prevention of disease by antenatal diagnosis and genetic counselling
----------------------------------------------------------------------------------------------------------------------------------------
5. Iron deficiency Anemia – Causes, Dx & treatment [11, 03]
a. Clinical features of Iron deficiency [21]
b. Serum Ferritin [18]
Ans.
: Depletion of iron stores results in inadequate
synthesis of Hb causing anemia
:
Fatigue, pallor; Alopecia, koilonychia; Pica; Plummer-Vinson
syndrome
IDA also impairs cognitive abilities and can affect mental, social
and emotional development
As iron is a cofactor for many enzymes in cellular metabolic pathways, IDA can depress immunity
and predispose to secondary infections.
:
:
Hemoglobin, hematocrit (PCV) & Reticulocyte count: decreased
↓ in MCV, MCH & MCHC; ↑ In RDW
RBCs: microcytic, hypochromic,
ring/pessary cells, anisocytosis &
poikilocytosis (pencil/cigar shaped cells).
WBCs & Platelets: Normal
Serum Iron Profile:
▪ Reduced: Serum iron, ferritin & % transferrin saturation
▪ Increased: TIBC, TFR (transferrin receptor) and red cell protoporphyrin.
BM Findings
Cellularity: Moderately hypercellular
M:E ratio: varies from 2:1 to 1:2 (normal 2:1 to 4:1).
Erythropoiesis: Hyperplasia and micronormoblastic maturation.
Myelopoiesis & Megakaryopoiesis: Normal
Absence of bone marrow iron: “Gold standard” test, demonstrated by ve Prussian blue reaction.

Identify & treat the cause


Dietary counselling
Iron Supplements

a) Intolerance to oral iron


It is preferred for the treatment of b) Malabsorption of iron
c) Non-compliance to oral iron
Indications iron-deficiency anemia as it is safer
d) Severe iron deficiency
and cheaper than parenteral iron
e) Along with erythropoietin in patients with
renal disease
Therapeutic dose: 100–200 mg 1) Iron-dextran {IM or IV}
elemental iron per day (children 3–5 2) Iron-sorbitol-citric acid {IM}
mg/kg/day): 3) Ferrous-sucrose {IV}
Preparations 1) Ferrous sulfate 4) Ferric-carboxymaltose.
2) Ferrous fumarate Dose of iron = (2.38 x W x D) + 1000
3) Ferrous gluconate  W = weight in kg
4) Colloidal ferric hydroxide  D = hemoglobin deficit = 15 - patient’s Hb
 The injections are painful, may cause
Nausea, vomiting, epigastric abscess and discolouration of the skin at
discomfort, dyspepsia, metallic the site of injection.
A/E taste, constipation or diarrhoea &  The systemic side effects are headache,
staining of teeth (with liquid pyrexia, nausea, vomiting, arthralgia,
preparation) lymphadenopathy, urticaria and
anaphylactic reaction.
----------------------------------------------------------------------------------------------------------------------------------------
6. Hodgkin's lymphoma – C/F, Dx & Mx [10]
a. Chemotherapy of Hodgkin's Lymphoma [04]
Ans.
Hodgkin Lymphoma (HL) arises in a single node or chain of nodes and spreads in a predictable way to
anatomically contiguous lymphoid tissue.
 Genetic alterations: In classical forms, mutations activate transcription factor NF-kB
 Risk factor EBV association in classical forms
 Unique Features of HL: HL is confined to Lymph
Ann Arbor Classification
Stages Description
nodes with little Extranodal involvement & it
I One LN
consist of Reed-Sternberg cells (R-S cells),
II >2 LN on one side of Diaphragm
pathognomonic of Hodgkin lymphoma III >2 LN on BOTH sides of Diaphragm
IV Extralymphatic spread
Clinical Staging Ann Arbor Classification – I to IV stages. All stages are further divided on the basis
of the absence (A) or presence (B) of systemic manifestations (fever, night sweats &/or weight loss).
Types of Hodgkin’s Lymphoma
Classical Types Lymphocyte
Features
Nodular Mixed Lymphocyte Lymphocyte predominant
sclerosis cellularity rich depletion type
Age Young adults All ages Older Elderly < 35
Sex M=F M>F M>F M>F M>F
EBV
associatio Uncommon 70 % 40 % 90 % ABSENT
n
Painless Painless
Painless lymphadenopa lymphadenopa
Painless Painless
Clinical lymphadenopat thy with thy with
lymphadenopat lymphadenopat
Presentati hy; systemic signs systemic signs
hy; No systemic hy; No systemic
on No systemic (night sweats, (night sweats,
signs signs
signs fever & weight fever & weight
loss) loss)
Stage I - II III - IV I - II III - IV I - II
MC type. Least Common Cervical &
type axillary lymph
Other
Often involves nodes are
Points
mediastinal HIV infection is commonly
nodes a Risk Factor involved
Spread: Lymph nodes are involved first, then spleen, liver and finally bone marrow & others
FNAC & biopsy of involved Lymph nodes or tissues
ESR & Chest X-ray may show a mediastinal mass
LFTs - to know hepatic infiltration.
CT scan of chest, abdomen and pelvis permits staging.
Positron emission tomography (PET) scanning identifies nodes involved with HL.

Radiotherapy - for stages IA & IIA disease.


Chemotherapy regimens for HL:
 ABVD = driamycin; leomycin; incristine; acarbazine
 COPP = yclophosphamide, vincristine ( ncovin), rocarbazine, And rednisolone
 These drugs are given every 3 to 4 weeks for a total of 6-8 cycles]
Combined Modality Treatment - 1st chemotherapy  then radiotherapy
Autologous Stem Cell Transplantation - for patients who relapse after initial chemotherapy.
----------------------------------------------------------------------------------------------------------------------------------------
7. Bone marrow transplantation – indications, technique,
immediate & late Complications [07, 2000]
a. Indications for Bone Marrow transplantation [17]
b. Indications for Bone Marrow Aspiration study [16]
Ans. Bone marrow transplantation is now called hematopoietic
stem cell transplantation.
Method of Bone Marrow Transplantation:
Umbilical cord blood can be taken {if it’s available}
Marrow is collected from
the donor's iliac crests with Diagnostic indications - {same as those for Bone Marrow Transplantation}
the donor under general or Therapeutic indications - Bone Marrow Transplantation
spinal anesthesia.
Hematopoietic stem cells can also be obtained from peripheral blood after giving the donor hematopoietic growth factors for 4-5 days.
Typically, 10 to 15 ml/kg of marrow is aspirated, placed in heparinized media, and filtered to
remove fat and bony spicules
Prepare the recipient: eradicate the immune system to prevent rejection of the transplanted
marrow by using regimens of busulfan, cyclophosphamide, melphalan, etoposide etc. and total-
body irradiation in various combinations.
Cells produced by transplanted stem cells begin to appear after a week in the peripheral blood.
Complications of Bone Marrow Transplantation:
Due to preparatory regimens: Infections due to immunosuppression (HSV, CMV, VZV);
cardiotoxicity; hemorrhagic cystitis; hair loss and pancytopenia.
GVHD (Graft-Versus-Host Disease).
Veno-occlusive disease of the liver.
Graft failure.
----------------------------------------------------------------------------------------------------------------------------------------
8. Outline the blood coagulation cascade. Write the etiology, C/F & treatment of Haemophilias [07]
a. Hemophilia [17]
Ans.
Hemophilia is the MC hereditary cause
of serious bleeding with an abnormality
of the intrinsic coagulation pathway

Cause: Sporadic or germ line


mutations causing factor VIII deficiency
Inheritance of the Disease:
▪ X-linked recessive disease.
▪ Genes for factor VIII are located on
the long arm of the X-chromosome.
▪ Males with a defective factor VIII
gene (hemophiliac gene) on their
single X chromosome (XH) suffer
from hemophilia.
▪ Heterozygous females are carriers
and do not express the full clinical
disease because of the paired
normal X-chromosome.
Genetic alterations: deletions, inversions, point
mutations and insertions
Clinical Presentation:
1) Easy bruising, massive hemorrhage after trauma
or operative procedures.
2) Spontaneous bleeding into joints (hemarthrosis).
Differential Diagnosis: Hemophilia B; Von Willebrand disease
Lab Investigations to confirm Dx
Bleeding time: Normal
Clotting time: ↑
Platelet count: Normal
Prothrombin time: Normal
Activated partial thromboplastin time (APTT): ↑ (normal 30–40 seconds)
Factor VIII assay: ↓; Essential for the Dx & to assess the severity of disease {Normal range for factor VIII:
45–158 IU/dL}

Fibrinogen assay: Normal


FDP: Negative
Detection of carriers: By DNA markers
Complications due to Hemophilia: Anemia, intramuscular hematomas & Joint deformities
Management
 Appropriate factor
replacement; Gene therapy
 Physiotherapy to prevent
chronic joint disease
 Counselling for injury
prevention
 Desmopressin, synthetic
analogue of vasopressin – induce
factor VIII levels and is tried
in mild cases
 Acute bleeds should be
treated early (within 2 h if
possible) – via
Cryoprecipitate & fresh
frozen plasma {Cryoprecipitate
doesn’t contain factor IX so it cannot be
used in hemophilia B}
 Avoid IM injection & arterial puncture
Complications following the therapy
Viral hepatitis: Hepatitis B, C & D in patients who received multiple transfusions of
FFP/cryoprecipitate.
AIDS: In individuals who received fresh frozen plasma (FFP) or cryoprecipitate, when screening
tests for HIV were not available
Development of antibodies against factor VIII, following infusions of recombinant factor VIII makes
further management difficult.
----------------------------------------------------------------------------------------------------------------------------------------

1) Myelodysplastic Syndromes [21]


Ans.
Myelodysplastic syndromes (MDSs) constitute a group of clonal haematopoietic disorders with the
common features of ineffective blood cell production and a tendency to progress to AML.
: idiopathic
Risk Factors: radiation exposure; chemotherapy; chromosomal abnormalities such as deletions of
long arms of chromosomes 5 and 7 may be seen.
MDS is characterized by dysplasia in one or more lineages but blast count will be <20% in BM smear
MDS can lead to pancytopenia, myelofibrosis or may progress to AML.
:
Seen in elderly.
Patients usually present with fatigue (due to anemia),
infection (due to leukopenia), or bleeding (due to
thrombocytopenia) related to bone marrow failure.
Examination reveals pallor, bleeding, and signs of
infection; Hepatosplenomegaly (due to extramedullary
hematopoiesis).
:
Anemia is treated by RBC transfusions.
Erythropoietin injection given weekly subcutaneously
reduces the RBC transfusion requirement.
Myeloid growth factors such as G-CSF (granulocyte
colony stimulating factor) help patients with severe neutropenia.
Azacytidine (5-azacytidine) - rate of transformation to leukemia and the need for transfusions.
Stem cell transplantation is the only curative therapy for myelodysplasia.
----------------------------------------------------------------------------------------------------------------------------------------
2) Mention 4 important clinical features of Whipple's disease [19, 17, 14]
Ans.
Whipple disease is a rare bacterial infection that most
often affects the joints and digestive system
Causative organism: Gram-positive actinomycetes-
Tropheryma whipplei
Pathogenesis: the bacteria reside within the
macrophages in the bowel mucosa  densely packed
macrophages in the lamina propria  obstruct
lymphatic drainage and cause fat malabsorption
Clinical Features: The disease is a multisystem one
and almost any organ can be affected
Treatment:
IV ceftriaxone (2 g daily for 2 weeks), followed by oral
co-trimoxazole for at least 1 year.
Alternative: doxycycline and hydroxychloroquine;
given for 1 year
Lifelong doxycycline to relapse.
----------------------------------------------------------------------------------------------------------------------------------------
3) Sickle-cell anaemia [19, 15]
a. Sickle cell anaemia-Lab diagnosis [05]
Ans.
It is a hereditary hemoglobinopathy
Mode of inheritance: Autosomal recessive disorder
Genotype: Homozygous form
Genetic alterations: Substitution of valine for glutamic acid at the 6th position of the β globin chain
forms sickle hemoglobin (HbS). It is an Example of Missense point mutation
Predisposing factors: intracellular pH; dehydration; Inflamed microvascular beds with slow
transit times.
Pathogenesis
Deoxygenation favors HbS formation → HbS molecules aggregate (tactoids) within RBCs →
tactoids grow in length beyond the diameter of RBCs → RBC become elongated and assumes a
shape like sickle (or crescent moon or holly-leaf or boat) → Sickle cells express higher levels of
adhesion molecules and thus become abnormally sticky to the endothelium → vascular
occlusions.
When the oxygen tension returns to normal, the sickled red cell returns to normal shape.
Recurrent sickling damage the red cell membrane and these RBCs become irreversibly sickled cells
Sickle cells undergo hemolysis causing anemia
HbF inhibits polymerisation of HbS and reduces sickling  mild clinical course with few crises
Lysed sickle cells liberate free hemoglobin, which binds and inactivates nitric oxide (NO). This
narrows the vessels and produces microvascular stasis and sickling.
Clinical Presentations:
Impairment of growth & development
Chronic hemolytic anemia with gall stones.
Vaso-occlusive crises (pain crises): Major cause of morbidity & mortality.
▪ Cause severe pain in the affected regions (bones, lungs, liver, brain or penis).
▪ E.g., Acute chest syndrome; Priapism; Strokes; Hand-foot syndrome etc.
Aplastic crises with parvovirus B19
infection, worsens anemia.
Sequestration crises with intact
spleen may cause splenomegaly
Splenomegaly seen in early childhood
but adults manifest autosplenectomy
Increased risk of infections with
Strep. Pneumoniae & H. influenza.
Investigations:
Hemogram: Hb, PCV – Low;
Reticulocyte count – elevated
Peripheral smear: RBC – Sickle cells,
target cells, and Howell-Jolly bodies
are seen
Hb electrophoresis detects HbS
Hyperbilirubinemia
Prenatal diagnosis by analysis of fetal
DNA
Bone Marrow Hyperplastic marrow with erythroid hyperplasia
Sickle cell slide test: Test used for detection of hemoglobin S.
 Principle: When red cells containing HbS are deprived of oxygen, they become sickle-shaped.
 Reagent: 2% sodium metabisulphite is used a reducing agent.
 Positive test: Red cells become sickle shaped or holly-leaf shaped
 False –ve result: Usage of outdated reagent; Low concentration of HbS
 False +ve result: Excessive usage of reagent; Drying of wet preparation
Treatment:
Prophylaxis with daily folic acid
In Vaso-occlusive crises - aggressive rehydration, oxygen therapy, adequate analgesia & antibiotics
Transfusion in aplastic crisis
Allogeneic stem cell transplantation is curative.
----------------------------------------------------------------------------------------------------------------------------------------
4) Causes of Polycythemia [18, 16]
a. Polycythemia Rubra Vera & it’s treatment [15, 10]
b. Polycythemia [12]
Ans.
Polycythemia is defined as an increase in circulating RBCs
above normal.
 Absolute Polycythemia – number of cells
 Relative Polycythemia – plasma volume is decreased
without actual increase in number of cells.
Clinical features:
Males are affected more often.
Initial symptoms are vague, such as headache, dizziness,
tinnitus, weakness,
lassitude, and
fatigue.
Thrombotic
manifestations
range from digital
ischemia to Budd-
Chiari syndrome
with hepatic vein
thrombosis.
Neurologic
symptoms such as
vertigo and visual
disturbances.
Hypertension is
present.
Fundoscopy reveals
congestion of the
discs, engorged veins
& hemorrhages
Hyperuricemia secondary gout.
Severe pruritus, especially after a hot bath, is a common symptom.
Erythromelalgia (burning pain in the feet or hands accompanied by erythema, pallor, or cyanosis)
is due to microvascular thrombotic occlusions.
Congestion and plethoric appearance of the face with congested conjunctivae.
Splenomegaly is present in polycythemia vera but absent in secondary polycythemia.
Investigations:
Examination of blood reveals Hb, hematocrit and decreased ESR.
Leukocyte alkaline phosphatase is high.
Bone marrow shows hypercellularity
PCR – to detect JAK2 mutation is present in virtually all patients with PV.
Other investigations to detect the underlying cause.
Treatment:
Venesection (Phlebotomy) –blood is withdrawn to bring down the hematocrit to normal.
Antiplatelet Agents – Low dose aspirin or clopidogrel – to prevent thrombotic events.
 Anagrelide also inhibits platelet aggregation
Radioactive Phosphorus 32P – to irradiate the marrow – contraindicated during pregnancy.
Cytotoxic Drugs – Ex: busulfan, cyclophosphamide, and chlorambucil – used in severe cases.
Hydroxyurea is a safe cytotoxic drug without tumour-producing potential.
Ruxolitinib is an inhibitor of JAK2 pathway and is approved for the treatment of patients who do
not respond to hydroxyurea.
Allopurinol – for hyperuricemia.
----------------------------------------------------------------------------------------------------------------------------------------
5) Complications of Blood Transfusion [17, 04]
Ans.

----------------------------------------------------------------------------------------------------------------------------------------
6) ITP – Idiopathic thrombocytopenic purpura & it’s Treatment [16, 10]
a. Treatment of ITP (Idiopathic thrombocytopenic purpura) [16]

Ans. Immune thrombocytopenic purpura (aka Idiopathic Thrombocytopenic Purpura)


(ITP) – occurs due to Autoantibody mediated destruction of platelets causing thrombocytopenia.
Sex: F>M
Etiology: Primary or secondary.
 Primary or idiopathic: No known risk factors.
 Secondary: Seen with SLE, HIV or CLL.
Pathogenesis:
 Production of autoantibodies (IgG) against platelet membrane glycoproteins IIb-IIIa or Ib-IX.
 Autoantibody coated platelets are phagocytosed and destroyed in spleen.
Clinical Picture: Bleeding manifestations such as petechiae,
ecchymoses; Easy bruising, nosebleeds, and bleeding from
gums; Melena, hematuria or excessive menstrual flow.
Diagnosis:
Peripheral smear: Low platelet count with large platelets
(megathrombocytes).
Bone marrow: raised megakaryocytes.
PT & PTT: Normal
Tests for platelet autoantibodies.
Complications: Subarachnoid hemorrhage & intracerebral
hemorrhage.
Management
Children with active bleeding should be given IV immunoglobulin 1 g/kg/day for 1–2 days
Prednisolone 1–4 mg/kg/day for 2–4 weeks then tapered
Dexamethasone at 20 mg/m2 over 4 days every 3 weeks for 4–6 courses.
Routine Platelet transfusion should be avoided {done only in serious hemorrhages}
Refractory cases—immunosuppressive drugs like vincristine, cyclosporine, azathioprine, rituximab
----------------------------------------------------------------------------------------------------------------------------------------
7) Glucose - 6 - Phosphate dehydrogenase deficiency [13]
Ans.
 G6PD deficiency is inherited as X-linked recessive
trait.
 The deficiency therefore affects males and rare
homozygous females, but it is carried by females.
Pathogenesis:
G6PD is the first enzyme in the hexose
monophosphate shunt pathway which generates
NADPH.
NADPH is required to keep the glutathione in
reduced state in RBCs & Neutrophils.
Depletion of cellular glutathione results in damage
to RBCs by oxidizing agents and various drugs
leading to hemolysis.
Treatment:
Asymptomatic individuals require no treatment.
Mild hemolytic episodes are treated by
withdrawing the offending drug, or treatment of the
concurrent infection.
Severe hemolytic episodes require RBC transfusions to correct anemia & measures to prevent
renal failure due to hemoglobinuria.
----------------------------------------------------------------------------------------------------------------------------------------
8) Acute leukaemia & it’s Treatment [12]
Ans.
It is a tumor of hematopoietic progenitors defined by presence of > 20 % immature blast cells in
the Bone Marrow
 Lymphoblasts in ALL & Myeloblasts in AML
ALL is the MCC of cancer deaths in children
AML {Acute Myeloblastic Leukaemia} ALL {Acute Lymphoblastic Leukemia}
Origin Hematopoietic progenitor cells Precursor B or T cells
Structural abnormality:
1) Translocations such as t (8;21) & t (15;17)
T- ALLs – Gain of function mutations in
are seen with AML arising de novo in
NOTCH 1 gene.
patients with no risk factors
Genetic B-ALLs – Loss of function mutations in
2) Deletions or monosomies involving
Alterations PAX5, E2A & EBF genes or a balanced
chromosomes 5 & 7 are seen with AML
translocation t (12; 21)
that follow MDS or exposure to DNA-
Numerical abnormality: Hyperploidy is
damaging agents
common
Genetic abnormalities ➔ Defective
Genetic aberrations block the Maturation of Lymphoblast ➔ Can form
Pathogenesis maturation of myeloid progenitors with Pre-B or Pre-T ALLs.
their proliferation & survival It can be Extramedullary (Lymphoblastic
LYMPHOMA) involvement is seen.
1) Abrupt onset of Fatigue & weakness; Abrupt onset of Bone pain & marrow
Recurrent infections with fever; Bleeding failure  pancytopenia &
manifestations. complications associated with it
Clinical 2) DIC may be seen in M3.
Lymphadenopathy (usually cervical)
Features 3) Gingival hyperplasia may be seen in M5.
4) Rarely, localized soft tissue mass Mediastinal thymic mass is common
(myeloblastoma, granulocytic sarcoma or in T-ALL
chloroma) may be seen It metastasizes to CNS & Testicles.
Hypercellular marrow showing > 20 %
Bone Marrow Hypercellular marrow showing > 20 % myeloblasts lymphoblasts with interspersed macrophages gives
starry sky appearance
Peripheral Blood Hb + Markedly raised Total WBC Count
– Normocytic normochromic anemia – Normocytic normochromic anemia
- -
 Differential count: > 20% myeloblasts Differential count: > 20% lymphoblasts
 Auer rods in the cytosol of myeloblasts - 
(M2 and M3 subtype).
-
Peripheral

Smear

Lymphoblasts are myeloperoxidase


negative & PAS positive.
Myeloblasts are myeloperoxidase Immunophenotyping
Cytochemistry positive & PAS negative {except Monoblasts  Immature B-cells +ve for pan B cell
which are MPO & PAS negative} marker CD19 and CD10
 Precursor T ALL cells are +ve for CD2,
CD5 and CD8.
1) Good for AMLs with the t (15;17).
2) for AMLs following MDS (Myelodysplastic if below 2-years, male child, t(9;22);
Prognosis Syndrome) or genotoxic therapy are seen with meningeal involvement etc.
older adults.
Treatment of Acute leukaemia: ICMR
The aim of treatment is to destroy the leukemic clone of cells
without destroying the residual normal stem cell compartment
from which repopulation of the haematopoietic tissues will
occur. VIP - MTX for ALL in induction & maintainance
There are 3 phases:
Remission induction: In this phase, a fraction of the tumour
is destroyed by combination chemotherapy.
Remission consolidation – If remission has been achieved,
residual disease is attacked by other therapy during the
consolidation phase.
Remission maintenance: If the patient is still in remission
after the consolidation phase for ALL, a period of
maintenance therapy is given, with the individual as an
outpatient. This may extend for up to 3 years if relapse does not occur.
Treatment of M3: ATRA
----------------------------------------------------------------------------------------------------------------------------------------
9) Laboratory features of acute intra vascular haemolysis [09]
a. Laboratory test for red blood cell haemolysis [07]
Ans.

----------------------------------------------------------------------------------------------------------------------------------------
10) Erythrocyte sedimentation rate [09]
Ans.
ESR measures the rate of settling of erythrocytes in anticoagulated whole blood
: When anticoagulated blood is placed in a vertical tube and is allowed to stand, RBCs settle
towards the bottom of the tube. The speed of sedimentation of red cells in plasma over a period of 1
hour is measured by the length of the sedimented RBC column and is expressed in mm. RBCs have net
negative charge on their surface and tend to repel each other. The repulsive forces are partially or
totally counteracted if there is an increase in the positively charged plasma proteins.
Old age, pregnancy, Anemia, Macrocytosis, raised fibrinogen
Polycythemia, Microcytosis, low fibrinogen.
Westergren and Wintrobe methods.
Westergren method:
▪ Westergren tube open at both ends, capacity 1mL, calibrated in mm from 0–200.
▪ Anticoagulant used: 3.8% trisodium citrate
▪ Blood to anticoagulant ratio should be 4:1.
Infections (TB; Bacterial endocarditis;
Osteomyelitis) Inflammatory diseases (Rheumatoid arthritis;
Temporal arteritis); Acute myocardial infarction; Malignancy
: Congestive cardiac failure; Sickle cell anemia; Hereditary spherocytosis

 Suspicion of infectious, inflammatory or neoplastic conditions


 To monitor disease activity: For example, rheumatic fever, tuberculosis
 As a diagnostic criterion: For example, temporal arteritis.
----------------------------------------------------------------------------------------------------------------------------------------
11) Disseminated Intra-vascular Coagulation [08, 04, 03]
Ans.
Disseminated intravascular coagulation (DIC) is a state characterized by widespread intravascular
coagulation associated with increased consumption of platelet and clotting factors.
Etiology: DIC usually results from exposure of tissue factor to blood, initiating the coagulation cascade.
DIC most often occurs in the following circumstances:
Obstetric complications (amniotic fluid embolism, dead retained fetus) - MC
Cancers (adenocarcinomas of the lung & pancreas; Acute promyelocytic leukemia)
Sepsis (meningococcemia)
Tissue injury (major trauma, severe burns, snake bite)
Clinical Features
DIC leads to both bleeding and thrombosis.
Bleeding may occur at any site, but spontaneous bleeding and oozing at venipuncture sites or
wounds are important clues to the diagnosis.
Thrombosis is most commonly manifested by digital ischemia and gangrene.
In severe cases, dysfunction of liver, kidney, lungs & CNS also occur.
DIC also causes microangiopathic
hemolytic anemia.
Laboratory Findings
• 1) Platelet count: Low
• 2) Fibrinogen levels: Reduced
• 3) PT & PTT: Prolonged
• 4) Fibrin degradation products: Elevated {D-dimer}
• 5) Peripheral smear shows fragmented RBCs due to microangiopathic hemolytic anemia.
• 6) Antithrombin III levels may be markedly reduced.
Treatment – Identify & treat the cause appropriately.
Replacement therapy –
 Transfuse Platelets;
 Replace Fibrinogen with cryoprecipitate.
 Fresh Frozen Plasma – to replace coagulation factor deficiency
If antithrombin III levels are very low, it can be raised by giving fresh-frozen plasma or antithrombin
III concentrates.
----------------------------------------------------------------------------------------------------------------------------------------
12) Hypersplenism [07, 04, 2000]
a. 4 causes of Splenomegaly [21]
b. Clinical splenomegaly [95]
Ans.
Spleen is the major organ of the monocyte/macrophage system.
: Any disease which produces splenic enlargement
can produce hypersplenism.

As the blood passes through the white and red pulp, old and defective
blood cells are removed by the spleen.
The macrophages in the spleen hold, retard, modify ("pitting"), or remove
("culling") old and senescent RBCs.
All these normal activities of the spleen can be markedly accentuated in a
large spleen leading to pancytopenia (anemia, neutropenia and
thrombocytopenia). This is called hypersplenism.

 Identify & treat the cause


 Consider splenectomy in severe cases of anemia

----------------------------------------------------------------------------------------------------------------------------------------
13) Clubbing [04, 99]
a. Causes of Clubbing [21]
b. Respiratory causes of clubbing [21]
Ans.
Clubbing is enlargement of soft
tissues
leading to both transverse and
longitudinal curving of nails.
Longitudinal curving of nails leads to loss of angle between the nail
and nail bed.
Normally this angle is less than 180 degrees. In clubbing, it is more than 180 degrees.
Pulmonary tuberculosis Mesothelioma
Lung abscess Interstitial lung disease
Resp. System
Bronchiectasis Empyema thoracis
Bronchogenic carcinoma Cystic fibrosis
• Infective endocarditis •Cyanotic congenital heart diseases
CVS
• Atrial myxoma
 Ulcerative colitis  Hepatocellular carcinoma
GIT
 Primary biliary cirrhosis  Crohn's disease
Endocrine Myxedema & Acromegaly
▪ U/L clubbing  Pancoast tumor, subclavian artery aneurysm
Miscellaneous
▪ Unidigital clubbing-trauma
----------------------------------------------------------------------------------------------------------------------------------------
14) Sideroblastic anaemia [04]
Ans.
Sideroblastic anemias occur due to deranged synthesis of heme within red cell precursors
Etiology:
1) Hereditary sideroblastic anemia – X-linked recessive ALA synthase deficiency
2) Acquired sideroblastic anemia: idiopathic or secondary
Secondary is due to:
Drugs: INH, cycloserine, chloramphenicol and alkylating agents (e.g., cyclophosphamide).
Alcohol and lead also cause sideroblastic anaemia
Haematological disorders: These include MDS, polycythaemia vera, acute leukaemia,
myeloma, lymphoma and haemolytic anaemia
Pathogenesis: Each of the etiological agent inhibit mitochondrial heme synthesis via different
mechanisms  Fe overload in Nucleated RBCs  Fe accumulates around nucleus to form a ring 
Ring sideroblasts in the bone marrow aspirate
Clinical Presentation:
Features of anemia
Ring sideroblasts in BM smear
Dimorphic peripheral blood picture: Microcytic
hypochromic red cells in hereditary form and
macrocytic in the acquired forms of the disease mixed
with normochromic cells.
Iron-containing inclusions (Pappenheimer bodies) in the RBCs.
Increased serum iron concentration and markedly increased storage iron.
Treatment:
Anemia responds to large doses of pyridoxine (200 mg daily for 2–3 months).
Blood transfusions can be given for severe anemia
Iron overload can be treated by periodic phlebotomies and Desferrioxamine.
Recombinant human erythropoietin and GM-CSF (granulocyte-monocyte colony-stimulating factor)
are helpful in selected cases.
Bone marrow transplantation can be done in severe transfusion-dependent patients.
----------------------------------------------------------------------------------------------------------------------------------------
15) Rh-incompatibility [02]
Ans.

----------------------------------------------------------------------------------------------------------------------------------------
16) Cyanosis [02]
Ans.
Cyanosis refers to a bluish discoloration of the skin and mucous
membranes.
Cyanosis is seen when concentration in
capillary blood is more than 5 g/dl.
Cyanosis can be divided into two types, central and peripheral.
 In congestive heart failure both peripheral and central cyanosis
may coexist.
 Cyanosis affecting only lower
limbs but not upper limbs is
called differential cyanosis. It is
seen in patients with patent
ductus arteriosus with reversal
of shunt.
 Cyanosis of only upper limbs can
occur in patent ductus arteriosus
with reversal of shunt with
transposition of great vessels.
----------------------------------------------------------------------------------------------------------------------------------------
17) Von Willebrand's disease [02]
Ans.
It is the MC inherited bleeding disorder caused by mutations in vWF resulting in either qualitative or
quantitative deficiency of vWF.
:
 Inheritance: Autosomal dominant (MC) – type 1 & 2; {type 3 is AR}
 vWF is important for platelet aggregation. vWF also protects factor VIII in the circulation
 Types: Type 1 & type 3: Quantitative defects in vWF.
 Type 1: MC type; Autosomal dominant; Mild to moderate vWF deficiency; Mild bleeding (e.g.,
epistaxis)
 Type 3: Autosomal recessive; Vey low levels of vWF; Severe disease (similar to hemophilia A)
 Type 2: Qualitative defects in vWF; Autosomal dominant; Mild to moderate bleeding from
wounds or menorrhagia
: Spontaneous mucosal bleeding (epistaxis, gingival bleeding, menorrhagia).
 Characteristically, bleeding decreases during pregnancy or estrogen use
 Bleeding tendency is exacerbated by aspirin
: Bleeding time is prolonged in the presence of normal platelet count
Platelet count & PT: Normal
BT, CT & APTT: ↑
Tourniquet test (Hess test): + ve due to defect in platelet adhesion
vWF assay: ↓ in type 1 & 3
Factor VIII levels: Low in type 1 & 3
Platelet function test: Defective ristocetin-induced platelet aggregation test is diagnostic of vWF
:
Since the bleeding is mild, no treatment is necessary except before surgery or dental procedures.
Desmopressin acetate (DDAVP) can increase the vWF levels by releasing stored vWF from
endothelial cells. It can be given before surgery or dental procedures.
The antifibrinolytic agent epsilon aminocaproic acid (EACA) is useful as adjunctive therapy
----------------------------------------------------------------------------------------------------------------------------------------
18) Erythropoietin [2000]
Ans.
It is a glycoprotein growth factor which stimulates
erythropoiesis, and RBC maturation.
Source of EPO: kidney and a small amount (<10 %) by the
liver.
Stimulus for release of EPO: Hypoxia
In patients with chronic renal failure, anemia is common due
to EPO production.
Ectopic sources of EPO include cerebellar Hemangioma,
uterine leiomyoma, pheochromocytoma, & hepatoma.
----------------------------------------------------------------------------------------------------------------------------------------
19) Haemochromatosis [2000]
Ans.
Hemochromatosis refers to abnormal accumulation of iron in various organs such as liver, pancreas &
heart leading to organ damage
:
Hereditary causes
Hereditary hemochromatosis - caused by mutations in the HFE gene - inherited as an
autosomal recessive disease
Transferrin and ceruloplasmin deficiency  impaired transportation of iron from the organs
Secondary causes
Dietary iron overload (prolonged oral iron therapy) & African iron overload (Bantu siderosis)
Repeated blood transfusion & Ineffective erythropoiesis (thalassemia, sideroblastic anemia)
Liver disease
Porphyria cutanea tarda
• Excess iron  produces free radical  cell injury and fibrosis
:
Serum Iron & ferritin. TIBC is low.
CT – reveal excess hepatic iron.
Liver biopsy – can confirm the diagnosis.
Genetic testing – for mutations of the HFE gene

Venesection (phlebotomy) – for hereditary


cases – Weekly venesection of 500 ml blood (250
mg iron) is done until the serum iron is normal;
Chelation therapy is mainly used for secondary
hemochromatosis – Deferoxamine, Deferasirox
and deferiprone etc.
Diabetes and cirrhosis are treated as per
standard guidelines.
First-degree family members should be screened for hemochromatosis by genetic testing.
----------------------------------------------------------------------------------------------------------------------------------------
20) Hairy cell leukemia [2000]
Ans. This is a rare chronic B-cell lymphoproliferative disorder {mature B-Cell leukemia}.
a/w Mutation in the BRAF gene.
: occurs in older males
 Splenomegaly & features of pancytopenia {esp. monocytes}

Molecular Marker: B-raph; CD25 and CD103.


Morphology: typical “hairy” neoplastic lymphocytes are seen
in spleen, BM & peripheral blood
: Opportunistic infections & Vasculitis
: Purine analogue {Cladribine} + splenectomy
----------------------------------------------------------------------------------------------------------------------------------------

1. Drugs causing agranulocytosis [22]


Ans.

----------------------------------------------------------------------------------------------------------------------------------------
2. Four causes of purpura [21]
Ans.

 Purpura means purple.


 They are hemorrhagic spots in the skin, mucous membranes
or serosal surfaces, slightly larger (>3 mm) than petechiae.

----------------------------------------------------------------------------------------------------------------------------------------
3. Warfarin [19]
Ans. Warfarin is an Oral Anticoagulant
: Warfarin is a coumarin derivative and has a structure similar to that of vitamin K. Hence,
warfarin competitively inhibits epoxide reductase enzyme → inhibits the synthesis of vitamin K-
dependent factors —II, VII, IX and X as well as anti-clotting proteins, protein C and protein S, →
anticoagulant effect
: treat blood clots (such as in DVT or pulmonary embolus) and/or to prevent new clots from
forming in susceptible patients (like those with atrial fibrillation, heart valve replacement, recent
heart attack, and certain surgeries (such as hip/knee replacement).

is the most common side effect of warfarin. Fresh frozen plasma should be given in severe bleeding for
immediate replacement of clotting factors
effect: It crosses the placenta and can cause fetal warfarin syndrome; also known as
Contradi syndrome (growth retardation, stippled epiphyses, hypoplasia of nose & hand bones etc.)

Other rare side effects: These include diarrhoea, alopecia, urticaria, dermatitis, abdominal
cramps and anorexia.
----------------------------------------------------------------------------------------------------------------------------------------
4. Coomb’s test [19]
Ans.

Indications:
1) Hemolytic disease of new-born: Mother’s serum
is tested to detect anti-Rh antibody.
2) Cross-matching for blood transfusion: To detect
incompatibility of recipient’s serum with donor’s
cells.

----------------------------------------------------------------------------------------------------------------------------------------
5. Waldenstrom's Macroglobulinemia [14] {compare with multiple myeloma}
Ans.
▪ Aka Lymphoplasmacytic lymphoma
▪ it consist of neoplastic cells which secrete IgM
Clinical Presentation: Patients classically present with features of Hyperviscosity {since IgM is
pentamer}  thrombus formation
 Visual disturbances, Neurologic deficits etc.
 Bleeding tendencies {since all platelets are used up in thrombus formation}
 Anaemia, systemic symptoms, splenomegaly or lymphadenopathy.
Management:
Plasmapheresis – to remove IgM
Chemotherapy with alkylating agents – Ex: chlorambucil; Fludarabine;
Rituximab & ibrutinib can also be used
Blood transfusion – for anemia.
----------------------------------------------------------------------------------------------------------------------------------------
6. Drug induced Purpura [12]
Ans. DRUGS THAT INDUCE PURPURA: Quinine; Quinidine; Sulphonamides; NSAIDS & Valproate
----------------------------------------------------------------------------------------------------------------------------------------
7. Thrombocytopenia. [11]
Ans.
Definition: Decrease in the platelet count below the lower limit of 150,000/cu mm (150 × 109/L).
Clinical Features:
 Cutaneous bleeding appears as
pinpoint hemorrhages (petechiae) and
ecchymoses
 Mucosal bleeding
 Intracranial bleed (subarachnoid and
intracerebral hemorrhage) rare {if platelet
count is <10,000/cu mm}

----------------------------------------------------------------------------------------------------------------------------------------
8. Microcytic hypochromic anaemia [09]
Ans.

----------------------------------------------------------------------------------------------------------------------------------------
9. Indications for blood transfusion [05]
Ans.

----------------------------------------------------------------------------------------------------------------------------------------
10. Pancytopenia [03]
Ans.
1) Write the aetiology, pathogenesis, C/F, Dx, complications of Bronchial asthma & Mx of acute
severe asthma [21, 20, 17, 15, 14]
a. Status asthmaticus [13, 2000]
b. Role of steroids in management of asthma [11]
c. Pulmonary function test in bronchial asthma [09]
Ans.

Bronchial
Hyperresponsiveness test -
- even low doses of
methacholine  constrict
airways {FEV1<20%}
suggest asthma
Pulmonary functions tests
(Spirometry) - reveals
FVC, PEFR & FEV1.
The FEV1/FVC ratio is
less than 75%.
The diagnosis of asthma
is established by
demonstrating
reversible airway
obstruction.
Reversibility is
traditionally defined as
a ≥15% increase in FEV1
after two puffs of a β2-
adrenergic agonist.
Chest X-ray may show
hyperinflation.
Blood tests -  eosinophils
& IgE
Skin prick tests - to identify
the allergen in case of
allergic or atopic asthma
Arterial blood gas analysis
shows respiratory alkalosis
and in severe attacks
hypoxia

:
Growth delay in children Status asthmaticus Medication side effects
Pneumonia Rib fractures risk of obesity & depression
: to Suppress inflammation and bronchial
hyper-reactivity

: Beclomethasone dipropionate, Budesonide, Fluticasone and Ciclesonide


: all cases of persistent asthma when inhaled β2 agonists are required almost daily.
: Hoarseness of voice, dysphonia, sore throat & oropharyngeal candidiasis are the MC side effects.
▪ These can be minimized by the use of a spacer, and by gargling after every dose (to wash off the drug deposited on oral and
pharyngeal mucosa)
▪ Long term use → gastric irritation, Na+ and water retention, hypertension, muscle weakness, osteoporosis, HPA axis
suppression, etc.
Inhaled steroids are safe during pregnancy.
:
 Severe chronic asthma: not controlled by bronchodilators and ICS (Inhaled Corticosteroid)
 Status asthmaticus/acute asthma exacerbation
 COPD
(Acute Severe Asthma):
 When an attack of asthma is prolonged with severe
intractable wheezing, it is known as acute severe
asthma.
 Clinical presentation:
Dyspnoea, Cyanosis, paradoxical pulse, use of accessory
muscles, inability to speak in sentences, unable to recline,
and marked hyperinflation of the chest
A silent chest on auscultation suggests that there is no air
movement in and out of lungs due to severe airway
obstruction and is a sign of impending respiratory failure
 Investigations - to rule out other causes of
breathlessness such as pulmonary edema, pneumonia,
pneumothorax, MI etc.

Supplemental oxygen inhalation via venturi masks.
Nebulized β2-adrenergic agonist (salbutamol 5mg/ terbutaline 10 mg) + Anticholinergic Agent
(ipratropium bromide 0.5 mg).
 Salbutamol/terbutaline 0.4 mg i.m./s.c. may be added, since inhaled drug may not reach
smaller bronchi due to severe narrowing/ plugging with secretions
Systemic Glucocorticoids: Hydrocortisone 200 mg IV stat followed by 100 mg 6th hourly
(or)
Oral Prednisolone 30-60 mg/day depending on the patient’s condition.
IV Fluids to correct dehydration
K+ supplements to correct hypokalemia caused due to repeated doses of salbutamol/terbutaline.
Sodium bicarbonate to treat acidosis
IV magnesium sulphate (2 gm infused over 20 min) - in patients not responding to above therapies.
It relaxes bronchial smooth muscle by inhibiting calcium influx.
Intubation and mechanical ventilation, if needed
Antibiotics to treat infection (since Upper respiratory tract infection is the most common precipitant of status asthmaticus).

NSAIDs like aspirin, ibuprofen, diclofenac, etc. (Paracetamol can be used).


β- Adrenergic blockers & Cholinergic agents.
----------------------------------------------------------------------------------------------------------------------------------------
2) Cystic fibrosis – etiology, C/F & Mx [21, 13, 02]
a. Sweat chloride test [10]
Ans. Cystic fibrosis (CF) is a genetic disorder that presents as a multisystem disease.
Cystic fibrosis is characterized by recurrent airway infection (which leads to bronchiectasis), exocrine pancreatic
insufficiency and intestinal dysfunction, abnormal sweat gland function, and urogenital dysfunction.

Investigations of CF
Sweat Chloride test - If sweat Cl- concentration is > 70 mEq/L, it suggests cystic fibrosis.
Genetic testing - for the presence of CFTR gene mutation.
The nasal transepithelial potential difference is raised in CF.
Other useful tests include: chest X-ray, CT chest, pulmonary function tests, etc.
Treatment of CF
Supportive measure:
Avoid smoking
Hypertonic saline inhalation - to increase hydration of airway
Adequate salt should be taken during hot weather since excess salt is lost in the sweat
Adequate immunizations, including influenza, and pneumococcus are mandatory
Antibiotics - For Lung infections. The choice of antibiotics is based on sputum cultures & sensitivity.
Chest Physiotherapy - Chest percussion & postural drainage - to clear purulent secretions.
Bronchodilators - during exacerbations and in hospitalized patients.
Supplementation of Pancreatic Enzymes (at mealtimes) & fat-soluble vitamins.
Ivacaftor - CFTR potentiator - can be tried to improve lung function.
Deoxyribonuclease - {DNA released from neutrophils forms long fibrils which increases the viscosity of sputum}.
Gene Therapy - transfer a normal CFTR gene or cDNA into the affected cells.
Lung transplantation
----------------------------------------------------------------------------------------------------------------------------------------
3) Pneumothorax – Types, etiopath, C/F, inv. & Mx [21, 18, 12]
a. Tension pneumothorax [07]
b. What is the normal anatomy & physiology of pleura? Write the C/F & Mx of pneumothorax [06]
c. Spontaneous pneumothorax [01]
Ans.
Pneumothorax is the presence of air in the pleural cavity
which can either occur spontaneously, or result from
iatrogenic injury or trauma to the lung or chest wall.
:
: Occur due to rupture of
emphysema blebs in smokers.
Most common site is the apical region of lung.
: Due to complication of surgical
thoracic procedures such as lung or pleural biopsy, neck line
insertion, positive pressure ventilation, needle
thoracocentesis or liver abscess aspiration
: Occurs as a result of blunt or
penetrating thoracic trauma

Sudden-onset unilateral pleuritic chest pain or breathlessness.


Decreased or absent breath sounds
& resonant percussion.
By contrast, in tension pneumothorax there
is rapidly progressive breathlessness a/w
tachycardia, hypotension, cyanosis and
tracheal displacement away from the side of
the silent hemithorax.

Chest X-ray shows the sharply


defined edge of the deflated lung with complete translucency (no lung markings)
CT is used in difficult cases to avoid misdirected attempts at aspiration.

✓ Percutaneous needle
aspiration of air - in young
patients
✓ Intercostal tube drainage - for
Secondary pneumothorax.
✓ Supplemental oxygen  rate
at which nitrogen is
reabsorbed by the pleura.
✓ Patients with a closed
pneumothorax should be advised not to fly, as the trapped gas
expands at altitude.
:
It occurs due to an opening which allows air to enter the pleural cavity during expiration but does
not allow its drainage during inspiration
This results in positive intrapleural pressure which causes lung collapse first ipsilateral, then
contralateral mediastinal shift and finally contralateral lung collapse if not treated
: This is a clinical diagnosis and no investigation is necessary.

First take care of airway with cervical spine protection as


per the ATLS protocol
1st step is to do needle decompression in the
midclavicular line in the 2nd intercostal space to drain the air trapped in tension.
Once patient is better, chest X-ray is done
This is followed by tube thoracostomy (intercostal tube drain) through midaxillary line in 5 th or 6th
intercostals space
The final step is wound closure
Give Antibiotic & analgesics
----------------------------------------------------------------------------------------------------------------------------------------
4) Acute Lobar Pneumonia – causes, C/F & treatment [19]
a. Community acquired pneumonia – etiology, C/F, Dx & Mx [11, 08, 06, 05]
b. Lobar pneumonia – etiology, C/F, Dx Complications & Rx [03, 97, 74]
Ans.
Any infection of the lung
parenchyma is known as pneumonia

In lobar pneumonia an entire lobe of


lung is involved.
Clinical Presentation of L. Pneumonia: High STAGES OF MORPHOLOGY
fever with chills, weight loss, cough with LOBAR
GROSS MICRO
PNEUMONIA
mucopurulent sputum
1) Congestion Red Lungs &
There are 4 stages in the course of lobar intra alveolar fluid
(Occurs during first 24 doughy
with many bacteria
pneumonia. hours) consistency
Complications of Lobar Pneumonia Lungs are
Exudate,
Inflammation can involve pleura  Pleuritic 2) Red
red, firm and
neutrophils, and
airless with
chest pain, Pleural effusion & Pleural rub hepatization RBCs filling the
liver-like
Empyema alveolar air spaces
consistency
Pneumothorax - esp. with Staph. aureus Lungs look degradation of red
Lung abscess 3) Gray
grayish cells within the
hepatization
ARDS brown exudate
Sepsis with multiorgan failure 4) Resolution Enzymatic digestion of exudates

Community-acquired pneumonia (CAP) is defined as an acute infection of the pulmonary parenchyma


in a patient who has acquired the infection in the community.
 Bacteria: Streptococcus pneumoniae {MC}, H. influenzae, Moraxella catarrhalis, Mycoplasma
pneumoniae, Legionella, Gram-negative bacilli, anaerobes, Mycobacterium tuberculosis etc.
 Viruses: Influenza virus, parainfluenza virus, respiratory syncytial virus etc.
 Fungi: Cryptococcus, Histoplasma capsulatum etc.
Risk Factors for Pneumonia: HIV; steroid therapy; Uncontrolled DM;
Splenectomy is an important risk factor for pneumonia with S. pneumoniae.
Anatomical defects such as obstructed bronchus, bronchiectasis, or fibrosis lead to recurrent
pneumonia.

CBP - reveals
neutrophils
ESR; C-RP
Chest X-ray
Urine - to detect Pneumococcal or Legionella antigen
Sputum samples - Gram stain, culture and antimicrobial
sensitivity testing
Pleural fluid aspiration and culture with ultrasound guidance
Oropharynx swab - for PCR
serum urea
Cold agglutinins - found in mycoplasma
Arterial blood gases - in severe cases to assess ventilatory failure or acidosis

General Measures - Oxygen; IV fluids;


Bronchodilators and mucolytics may enhance sputum clearance.
Physiotherapy to teach effective coughing techniques
Mechanical ventilation may be required in patients with
respiratory failure.

----------------------------------------------------------------------------------------------------------------------------------------
5) Bronchiectasis – Classify, Causes, C/F, inv. Complications, & Mx [17, 14, 13, 11, 06]
Ans. Bronchiectasis is an abnormal and permanent dilatation of bronchi.
:
Based on onset - congenital or acquired
Based on site - localized (single lobe/segment) or Diffuse (Both Lungs)
Based on shape - 3 types -- cylindrical, fusiform, and saccular (cystic).
 In the cylindrical type, there is uniform dilatation of bronchi.
 In the fusiform type, dilatation is irregular with tapering at both ends.
 In saccular type, there are multiple bulging from side wall of bronchi.

Development of bronchiectasis is mainly due to two factors; infection


and obstruction or both.
▪ Infections: Childhood pneumonias as in measles, whooping cough,
Pulmonary tuberculosis, aspergillosis etc.
▪ Bronchial Obstruction: Foreign body, Tumor, Stenosis, enlarged lymph nodes, Impacted secretions etc.
▪ Miscellaneous: Cystic fibrosis, Kartagener's syndrome, α1 antitrypsin deficiency, Immotile cilia
syndrome etc.
Infection leads to inflammatory destruction of the bronchial wall  impairs mucociliary
clearance pooling of secretions, and dilatation of bronchi.
Dilated bronchi become more susceptible to infection and thus, a vicious cycle results.

In congenital cases - failure to thrive (due to defect in immunity & mucus clearance)
In adults - Persistent or recurrent cough with copious sputum for several years.
Sputum is blood-stained (Hemoptysis), foul-smelling; a/w fever - indicates secondary infection
Some patients may have no sputum with cough. This entity is called bronchiectasis sicca.
Patients may have dyspnea and wheezing when underlying COPD is also present.
Physical examination:
▪ Clubbing {due to hypoxia}.
▪ Persistent inspiratory adventitious sounds (crackles>wheezing) and bronchial breath sounds over
the area of bronchiectasis reflecting damaged airways with secretions.

Chest X-ray may show cystic lesions in cystic bronchiectasis.


 When seen longitudinally, the dilated and thickened bronchi appear as "tram tracks".
 Honeycombing (ring shadows) reflecting end on view of dilated bronchi can be seen in diffuse cases.
Bronchography (radiopaque dye into airways) and then taking X-ray images. But now this
technique has been replaced by HRCT.
HRCT of the chest is now the preferred method for diagnosis
Bronchoscopy may be done if a foreign body or adenoma is suspected to be the cause.
: Massive haemoptysis (in bronchial artery erosion), empyema, respiratory failure, cor
pulmonale, pericarditis, metastatic abscesses, and secondary amyloidosis.

-
Expectorants, bronchodilators and Broad-spectrum antibiotics for secondary infections.
Postural drainage of the secretions
Mucolytics like N-acetylcysteine and bromhexine
Regular physiotherapy prevents accumulation of secretions and repeated infections.
- for localised bronchiectasis
can be considered in patients with recurrent large hemoptysis.
----------------------------------------------------------------------------------------------------------------------------------------
6) Pleural effusion – causes, C/F, DDx, Lab inv. and Mx [16, 11, 09, 04]
a. Light's Criteria for Pleural Effusion [21]
b. Pleural fluid analysis in synpneumonic effusion [19]; Pleural fluid analysis in malignant pleural effusion [18, 14]
c. Pleurodesis [18]
d. Technique of pleural aspiration [10]
Ans.
Pleural effusion is an abnormal collection of fluid
in the pleural space
:

 Dyspnea & Chest pain - indicates pleural


irritation - Pleuritic pain -- sharp or stabbing
in nature and is exacerbated with deep   chest movements
inspiration  Vocal fremitus & vocal resonance
Pain may be absent in severe pleural effusion  Stony dull percussion note.
(separates inflamed pleural surfaces from each other)  Absent breath sound on the affected side;
 Cough - non-productive; Severe cough with Pleural rub may be heard
purulent sputum suggests an underlying  Signs of underlying disease may be present -
pneumonia Ex: Peripheral edema, distended neck veins,
 Symptoms of underlying disease may be and S3 gallop suggest CCF
present - Ex: Lower limb edema, orthopnoea,
and PND in CCF

Chest X-ray: Pleural effusion appears as a curved


shadow at the lung base, blunting the costophrenic
angle and ascending towards the axilla on the erect
chest X-ray. Minimum 200 ml of fluid is required to
produce a shadow on chest X-ray
Ultrasound chest - more accurate than chest X-ray
& can also be used to guide pleural aspiration and
pleural biopsy.
CT scan - better than both ultrasound & x-ray
Pleural fluid aspiration and analysis - to know the cause
Colour - Ex: Straw colored in transudates, turbid and purulent in empyema and hemorrhagic in
pulmonary infarction or malignancy.
Biochemical analysis allows classification into transudate and exudates (see lights criteria).
 Adenosine deaminase level (ADA) & IFN- γ in pleural fluid is highly suggestive of TB.
 triglyceride and cholesterol levels are seen in chylothorax.
Microbiological investigations & PCR to know the causative agent
Pleural biopsy - An Abrams needle is used for pleural biopsy under ultrasound or CT guidance
Asymptomatic transudative effusions need not be drained.
Treatment of the underlying cause: For example, heart failure, nephritic syndrome, pneumonia, etc.
will often be followed by resolution of the effusion
Thoracentesis (pleural tap) to remove excess fluid in symptomatic patients (e.g., dyspnea).
Insertion of intercostal drainage (ICD) tube - in complicated parapneumonic effusions & empyema.
Pleurodesis: used for recurrent malignant effusions
 It involves instilling an irritant (such as talc, doxycycline) into the pleural space to cause
inflammatory changes that result in bridging fibrosis between the visceral and parietal pleural
surfaces, obliterating the pleural space.
----------------------------------------------------------------------------------------------------------------------------------------
7) Enumerate causes of suppurative lung diseases. Discuss the C/F & Mx of Empyema Thoracis [16]
a. Lung Abscess – Aetiology (mode of formation), C/F & Mx [15, 90]
Ans.
Suppurative Conditions of Lungs: They are characterized by the production of large amounts of purulent exudate.
 Causes: Pulmonary TB, Cystic Fibrosis, Lung abscess, Bronchiectasis, Pneumonia, Atelectasis etc.
- It is collection of pus in
pleural cavity.
:
:
 Chest is withdrawn inwards and is immobile {frozen
chest} Difficulty in breathing
 Pain in the chest, tenderness, fever
 Dullness on percussion, absence of breath sounds
 Often pus perforates through intercostal space and
forms empyema necessitans.
: Frozen chest (functionless lung); Empyema necessitans; Osteomyelitis/chondritis of
ribs or vertebra; Pericarditis; Mediastinitis; Bronchopleural fistula; Dissemination of infection
: Chest X-ray, ESR; Peripheral smear; Diagnostic aspiration; Pus C/S, AFB; Bronchoscopy;
CT scan and MRI for carcinoma bronchus
:
Stage 1: Antibiotics; Repeated aspirations; Intercostal tube drainage; Anti-tuberculous drugs
Stage 2: ICT drainage; Rib resection (Eloiser's method); Antibiotics, Anti-tuberculous drugs;
Respiratory physiotherapy
Stage 3: Decortication via thoracotomy approach; lobectomy; pneumonectomy

It is a localized collection of purulent exudates within the lung.


:
Organisms: Anaerobic bacteria (MC), ± mixed aerobic infections like Staph. aureus, K. pneumoniae,
Strep. pyogenes and Pneumococcus
Predisposing conditions: Oropharyngeal surgical or dental procedure; Sinobronchial infections;
Bronchiectasis
Mode of Formation: Lung abscess may be primary or secondary.
Primary Lung Abscesses develop due to aspiration of infective material from infected nasal
sinuses or tonsils
Single or multiple abscesses, varying in size & may affect any part of lung.
: Cough with foul- smelling, purulent sputum, fever, chest pain, weight loss, &
clubbing of digits
:
Septic emboli may cause brain abscesses or meningitis.
Extension of the infection into the pleural cavity → empyema/pneumothorax
Bronchopleural fistula
Pericarditis
Massive hemoptysis

IV clindamycin or amoxicillin-clavulanate can be used - till C/S report.


Repeat Chest X-ray after 6 to 8 weeks of antibiotic therapy to check for resolution of abscess.
Physiotherapy in the form of postural drainage can help clear the secretions.
Chronic abscesses not responding to medical therapy require surgical resection
----------------------------------------------------------------------------------------------------------------------------------------
8) Pulmonary TB - C/F, Dx and treatment; Mention the Adverse Rxns of major drugs. [07, 06]
a. MDR-TB – Dx & Mx [22, 10, 08, 04, 03]
b. DOTS regimen [11, 05]
c. RNTCP [09]
d. Mantoux test [09]; Tuberculin test [05] ➔ refer pg. no. 69 in Paper-II
e. Histology of tuberculosis [08]
f. Primary complex [03, 01]
Ans.
Features Primary Pulmonary tuberculosis Secondary Pulmonary TB
Occurs 1. Reinfection
Infection with tubercle bacilli
due to 2. Activation of the latent primary lesion
Age
Usually affects Children Adults
group
Lungs
Site Apex of upper lobes
(Upper lobe - Lower part; Lower lobe – Upper part)
Ghon focus: 1-1.5 cm gray-white lesion Small circumscribed, gray-white to yellow
with cheesy white appearing center. apical foci with central cheesy-white
Lesions Ghon complex: Ghon focus + lymphadenopathy . appearance are seen
Fibrosis Erosion into airways is associated
Ranke complex: Radiologically detectable
calcifications
Assmann focus [calcified nodules]
Asymptomatic/ fever, productive cough • Similar but severe.
Clinical
(with/without hemoptysis) & occasionally Necrosis & tissue destruction → cavity
Picture:
chest pain, night sweating, weight loss formation
Necrotic material breaks into the airways, causing:
Expectoration of bacteria-laden sputum
1) Healed lesion - organisms are not visible. Hematogenous spread to other organs
Bronchogenic spread to the same or opposite lung →
Fate: 2) Latent lesion - organisms are dormant. forms satellite lesions, which coalesce to produce
3) May progress to Miliary TB caseating pneumonia
Fibrocalcific scars in immunocompetent persons
May progress to Miliary TB
Under RNTCP (Revised National Tuberculosis Control Programme), DOTS chemotherapy is being implemented
DOTS is a strategy to ensure cure by providing the most effective medicine & confirming that it is taken
In DOTS, during the intensive phase of treatment a health worker or other trained person watches as
the patient swallows the drug in his presence.
During continuation phase,
the patient is issued
medicine for 1 week in a
multiblister combipack of
which the 1st dose is
swallowed by the patient
in the presence of health
worker or trained person.
The consumption of
medicine in the
continuation phase is also
checked by return of empty
multiblister combipack,
when the patient comes to
collect medicine for the next week
:

• DOC for TB preventive therapy


(Chemoprophylaxis): H (isoniazid)
– for known contacts of pulm. TB
case
• MDR-TB - resistant to both
rifampicin & isoniazid
• RNTCP was renamed to 'National
Tuberculosis Elimination Program
(NTEP) on 1st January 2020 with
the aim to eliminate TB by 2025
in India

----------------------------------------------------------------------------------------------------------------------------------------

1. Pulmonary Thromboembolism [21]


a. Pulmonary embolism [08]
b. Acute pulmonary thromboembolism – etiopathogenesis, C/F, inv. & Mx [2000]
Ans.
Pulmonary embolism (PE) refers to exogenous or endogenous material traveling to the lungs blocking
pulmonary artery or its branches.
: DVT of the lower legs; septic emboli (from endocarditis affecting the
tricuspid or pulmonary valves), tumour (especially choriocarcinoma), fat following fracture of long
bones such as the femur, air, and amniotic fluid, which may enter the mother’s circulation
following delivery
Miscellaneous Pulmonary Emboli:
:
» Talc emboli: in IV drug abusers who use talc as a carrier for illicit drugs.
» Cotton emboli occur due to cleansing of the skin by cotton prior to venepuncture.
occur during hematogenous dissemination of cancer
: Fragments of atheromatous plaque may embolize.
: DVTs undergo fragmentation → Thromboemboli are carried through progressively
larger vascular channels → into the right side of the heart → Right ventricle → enter into the
pulmonary arterial vasculature
: Main pulmonary artery, bifurcation of pulmonary artery, or smaller, branching
arteries.

Chest X-ray PA view - Westermarck sign


CT scan and MRI
Pulmonary angiography is "gold standard" for
diagnosis
Doppler study and venography – to rule out DVT.

General Measures:
 High-flow oxygen (60-100%) should be given to all patients.
 IV fluids and inotropic agents (dopamine and noradrenaline) should
be started if there is hypotension.
 Intubation & mechanical ventilation
- if there is respiratory compromise.
Anticoagulation: LMWH (e.g.,
dalteparin, enoxaparin); unfractionated
heparin; Oral warfarin etc.
----------------------------------------------------------------------------------------------------------------------------------------
2. Klebsiella pneumonia [21]
Ans:
Klebsiella pneumoniae is responsible for severe lobar pneumonia, UTIs, meningitis (neonates),
septicemia and pyogenic infections such as abscesses and wound infections
It frequently colonizes the oropharynx of the hospitalized patients  nosocomial infections
Clinical Features - More common in men, alcoholics and diabetics.
Pneumonia tends to be destructive with production of thick, mucoid, sputum.
Some time, the sputum has a thin and currant jelly-like appearance.
Investigations
Chest X-ray: Upper lobe involvement is common
Homogeneous lobar or segmental opacity.
Multilobar shadowing. cavitation, and pneumatoceles
Abscess may be present with an air fluid level.
Sputum: Gram stain and culture sensitivity
Treatment - Treatment of Klebsiella is global challenge owing to marked drug resistance
Preferred: Third-generation cephalosporin. For severe infections, add an aminoglycoside.
Alternatives: Aztreonam, imipenem, meropenem, aminoglycoside, or a fluoroquinolone.
----------------------------------------------------------------------------------------------------------------------------------------
3. Extra pulmonary manifestations of bronchogenic carcinoma [21]
a. Paraneoplastic manifestations in lung cancer [16]
Ans. Paraneoplastic syndromes are clinical syndromes due to nonmetastatic systemic effects of a cancer
They occur due to substances produced by the tumor
Paraneoplastic syndromes resolve by treatment of the primary tumor
Paraneoplastic manifestations in lung cancer (bronchogenic carcinoma) are:
SIADH  hyponatraemia <==small cell cancer
Ectopic ACTH hormone secretion <==small cell cancer
Endocrine Hypercalcaemia & hypophosphatemia - due to secretion of PTH-related peptides <==
manifestations squamous cell carcinoma
Carcinoid syndrome
Gynaecomastia
Polyneuropathy & Myelopathy
Cerebellar degeneration
Neurological
Myasthenia (Lambert–Eaton syndrome} & Retinal blindness <==small cell
cancer
Rheumatologic Hypertrophic pulmonary osteoarthropathy - Digital clubbing & tenderness over
manifestations the long bones
Hematologic Anemia, thrombocytosis, DIC & leukemoid reactions
Systemic
Anorexia, cachexia, weight loss, fever, and suppressed immunity
manifestations
Nephrotic syndrome
Other
Polymyositis and dermatomyositis
manifestations
Eosinophilia
----------------------------------------------------------------------------------------------------------------------------------------
4. Types of respiratory failure [21]
a. Enumerate salient differences between different types of respiratory failure. Mention in brief the treatment principle [12, 10]
b. Type I respiratory failure. [09]
c. Respiratory failure – Classify, Etiology & clinical signs [07, 03]
Ans. The term ‘respiratory failure’ is used when pulmonary gas exchange fails to maintain normal
arterial oxygen and carbon dioxide levels.
- based on absence or presence of hypercapnia:
Types of RF Causes Typical Features
It is caused by processes that impair oxygen
Type 1 transfer in the lung, e.g., Acute asthma, present,
{ RF} pulmonary edema, pulmonary embolism, pCO2 normal or low
pneumonia & ARDS
It is caused by inadequate ventilation which leads
Type 2 to retention of CO2 & hypoxemia, e.g., COPD,
{ RF} myasthenia gravis, brainstem injuries, Narcotic Hypoxemia usually present
drugs and stroke
Here, there is a combination of type I and type II respiratory failure (acute-on
Mixed RF
chronic respiratory failure).
Respiratory failure may be further classified as either acute or chronic.
Acute respiratory failure develops over minutes to hours, e.g., pneumothorax, and pulmonary
edema.
Chronic respiratory failure develops over several days or longer, e.g., COPD.
:
Of the underlying disease will be present
Related to Hypoxemia - Dyspnea, cyanosis, restlessness, confusion, anxiety, delirium, tachypnea,
hypertension, cardiac arrhythmias, and tremor.
Related to Hypercarbia - Dyspnea, headache, peripheral flushing, bounding pulses, hypertension,
tachycardia, tachypnea, altered sensorium, papilledema, and flapping tremors (asterixis).

Identify & treat the underlying disease - Ex: pulmonary edema, COPD, myasthenia gravis, etc.
which have caused respiratory failure.
Respiratory support - supplemental oxygen
Low flow oxygen should be used in COPD because their respiratory drive may be due to
hypoxia.
Mechanical ventilator support is required if the patient is not responding to oxygen
supplementation.
Extracorporeal membrane oxygenation (ECMO) is indicated in severe respiratory failure not
responding to even mechanical ventilation.
General supportive care - Adequate hydration & nutrition, preventing stress ulcers in the stomach
by using sucralfate syrup, preventing bedsores, and preventing DVT by using subcutaneous heparin
or low molecular weight heparin
----------------------------------------------------------------------------------------------------------------------------------------
5. ARDS [20, 13, 08, 01]
Ans.
Chest X-ray - bilateral infiltrates which become confluent with sparing of costophrenic angles.
Heart size is normal, and pleural effusions are nil or minimal.
ABG analysis shows marked hypoxemia that is refractory to supplemental oxygen.
Bronchoscopy and lung biopsy to know cause of ARDS

by tracheal intubation, supplemental oxygen and positive-pressure mechanical


ventilation.
Prone positioning to improve oxygenation
Fluid administration should be restricted and enough to maintain adequate cardiac output.
- to  intravascular volume if pulmonary capillary wedge pressure is elevated.
have shown benefit in late-phase ARDS.
General supportive care - Adequate hydration & nutrition, preventing stress ulcers in the stomach
by using sucralfate syrup, preventing bedsores.
----------------------------------------------------------------------------------------------------------------------------------------
6. Pulmonary function tests (Name two) [20, 12, 09]
a. FEV1 [19]
b. Peak flow rate and its significance [10]
c. Vital capacity [07]
Ans.
Indications for Pulmonary Function Testing
Assessment of the type and severity of lung dysfunction.
Diagnosis of causes of dyspnea and cough.
Monitoring lung function in certain occupations at high risk of lung damage (e.g. mine workers).
Monitoring response to treatment.
Preoperative assessment.
Disability evaluation.
Pulmonary Function Tests – this includes the following:
– (spirometry)  3 parameters  Lung volumes, Lung capacities &
Mechanics of breathing
1. Tidal volume – volume of air inspired or expired during quiet breathing  500ml in adults
2. Inspiratory reserve volume – volume of air inspired with a maximal inspiratory effort in excess of
the tidal volume  3L in men & 2L in women
Lung
3. Expiratory reserve volume – volume of air that can be expired with a maximum expiratory effort
Volumes after passive expiration  1L in men & 0.7L in women
4. Residual volume – volume of air left in the lung at the end of a maximal expiratory effort  1.2L
in men & 1.1L in women
Vital capacity (VC) is the maximum amount of air that can be expired forcefully after a maximal
inspiratory effort; VC = TV + IRV + ERV;  4.5L in men & 3.3L in women
Inspiratory capacity – it is the max. amount if air that can be inspired from the resting expiratory
Lung level; IC = IRV + TV  3.5L
Capacities Functional residual capacity – it the amount of air remaining in the lungs at the end of a normal
expiration at rest; FRC = ERV + RV  2L
Total lung capacity – it is the amount of air present in the lungs at the end of maximal inspiration;
TLC = VC + RV  6L in men & 4.2L in women
Timed vital capacity – aka FEV1 = Forced Expiratory Volume in 1st second – it’s the fraction of VC
expired in the 1st second  80-85% normally; it helps to detect airway obstruction
MMEFR – maximum mid-expiratory flow rate – it is the max. flow achieved during middle third of
total expired volume
MVV – maximum voluntary ventilation – aka max. breathing capacity {MBC} – it is the max.
Mechanics volume of air that can be breathed out per minute by maximal voluntary effort
of PEFR – Peak Expiratory Flow Rate –
breathing  It is the max. velocity in litres per minute with which air is forced out of the lungs  6-
10L/sec normally;
 It is measured by a peak flow meter
 PEFR is reduced in airway narrowing and expiratory muscle weakness
MEFV – maximum expiratory flow volume curve
Closing volume – it detects small airway obstruction
Obstructive dysfunction -  ratio of FEV1 to FVC  seen in asthma, COPD, bronchiectasis,
bronchiolitis, and upper airway obstruction
Restrictive dysfunction – normal to increased FEV1/FVC ratio seen in interstitial lung diseases,
weak respiratory muscles, pleural disease, and prior lung resection

–Uniformity of ventilation is
assessed by:
1) Nitrogen washout method (Breath nitrogen test)
2) Radioactive xenon method
– measure pO2 &
pCO2 in arterial blood

Ex: pulmonary artery pressure


etc.
----------------------------------------------------------------------------------------------------------------------------------------
7. Hemoptysis [19]
a. Mention the causes of hemoptysis [17, 13]
Ans.
Hemoptysis is coughing out of blood that originates below the vocal cords.
Lungs have two sources of blood supply: Pulmonary and Bronchial arteries. Though it contributes
to only 1-2% of total pulmonary blood flow, bronchial circulation is frequently the source of
hemoptysis
Hemoptysis is classified as trivial, mild, or massive.
Massive hemoptysis is defined as coughing out more than 200-600 ml in 24 hours or that which
leads to hemodynamic compromise.
Causes of Hemoptysis
Respiratory causes: Tuberculosis (MC), Chronic bronchitis,
Bronchiectasis, Bronchogenic carcinoma, Aspergilloma,
Pulmonary embolism, Pneumonia, Lung abscess
Cardiac causes: Left ventricular failure, Mitral stenosis
Hematologic causes: Thrombocytopenia, Hemophilia, DIC
Clinical Features
Patient gives h/o of blood-streaking of sputum or frank
hemoptysis.
Massive hemoptysis may have signs of hemodynamic compromise
such as tachycardia, hypotension and cold peripheries.
Symptoms and signs of underlying disease causing hemoptysis may be present.
Investigations
 Chest X-ray
 Hemoglobin, PCV, complete blood count, including platelet count, RFTs, urinalysis, and coagulation
studies should be done.
 HRCT can diagnose unsuspected bronchiectasis & AV malformations
 Fibreoptic bronchoscopy can also be done
Management
Identify & treat the cause of hemoptysis.
In Massive hemoptysis –
Maintain airway and breathing. Place the Patient in the lateral decubitus position with the
involved lung dependent so that the blood does not enter the other lung.
IV fluids & blood transfusion.
Cough suppressants such as codeine syrup and mild sedation (with benzodiazepines).
Nebulized adrenaline
Oral tranexamic acid (500 mg t.i.d)
Uncontrollable hemoptysis needs rigid bronchoscopy and specific intervention like angiography
and embolization of the involved bronchial arteries.
Lung resection if not responding to any of the above measures.
----------------------------------------------------------------------------------------------------------------------------------------
8. Management of Acute Pulmonary Edema [16]
a. X-ray chest in pulmonary oedema [06]
Ans.
Acute pulmonary edema refers to the rapid buildup of fluid in the alveoli and lung interstitium that has
extravasated out of the pulmonary circulation
As the fluid accumulates, it impairs gas exchange and decreases lung compliance, producing dyspnoea
and hypoxia
:
Cardiogenic causes – Acute MI; Acute volume overload; MS; Atrial fibrillation; Infective
endocarditis etc.
Non-cardiogenic causes – ARDS; Eclampsia; Immersion/submersion; Toxic inhalation; High altitude;
Head injury etc.

Chest X-ray shows increased interstitial markings & butterfly pattern of distribution of alveolar
edema.
 Kerley’s B lines may be seen due to thick and tense lymphatics.
 Cardiomegaly may be present.
ECG may show evidence of ischemia, infarction, and arrhythmias.
Echocardiography shows low ejection fraction & atrial pressures in cardiogenic PE
Pulmonary capillary wedge pressure (PCWP) in cardiogenic pulmonary edema.
In non-cardiogenic pulmonary edema, the wedge pressure may be normal or even low

Treat the underlying cause in non-cardiogenic PE


Treatment of Cardiogenic Pulmonary Edema
General measures – sitting position with legs hanging down the side of the bed; High flow
oxygen & ventilatory support.
Morphine – venous capacitance & also relieves anxiety; Dose: 2–8 mg IV; repeat after 2–4 hours.
Diuretics – IV (furosemide, 40 mg, or torsemide 20 mg or higher doses) to  fluid overload.
Nitroprusside, nitroglycerin, and Nesiritide – act rapidly
Inotropic drugs – Ex: dopamine, dobutamine, amrinone & milrinone may improve cardiac
output and decrease pulmonary edema.
----------------------------------------------------------------------------------------------------------------------------------------
9. Mechanical ventilators [09]
Ans.
Mechanical ventilation is a method to mechanically assist or
replace spontaneous breathing by using a mechanical
ventilator

Non-invasive method – here, ventilation is provided


through a tight-fitting face mask, e.g., NPPV (non-invasive
positive pressure ventilation).
Invasive method – endotracheal intubation or
tracheostomy
:
(controlled mechanical ventilation) – used in patients who are making no respiratory effort
(e.g., spinal cord injury)
(assist control) – used in patients who are making little respiratory effort
Other modes: (intermittent mandatory ventilation); (synchronized intermittent
mandatory ventilation); (continuous positive airway pressure ventilation)
Barotrauma – rupture of alveoli  pneumothorax or systemic gas embolism
Atelectasis of the contralateral lung and overdistension of the intubated lung.
Acute respiratory alkalosis can occur due to hyperventilation.
Hypotension – because excess intrathoracic pressure prevents venous return to heart
Tracheal stenosis can occur if endotracheal tube is kept for long time.
Ventilator-associated pneumonia is another serious complication.
----------------------------------------------------------------------------------------------------------------------------------------
10. Tropical Pulmonary Eosinophilia [08, 04, 03]
a. Write about Pulmonary eosinophilia [20]
Ans.
Tropical pulmonary eosinophilia (TPE) is a syndrome that develops in some individuals with
lymphatic filariasis.
The clinical symptoms are due to allergic & inflammatory reactions elicited by the microfilariae in
the lungs
Interstitial fibrosis and lung damage can happen

Males are affected commonly from filaria-endemic areas.


They usually present with nocturnal dry cough and wheezing, weight loss & low-grade fever

Eosinophil count is high (usually >3000 eosinophils/µl).


Chest X-ray – reveal bronchovascular markings, diffuse miliary lesions or mottled opacities.
Pulmonary function tests show both restrictive and obstructive defects.
Serum IgE levels and anti-filarial antibodies are elevated.
: DEC – Dose: 4 to 6 mg/kg of body weight divided into 2 or 3 doses per day for 14 days.
----------------------------------------------------------------------------------------------------------------------------------------
11. Atypical pneumonia [05]
Ans.
Atypical (or interstitial) pneumonia refers to infection occurring in the interstitial space of lung
Cough is characteristically non-productive
:
 Respiratory viruses – Influenza viruses, corona viruses, RSV, EBV and adenoviruses
 Bacterial agents:
▪ Mycoplasma pneumoniae – aka “walking pneumonia” (as symptoms tend to be milder than others agents causing pneumonia)
▪ Chlamydia
▪ Legionella species  Legionnaires’ disease
▪ Rare: Coxiella burnetii (Q fever), Francisella tularensis (pulmonary tularemia), and Orientia
tsutsugamushi (scrub typhus).
: depends on the cause
Cold agglutination test +ve in mycoplasma
DOC in mycoplasma oral azithromycin, 500 mg on day 1, then 250 mg on days 2 to 5)
----------------------------------------------------------------------------------------------------------------------------------------
12. Stress Test [05]
Ans:
This is done in patients with unexplained dyspnea.
A bicycle ergometer or treadmill is used.
Minute ventilation, expired oxygen and CO2 tension, heart rate, blood pressure, and respiratory
rate are monitored.
Significance: it detects arrhythmias
----------------------------------------------------------------------------------------------------------------------------------------
13. Allergic broncho pulmonary aspergillosis [05]
Ans.
Aspergillosis refers to illness due to allergy, colonization, or tissue invasion by species of
Aspergillus.
Aspergillus species are A. fumigatus (most common), A. flavus, A. niger, A. nidulans etc.
Risk factors: immunocompromised and diabetic persons.

Allergic bronchopulmonary aspergillosis (ABPA) occurs in patients with asthma and cystic fibrosis
and lead to worsening of wheezing and breathlessness.
Patients usually have a history of chronic allergic rhinitis
Invasive aspergillosis pneumonia can occur in immunosuppressed individuals and is difficult to treat .

 Detection of hyphae in clinical specimens;


 Chest X-ray – reveals fungus ball in the lung.
 IgG antibody to Aspergillus antigens.
 Serum IgE antibody.
 Biopsy of the lungs, nose, and paranasal sinuses, etc.

Allergic bronchopulmonary aspergillosis responds to short courses of steroids.


Invasive aspergillosis is treated with voriconazole, or itraconazole or liposomal or conventional
amphotericin B.
Fungus ball of the lung usually requires lobectomy.
----------------------------------------------------------------------------------------------------------------------------------------
14. Fibreoptic Bronchoscopy [04]
Ans.
It is easy to use in patients with neck or jaw abnormalities and injuries where rigid bronchoscopy may
almost be impossible technically.
This can be performed under topical anaesthesia and is very useful for bedside examination of the
critically ill patients.
Flexible bronchoscope can also be easily passed through endotracheal tube or the tracheostomy
opening. However, it has limited utility in children because of the problems of ventilation.
----------------------------------------------------------------------------------------------------------------------------------------
15. Croup [04]
Ans.
Acute Laryngotracheobronchitis is an inflammatory condition of the larynx, trachea and bronchi
AETIOLOGY: To begin with, it is viral in origin (parainfluenza type I and II) but soon bacterial (G+ve)
invasion takes place. Male children are more often affected
PATHOLOGY: The loose areolar tissue in the subglottic region swells up and causes respiratory obstruction
and stridor. This, coupled with thick tenacious secretions and crusts, may completely occlude the airway.
CLINICAL FEATURES
o Disease starts as upper respiratory infection with hoarseness and croupy cough + mild fever.
o This may be followed by difficulty in breathing and inspiratory type of stridor.
o Steeple Sign on AP view of Neck X-Ray – {subglottic tracheal narrowing produces the shape of a church
steeple within the trachea itself}
TREATMENT
1) Hospitalisation – Administer inhalation anaesthesia (sevoflurane) and oxygen to the patient, secure
i.v. line and then perform laryngoscopy to make the diagnosis. Take laryngeal swabs for culture and
sensitivity tests and intubate the patient.
2) Antibiotics like ampicillin are effective against secondary bacterial infections.
3) Humidification helps to soften crusts and tenacious secretions which block tracheobronchial tree.
4) Parenteral fluids – to combat dehydration.
5) Steroids, e.g., hydrocortisone 100 mg i.v. may be useful to relieve oedema.
6) Racemic adrenaline administered via a respirator is a bronchodilator and may relieve dyspnoea
7) Tracheostomy is done if intubation is required beyond 72 h.
----------------------------------------------------------------------------------------------------------------------------------------
16. Acute Bronchitis [03]
Ans.
Acute bronchitis is inflammation of medium-sized airways.
It usually develops as a complication of an upper respiratory tract infection or as an exacerbation
of acute infection in COPD.

 It is usually due to viral infections  adenovirus, rhinovirus or influenza virus in adults and RSV or
parainfluenza virus in children and the elderly
 Secondary bacterial infection with Strep. pneumoniae and H. influenzae can occur.
 Atypical infections with Mycoplasma pneumonia, Chlamydia can rarely present as acute bronchitis.

Patient c/o fever, malaise and dry cough. There can be scanty mucoid sputum which may later
become mucopurulent. Dyspnea with wheezing is usually present.
Examination shows diffuse B/L rhonchi on auscultation + signs of upper respiratory tract infection.

 Chest X-ray is usually normal.


 Total leucocyte count may be high.
 Sputum gram stain and culture can give an idea about the infecting organism.

Antibiotics are prescribed if bacterial infection is suspected.


Cough syrups give symptomatic relief.
Bronchodilators may be needed if there are rhonchi on auscultation.
----------------------------------------------------------------------------------------------------------------------------------------
17. Chronic bronchitis [2000]
Ans.
It is defined as a chronic productive cough for 3 months in each of two successive years is a patient in
whom other causes of chronic cough (e.g.: bronchiectasis) have been excluded.
Risk Factors
 Cigarette smoking
 Airway hyperresponsiveness
 Occupational exposures: coal mining, gold mining, cotton textile dust etc
 Air pollution
 Genetic factors – e.g., α1anti-trypsin deficiency
Clinical Features
Patients with chronic bronchitis are called "blue bloaters," due to presence of cyanosis and fluid
retention
Patient sits in a characteristic "tripod" position to facilitate the actions of accessory muscles.
Signs of hyperinflation of lungs – barrel-shaped chest, pushed down diaphragm, and obliteration
of cardiac dullness. Tidal percussion reveals decreased movement of diaphragm as it is already
pushed down.
Investigations –in FEV1 and FEV1/FVC.
 Chest X-ray – reveal bronchovascular markings.
 ECG may show evidence of right ventricular hypertrophy.
Treatment – Only smoking cessation and oxygen therapy will alter the course of COPD. All other
treatments are aimed at improving symptoms and decreasing the frequency of exacerbations.
Drug therapy in chronic bronchitis
β2-agonists: SABA, LABA
Anticholinergics/muscarinic antagonists: SAMA, LAMA
Methylxanthines
Combination long-acting β2-agonists + corticosteroids in one inhaler
Systemic corticosteroids
Phosphodiesterase-4 inhibitors
----------------------------------------------------------------------------------------------------------------------------------------
18. BCG Vaccine [2000] ➔ pg. 63 in Paeds notes
----------------------------------------------------------------------------------------------------------------------------------------

1) Two causes of Solitary Pulmonary Nodule [21]


Ans:
Benign causes: Infectious granulomas, Wegener's granuloma, Localised pneumonia, Lung abscess,
Pulmonary infarct, Hydatid cyst, Bronchogenic cyst, Rheumatoid nodule, Pulmonary sequestration,
Pulmonary haematoma
Malignant causes: Primary lung cancer, Carcinoid tumor, Single metastasis, Lymphoma
---------------------------------------------------------------------------------------------------------------------------------------
2) Bronchodilators [21]
Ans.

---------------------------------------------------------------------------------------------------------------------------------------
3) Pneumoconiosis [18]
a. Name two occupational lung diseases [20, 11]
Ans:
▪ Pneumoconiosis is an interstitial fibrosis of the lungs which occur because of occupational
exposure to dust within the size range of 0.5 to 3 micron
▪ The important dust diseases are silicosis, anthracosis (coal dust), byssinosis (cotton dust), Bagassosis,
asbestosis and farmer's lung
▪ As no cure for the pneumoconiosis is known, it is essential to prevent these diseases from arising

----------------------------------------------------------------------------------------------------------------------------------------
4) Interstitial lung disease [11]
Ans.
The interstitial lung diseases (ILDs) also known as diffuse lung diseases (DLD) are a heterogenous
group of disorders characterized by diffuse parenchymal lung involvement.
Parenchyma of the lung includes – the alveoli, the alveolar epithelium, the capillary endothelium,
and the spaces between these structures.
Causes of ILD: Pneumoconiosis; Tropical pulmonary eosinophilia’ Sarcoidosis; SLE; Polymyositis;
dermatomyositis; Sjogren's syndrome; Pulmonary vasculitis etc.
----------------------------------------------------------------------------------------------------------------------------------------
5) Shifting dullness in chest [10]
Ans.
This is done to demonstrate the shift of fluid in pleural effusion and hydropneumothorax.
Shift of fluid in the chest can be demonstrated by the dull area percussed in the axilla in the sitting
posture, becoming resonant on lying down on the healthy side
In hydropneumothorax shifting occurs immediately, whereas it is very slow in case of pleural
effusion.
----------------------------------------------------------------------------------------------------------------------------------------
6) Cor pulmonale [08]
Ans.
Cor pulmonale is enlargement of the right ventricle with or without failure, secondary to diseases of
the lung, thorax, or pulmonary circulation
: Features of RV failure and evidence of primary lung diseases.

: prominent RA, RV and pulmonary artery.


: P pulmonale, right axis deviation, RVH with strain pattern.
: Secondary polycythaemia.

Treat the primary lung disorder


To workload - start diuretics, vasodilators & bronchodilators
Control infection (if found).
----------------------------------------------------------------------------------------------------------------------------------------
7) Types of Bronchial Breath Sounds [05, 03]
Ans.
The bronchial breath sound originates in the larger airways and are transmitted directly to the
chest wall without passing through the alveoli
Bronchial breath sounds are of 3 types: tubular, cavernous & amphoric.
- They are high pitched and present in - Pneumonic consolidation & Collapsed lung
- They are low pitched and heard in the presence of thick-walled cavity with a
communicating bronchus.
- They are low pitched, with a high tone and a metallic quality and present in:
Bronchopleural fistula & Tension pneumothorax.
1) Name anterior pituitary hormones. Thyrotoxicosis – etiology, C/F, Inv. & Mx [22, 15, 08]
a. Grave's Disease (Exophthalmic Goitre) – C/F, complications & Mx [14]
b. Thyroid crisis [10]
c. Thyrotoxic crisis – C/F, Dx, complications & Mx [05]
d. Thyroid storm [02]
Ans.
Anterior pituitary hormones: 👉🏻
Thyrotoxicosis
:
 Diseases of thyroid: Graves’ disease; Multinodular goitre;
Thyroiditis; Thyroid adenoma
 Iodide-induced: Drugs (amiodarone), Iodine
supplementation programme
 Extrathyroidal source of thyroid hormone: Factitious thyrotoxicosis, Struma ovarii
 TSH-induced: TSH-secreting pituitary adenoma, Choriocarcinoma and hydatidiform mole
 Follicular carcinoma ± metastases
: 👉🏻
Graves’ disease is an autoimmune thyroid
disorder which occurs typically in women aged
30–50 years. The most common manifestation is
thyrotoxicosis + Diffuse goitre.
Associations of Grave’s disease:  risk for other
autoimmune diseases like SLE, Pernicious anemia,
Type 1 DM & Addison Disease
Complications of Grave’s disease:
▪ Exophthalmos: Due to sympathetic overactivity
➔ wide, staring gaze and lid lag {typical signs}
▪ Pretibial myxedema {infiltrative dermopathy}:
presents as scaly thickening & induration of skin
– Life-threatening
complication of thyrotoxicosis. It’s a medical
emergency
Causes:
 It is precipitated by infection in a patient with
previously unrecognised or inadequately treated thyrotoxicosis.

 Also seen shortly after thyroidectomy or within


a few days of therapy
Clinical Features: Fever, agitation, delirium, tachycardia or atrial fibrillation and, in the older
patient, cardiac failure.
Management: Rehydrate + give propranolol, either orally (80 mg 4 times daily) or IV (1–5 mg 4 times daily).
▪ Sodium ipodate (500 mg per day orally) – This is a radiographic contrast medium that not only inhibits the
release of thyroid hormones but also  the conversion of T4 to T3. Dexamethasone (2 mg 4 times daily) and amiodarone
have similar effects.
▪ Oral carbimazole 40–60 mg daily – to inhibit the synthesis of new thyroid hormone.

Clinical findings + Laboratory findings will help in Dx


Laboratory findings:  Serum TSH.  Free T4.
Serum TSH is the most useful single screening test for hyperthyroidism, because it is  even at
subclinical stage.
Measurement of radioactive
iodine uptake by the thyroid
gland to determine the cause.
  uptake in the whole gland
➔ Graves’ disease
  uptake in a solitary nodule
➔ toxic adenoma
  uptake ➔ thyroiditis
👉🏻
----------------------------------------------------------------------------------------------------------------------------------------
2) Cushing Syndrome [21, 17, 13]
a. Iatrogenic Cushing's syndrome [06]
Ans. Cushing’s syndrome is caused by excessive activation of glucocorticoid receptors.
If it is due to pituitary origin, it is known as Cushing disease.
If it is due to adrenal origin, it is called Cushing syndrome
Etiology:
 It is most commonly iatrogenic, due to prolonged
administration of synthetic glucocorticoids such as
prednisolone.
 Endogenous Cushing’s syndrome is caused by chronic over-
production of cortisol by the adrenal glands, either as the
result of an adrenal tumour or because of ACTH by a
pituitary tumour or ectopic ACTH production by other
tumours. 👉🏻
Clinical features: The typical patient with Cushing's syndrome is a middle-aged plethoric woman with truncal obesity &
hypertension
 Disproportionate distribution of body fat, → Moon face,
Buffalo hump, Pot belly & lemon on stick appearance
 Thinning of extremities
 Skin manifestations – skin atrophy, easy bruisability &
purple striae in the trunk, breasts, and abdomen.
Hyperpigmentation seen in cases of ACTH (melanocyte-
stimulating effect)
 Bone resorption and osteoporosis due to protein depletion
 fractures & vertebral collapse. Low back pain is a
common presenting feature. resorption of bones 
hypercalciuria & renal calculi
 Weakening of muscles because of protein depletion
 Signs of adrenal androgen excess: Hirsutism, thinning of scalp hair, deepening of voice & clitoral
enlargement
 Hyperglycemia due to inhibition of peripheral utilization of glucose → Glucosuria & adrenal
diabetes
 Hypertension by the mineralocorticoid effects of glucocorticoids
 Immunosuppression resulting in susceptibility for infection
 Poor wound healing.
 Menstrual irregularities-oligomenorrhea, amenorrhea, etc.
 Neuropsychiatric symptoms – emotional liability, depression,
irritability, anxiety and panic attacks.
Investigations:
 Blood sugar and cortisol levels
 Urine analysis for 17-hydroxysteroids.
 Dexamethasone suppression test.
Medical Therapy
 Somatostatin analogues – Ex: Pasireotide – to inhibit ACTH secretion.
 Adrenal steroid inhibitors: Metyrapone, ketoconazole, etomidate.
 Glucocorticoid receptor antagonist: Mifepristone.
 Adrenolytic agents: Mitotane – This drug will cause adrenal cortical necrosis.
Surgery
▪ For Pituitary Adenoma – Trans-sphenoidal surgery with selective removal of the adenoma.
▪ Adrenal adenomas are removed via laparoscopy or a loin incision.
▪ Ectopic ACTH syndrome-localized tumors (e.g., bronchial carcinoid) should be removed.
▪ Unresectable malignancies may be treated by radiotherapy and chemotherapy.
▪ Medical therapy can be used for recurrences.
----------------------------------------------------------------------------------------------------------------------------------------
3) Addison's disease - etiology, C/F, Dx & Mx [20, 18, 12, 01]
a. Signs of Addison’s Disease [19]
b. Addisonian Crisis [14]
Ans.
Primary adrenal insufficiency (inability of the adrenals to produce hormones) is known as Addison's
disease.
Adrenal insufficiency can be acute or chronic – Acute adrenal insufficiency (acute adrenal crisis) is a
medical emergency

Genetic diseases: Congenital adrenal hyperplasia, Polyglandular syndromes


Idiopathic: Sporadic cases
Infections: Tuberculosis, HIV/AIDS, Histoplasmosis
Infiltrative diseases: Hemochromatosis, Sarcoidosis, Amyloidosis
Iatrogenic: Bilateral adrenalectomy, Postradiotherapy
Carcinoma: Metastatic carcinoma, Lymphoma
Adrenal hemorrhage: Waterhouse-Friedrichsen syndrome following meningococcal septicemia,
Anticoagulation, Trauma etc.
Drugs: Aminoglutethimide, metyrapone, ketoconazole
:
Mineralocorticoid Adrenal androgen
Glucocorticoid insufficiency ACTH excess
insufficiency insufficiency
▪ Weight loss, anorexia
Malaise, weakness
Nausea, vomiting Pigmentation of:
 Hypotension
▪ Diarrhoea or constipation Sun-exposed areas body hair and loss of
 Shock
▪ Postural hypotension; Pressure areas (e.g., libido, especially in
 Hyponatraemia
Shock elbows, knees) Palmar females
Hyperkalaemia
▪ Hypoglycaemia creases, knuckles, etc.
▪ Hyponatraemia (dilutional)
▪ Hypercalcaemia

Serum cortisol level < 3 µg/dl suggests adrenal insufficiency.


ACTH stimulation test (Synacthen test) – Cortisol levels fail to in primary adrenal insufficiency.
Serum ACTH level – It is low in ACTH deficiency and high in Addison's disease.
Serum electrolytes – hyponatremia and Hyperkalemia.
HIV test if risk factors for infection are present.
Plain X-ray abdomen may show adrenal calcification in tuberculosis.
Ultrasound abdomen is useful to assess the size of adrenals and also to detect any tumors.
CT or MRI of adrenals to look for size of adrenals and metastatic malignancy.
– Underlying cause should be treated
Patients should receive lifelong steroid replacement therapy – Cortisol 15 mg in the morning and 5
mg at 6 PM or prednisolone 5 mg in the morning and 2.5 mg in the evening.
Patient should carry a steroid card all the time which should give information regarding diagnosis,
steroid, dose and doctor.

: It can occur in the following situations:


 Serious infection or other major stress in a previously
undiagnosed patient with adrenal insufficiency.
 Skipping of steroid or failure to the dose in a patient with
known adrenal insufficiency during major illness or stress.
 Bilateral adrenal hemorrhage (Waterhouse-Friedrichsen
syndrome, anticoagulant therapy).
 Pituitary apoplexy.
 Rapid withdrawal of steroids in a patient who is taking
them for a long time.
Dehydration; Hypotension or shock.
▪ Nausea, vomiting and abdominal pain. Abdominal rigidity or rebound tenderness may be present
mimicking acute abdomen.
▪ Confusion or disorientation.
▪ Fever may be present due to underlying infection.
▪ There may be Hyperkalemia, hyponatremia, hypoglycemia, lymphocytosis and eosinophilia.
----------------------------------------------------------------------------------------------------------------------------------------
4) Diabetic Ketoacidosis – C/F, Complications, Dx & Mx [20, 17, 15, 04]
a. Neurological complications of DM [12]
b. Non-ketotic hyperosmolar coma in diabetes mellitus [10]
c. Diabetic Nephropathy [04]
d. Diabetes Mellitus – classify, pathogenesis, metabolic changes, Dx & Complications [03]
e. Juvenile DM [02]
Ans.
DKA is an acute
complication of DM
consisting of
Hyperglycemia, ketosis &
Acidemia

----------------------------------------------------------------------------------------------------------------------------------------
5) Myxedema – etiology, C/F & Mx [18, 12, 11]
a. Myxedema coma – C/F & Tt [21, 16, 08, 04]
b. Signs of Myxedema [19]
c. Pretibial myxedema [14]
Ans. Myxedema is the hypothyroidism in adults, characterized by generalized edematous appearance
Etiology: Autoimmune diseases (Hashimoto’s thyroiditis), Iodine deficiency & deficiency of TSH or TRH.
Signs and symptoms of myxedema: Typical feature of this disorder is an edematous appearance
throughout the body.
Swelling of the face; Bagginess under the eyes
Non-pitting type of edema because of accumulation of proteins with hyaluronic acid and chondroitin
sulfate.
Atherosclerosis: because of plasma level of cholesterol
Other general features of hypothyroidism
: This is a very rare presentation of hypothyroidism in which there is a level of
consciousness.
: Elderly patient who appears myxoedematous with body temperature as low
as 25°C & convulsions
: CSF pressure & protein content.
:
 Myxoedema coma is a medical emergency and treatment must begin before biochemical
confirmation of the diagnosis. The mortality rate is 50%.
 For Suspected cases  i.v. inj. of 20 μg liothyronine  if body temp. increase, then after 48-72
hours, switch to oral levothyroxine in a dose of 50 μg daily
 Hydrocortisone 100 mg IM 3 times daily (to cover secondary hypothyroidism)
 Other measures: slow rewarming, cautious use of i.v. fluids, broad-spectrum Abx & high-flow
oxygen.
This infiltrative dermopathy occurs in < 5% of patients with Graves’ disease
and has similar pathological features as occur in the orbit.
Raised pink-coloured or purplish plaques on the anterior aspect of the leg, extending on to the
dorsum of the foot.
The lesions may be itchy and the skin may have a ‘peau d’orange’ appearance with growth of
coarse hair; less commonly, the face and arms are affected.
Treatment is rarely required but in severe cases topical glucocorticoids may be helpful.
----------------------------------------------------------------------------------------------------------------------------------------
6) Hypothyroidism – etiology, C/F, complications, Inv., Dx & Mx [16, 13, 03]
Ans. Hypothyroidism is a clinical syndrome which occurs due to deficiency of thyroid hormones
:
Autoimmune Hashimoto’s thyroiditis, Graves’ disease.
Iatrogenic: Radioactive iodine ablation, Thyroidectomy, Drugs: Amiodarone, Lithium
Iodine deficiency
Infiltrative: Amyloidosis, Riedel’s thyroiditis, sarcoidosis etc
Congenital: Thyroid aplasia
Secondary hypothyroidism: TSH deficiency
Weight gain, fatigue, Extreme somnolence with sleeping up to 14-16 h per day,
General
cold intolerance, slurred speech, puffy face and loss of eyebrows
Dry, cold and pale skin, sweating, nonpitting edema(myxedema), carotenemia,
Skin
coarse hair & hair loss; xanthelasma;
Haematologic Anemia, macrocytosis
CVS ↓ in rate, FOC of the heart, cardiac output and blood volume
Hypoventilation, sleep apnoea, exertional dyspnea, pleural effusion;
RS
Frog-like husky voice.
Enlargement of the tongue, constipation (due to gut motility), ileus, taste
GIT
sensation, ascites.
Reproductive Oligomenorrhea, amenorrhea or menorrhagia,  fertility, risk of abortion,
system decreased libido, erectile dysfunction, delayed ejaculation.
Encephalopathy, myxedema coma, mental retardation in children, carpal tunnel
Neuropsychiatric syndrome, cerebellar ataxia, depression, psychosis, myotonia, delayed relaxation
of tendon reflexes.
Musculoskeletal Slow movement, myalgia, arthralgia, aches and stiffness

Serum T3, T4 is low and TSH elevated (>5).


Serum cholesterol, triglycerides, lactate dehydrogenase (LDH), creatinine kinase (CK) and AST may
be raised.
Serum sodium levels may be low.
Chest X-ray may show cardiomegaly.
ECG may show sinus bradycardia with low voltage complexes + ST segment & T wave abnormalities
The aim is to achieve a euthyroid status. TSH is the most sensitive indicator and
treatment should be aimed at normalizing TSH level
Levothyroxine (T4), with doses ranging from 50 to 200 µg/day. It is given once a day.
In elderly patients and those with coronary artery disease, the initial dose should be 12.5 to 25
µg/day and increased by 25 to 50 µg every 4 weeks to avoid precipitating angina and heart failure.
----------------------------------------------------------------------------------------------------------------------------------------
7) Hyperlipidaemias – classify, pathogenesis, C/F, inv. & Mx [03, 2000]
a. Gemfibrozil [02]
Ans. Frederickson’s
classification of
Hyperlipidaemias:

----------------------------------------------------------------------------------------------------------------------------------------
1. Hashimoto's Thyroiditis [22]
Ans.
Hashimoto’s thyroiditis is characterised by destructive lymphoid infiltration of the thyroid, ultimately
leading to a varying degree of fibrosis and thyroid enlargement.
Etiopathogenesis:
‘Patients with positive anti TPO Abs and a firm goitre who may or may not be hypothyroid.
There is an ↑ risk of thyroid lymphoma.
Clinical Features:
Incidence increases with age and affects women>men
Patient present with diffuse goitre, which is characteristically firm or rubbery in consistency.
Around 25% of patients are hypothyroid at presentation. In the remainder, serum T4 is normal &
TSH normal or , but these patients are at risk of developing overt hypothyroidism in future years.
Investigations: Anti TPO Abs in the serum.
Thyroid profile
In those under the age of 20 years, antinuclear factor (ANF) may also be positive.
Treatment: Levothyroxine therapy is indicated for hypothyroidism and also to shrink an associated
goitre.
----------------------------------------------------------------------------------------------------------------------------------------
2. Acromegaly – C/F & Mx [21, 17, 13, 01, 97]
Ans: Acromegaly is the clinical syndrome that results from excessive secretion of growth hormone
Etiology:
 Somatotroph (growth hormone-
secreting) adenoma of the anterior
pituitary – MCC
 Excess secretion of GHRH by
hypothalamic tumors, ectopic GHRH
secretion by nonendocrine tumors such
as carcinoid tumors or small-cell lung
cancers etc.
Management
 Surgical: Trans-sphenoidal surgery is
usually the first line of treatment and
may result in cure of GH excess,
especially in patients with
microadenomas.
 Radiotherapy: External radiotherapy is usually employed as second-line treatment if acromegaly
persists after surgery, to stop tumour growth and lower GH levels.
 Medical:
Somatostatin analogues (such as octreotide, Lanreotide or Pasireotide)
Dopamine agonists
Pegvisomant is a peptide GH receptor antagonist administered by daily self-injection
----------------------------------------------------------------------------------------------------------------------------------------
3. Hypoglycaemia – causes, C/F & treatment [20, 15, 12]
Ans: Hypoglycemia is low plasma glucose concentration <70 mg/dL.
Clinical Features:
➢ Autonomic symptoms (due to acute activation of ANS):
Sweating, Trembling, Pounding heart, Hunger, Anxiety
➢ Neuroglycopenic symptoms (due to glucose deprivation to the
brain): Confusion, Drowsiness, Speech difficulty, Inability to
concentrate, Incoordination, Focal neurological deficits
➢ Non-specific: Nausea, Tiredness, Headache
Measures to Prevent Hypoglycemia:
Do not skip meals after taking sulphonylurea or insulin.
Use the correct dose of insulin and oral antidiabetic agents as prescribed.
Avoid unaccustomed intense exercise especially on empty stomach.
Take light snacks in between major meals and also at bedtime.
Monitor blood sugar frequently.
Carry supply of fast-acting carbohydrate (sweets, sugar) and a glucagon injection while going for long travel.
Management of Hypoglycemia
If the patient is conscious and able to swallow: glucose (50 gm) or any other fast acting source of
carbohydrate (sweets, honey, etc.) can be given orally.
If the patient is in altered sensorium and unable to swallow: IV glucose (50 ml of 50% dextrose). If
IV access is a problem, Inj glucagon (1 mg IM) can be given.
If hypoglycemia has occurred after the use of a long acting insulin or drug: above treatment should
be followed by an infusion of 10% dextrose for few hours, to prevent recurrence of hypoglycemia.
If the patient fails to regain consciousness after blood glucose is restored to normal: cerebral
edema should be suspected. It treated with mannitol & high-dose oxygen.
----------------------------------------------------------------------------------------------------------------------------------------
4. Oral Hypoglycaemic Agents (OHA) [19]
a. Oral anti-diabetic drugs [11]
b. Metformin [08]
c. Biguanides [05]
Ans.
➔ Metformin
MOA: It activates the enzyme AMP dependent
protein kinase (AMPK) which results in -
✓ hepatic gluconeogenesis (major action)
✓ peripheral utilization of glucose in
skeletal muscle and fat. This leads to
increase in glycogen storage in the skeletal
muscle increased fatty acid oxidation and
decreased lipogenesis.
✓ Inhibition of alimentary absorption of
glucose
Pharmacokinetics: taken orally, well absorbed
through GIT & is excreted mostly unchanged in
urine.
Uses: in patients with type-2 DM either alone or in combination with other anti-diabetic agents.
A/E
• Metallic taste, anorexia, Nausea, vomiting, diarrhoea & Loss of Wt.
• skin rashes
• lactic acidosis is the most serious complication but is rare
• Vit B12 deficiency on prolonged use (due to malabsorption)
----------------------------------------------------------------------------------------------------------------------------------------
5. Diabetes insipidus [19, 81]
Ans: Diabetes insipidus (DI) results from a deficiency of vasopressin (ADH) due to a hypothalamic
pituitary disorder (central DI) or from resistance of the kidneys to vasopressin (nephrogenic DI).
Etiology:
Clinical Features ▪ Idiopathic
▪ Structural hypothalamic or high stalk
Polyuria and polydipsia lesion
Patient may pass 5-20 litres or ▪ Familial disease (DIDM0AD syndrome)
more of dilute urine in 24 hours. ▪ Neurosurgery or trauma, Cancer (primary
Central
Polyuria leads to excess polydipsia. brain tumors, metastases)
DI may lead to dangerous ▪ Hypoxic encephalopathy
hypovolemia if the patient does ▪ Infiltrative disorders (histiocytosis,
sarcoidosis)
not have access to water or there is
▪ Post-supraventricular tachycardia
impaired thirst mechanism.
▪ Genetic defects (vasopressin-2 receptor
Investigations mutation, aquaporin-2 mutation,
 Measurement of 24-hour urine cystinosis)
Nephrogenic ▪ Metabolic abnormality (hypokalemia,
volume and creatinine excretion.
hypercalcemia)
 Vasopressin challenge test:
▪ Drugs (lithium)
Desmopressin is given in an initial ▪ Poisoning (heavy metals)
dose of 5-10 µg intranasally (or 1 µg ▪ Polycystic kidney disease.
SC or IV). Urine volume is measured
for 12 h before and 12 h after administration. Patients with central DI notice a distinct reduction in
thirst and polyuria.
 Water deprivation test: This is done to confirm the diagnosis of DI and differentiate central from
nephrogenic causes.
 MRI of the pituitary and hypothalamus to look for mass lesions.
Management
✓ Treatment of central DI is with desmopressin (DDAVP). Desmopressin is usually administered as a
metered dose spray into the nose. In emergencies, desmopressin is given by IM injection.
The main side effect of desmopressin is excess water retention and hyponatremia.
✓ Nephrogenic DI is treated by thiazide diuretics, amiloride, and NSAIDs (e.g. indomethacin).
---------------------------------------------------------------------------------------------------------------------------------------
6. Hyperthyroidism – C/F, inv. & Mx [19, 90]
Ans: Hyperthyroidism is characterized by increased synthesis and secretion of thyroid hormones which
leads to the hyper-metabolic state.

Weight loss despite normal or increased appetite, Heat intolerance, Fatigue, Goiter
General
with bruit, Single or multiple nodules may be present in the thyroid
GIT Diarrhea, hyperdefecation, Anorexia, Vomiting
Systolic hypertension/increased pulse pressure, Palpitations, Sinus tachycardia,
CVS
Atrial fibrillation, High output cardiac failure, Angina
Resp. System Exacerbation of asthma, Dyspnea on exertion
Hematological Lymphadenopathy, Normochromic normocytic anemia (due to plasma volume)
Nervous Tremor, Muscle weakness, Periodic paralysis, Hyper-reflexia, Ill-sustained clonus,
system Proximal myopathy, Bulbar myopathy
Increased sweating, Pruritus, Hair thinning, alopecia, Palmar erythema, Pretibial
Skin myxoedema, Onycholysis, Hyperpigmentation, Vitiligo can occur in association with
autoimmune thyroid disorders
Genitourinary Amenorrhea/oligomenorrhea, Infertility, spontaneous abortion, Loss of libido,
system impotence, Gynecomastia, Urinary frequency and nocturia
Stare & lid lag, Gritty feeling or pain in the eyes, Excessive lacrimation, Diplopia,
Eyes Loss of acuity, Exophthalmos, Periorbital and conjunctiva! Edema, Corneal
ulceration, Ophthalmoplegia, Papilledema
Bone Osteoporosis (fracture, loss of height)
Psychiatric Anxiety, irritability, emotional liability, psychosis

Serum T3 and T4
Serum TSH is ↓ in primary thyrotoxicosis and ↑ in TSH induced thyrotoxicosis.
TSH receptor antibodies (TRAb) are ↑ in Graves' disease.
Anti-TPO Ab titres are ↑ in Graves' disease, and are ↓ or absent in toxic MNG and toxic adenoma.
Isotope scanning: ↑ in Graves' disease & ↓ in thyroiditis.
Thyroid ultrasound can identify nodules and distinguish solid from cystic lesions.
Ultrasound-guided FNAC
Refer 1st LQ
---------------------------------------------------------------------------------------------------------------------------------------
7. Human Insulins [18]
a. Insulin analogues [21]
b. Insulin therapy [14]
c. Newer Insulin delivery devices [08]
Ans:
They are produced by
recombinant DNA technology using E. coli or yeast.
They have the same amino acid sequence as
endogenous insulin.
They are least immunogenic; insulin resistance
and lipodystrophy at the site of injection are rare, e.g., human regular insulin, human NPH insulin,
etc.
Purified human insulins are the commonly used insulin preparations
----------------------------------------------------------------------------------------------------------------------------------------
8. SIADH [17, 03]
Ans: In SIADH, increased (inappropriate) ADH release occurs without any physiologic stimulation.
▪ Hypovolemia and hyperosmolality are physiological stimulations for ADH secretion
▪ Normal regulation of ADH release occurs from both CNS and chest via baroreceptors and neural
input. Hence, disorders affecting CNS and lungs commonly produce SIADH

Inappropriate ADH secretion water retention  hyponatremia


Nausea and malaise are the earliest findings
Headache, lethargy, obtundation, seizures, coma, and
respiratory arrest
Fatigue, nausea, dizziness, gait disturbances,
forgetfulness, confusion, lethargy, and muscle cramps

Hyponatremia
Low plasma osmolality <270 mmol/kg.
Urine osmolality > 150 mmol/kg. Normally urine should be maximally dilute in the presence of low
serum osmolality, but is typically> 150 in SIADH, i.e., inappropriately concentrated due to ADH action.
Urine sodium concentration >30 mmol/l.
Normal renal function tests, uric acid.
Exclusion of other causes of hyponatremia.
Appropriate clinical context.

Severe symptomatic hyponatremia should be corrected using hypertonic saline.


Fluid restriction to 600-1000 ml/day.
Treatment of the cause of SIADH (e.g., withdrawal of a drug causing SIADH).
Demeclocycline (600-900 mg/day) may enhance water excretion, by interfering with collecting duct
responsiveness to ADH.
Oral urea therapy (30-45 g/day) can provide a solute load to promote water excretion.
ADH receptor antagonists: conivaptan & tolvaptan. They promote the excretion of free water
----------------------------------------------------------------------------------------------------------------------------------------
9. Gynaecomastia and its causes [16] ➔ refer pg. no. 10 of Surgery-paper II
----------------------------------------------------------------------------------------------------------------------------------------
10. Pheochromocytoma [15, 03, 2000]
a. Screening urinary test for pheochromocytoma [19]
Ans: Pheochromocytoma is catecholamine-secreting tumor that arises from chromaffin cells of the
adrenal medulla and the sympathetic ganglia. The catecholamines secreted include norepinephrine,
epinephrine, and dopamine.
Origin: Chromaffin cells
Rule of 10s: 10% - Malignant; 10% - Bilateral; 10% - Extra adrenal (in sympathetic ganglia); 10% - familial
Etiology: Sporadic or familial (25%). The tumor cells synthesize and release catecholamines.
Genetic alterations: Some familial cases
are a/w MEN-2A, and MEN-2B, caused by
germline mutations of RET.
Clinical Picture: HTN with paroxysmal
episodes, associated with tachycardia,
palpitations, headache, sweating &
tremors.
Complications: cardiomyopathy with congestive heart failure, myocardial infarction & ventricular
fibrillation.
Investigations:
  Plasma catecholamines (epinephrine, norepinephrine and dopamine)
 plasma levels of free metanephrine – more sensitive
 ↑ urinary excretion of free catecholamines & their metabolites (vanillylmandelic acid, &
metanephrins);
 Biopsy.
 CT or MRI of the abdomen can localize the tumor
 MIBG scan – Metaiodobenzylguanidine (MIBG) resembles norepinephrine and is taken up by
adrenergic tissue. MIBG scan can detect tumors not detected by CT or MRI.
Metastasis (Malignant pheochromocytoma): Regional lymph nodes; Liver, lung & bone
Management:
 Surgical excision of the tumor is the treatment of choice. Preoperative preparation is done with α-
blocker phenoxybenzamine or labetalol
 If excision is not possible, medical therapy with α & β blocking drugs (phenoxybenzamine and
propranolol, or labetalol) is necessary.
- β-blockers should not be given alone, as unopposed α action will lead to hypertensive crisis.
----------------------------------------------------------------------------------------------------------------------------------------
11. Sheehan’s syndrome [15, 96]
Ans: Hypopituitarism due to infarction of the pituitary gland after PPH is called Sheehan's syndrome.
The pituitary gland is physiologically enlarged in pregnancy and is therefore very sensitive to the
decreased blood flow caused by massive hemorrhage and hypovolemic shock.
Clinical Features
Severe hypopituitarism manifests during the first days or weeks after delivery
Patients often have a history of severe PPH causing hypotension and requiring blood transfusion.
Failure to lactate (due to prolactin deficiency).
Many women also report amenorrhea or oligomenorrhea after delivery (FSH & LH deficiency).
Other features include fatigue, anorexia, weight loss (due to decreased ACTH), and features of
hypothyroidism (due to decreased THS).
Investigations
There is deficiency of all the hormones, i.e., GH, prolactin, gonadotropins, TSH and ACTH.
CT scan or MRI shows a small pituitary within a sella of normal size, sometimes read as an "empty
sella".
Treatment – Treatment is same as that for hypopituitarism – replace every pituitary hormone.
✓ ACTH deficiency is treated by giving hydrocortisone or other glucocorticoid.
✓ TSH deficiency is treated with L-thyroxine.
✓ In men with gonadotropin deficiency, testosterone replacement is indicated when fertility is not
desired. If fertility is desired, they are treated with gonadotropins.
✓ In women with gonadotropin deficiency, estrogen and progestin replacement is enough if fertility is
not desired. If fertility is desired, they should be treated with gonadotropin or pulsatile GnRH
therapy to induce ovulation.
✓ Growth hormone is replaced with recombinant human growth hormone.
----------------------------------------------------------------------------------------------------------------------------------------
12. Hyperparathyroidism {incl. C/F} [13, 09, 96]
a. Secondary hyperparathyroidism [22]
Ans: Hyperfunctioning of parathyroid gland with elevated PTH levels.
Types:
1) Primary Hyperparathyroidism
2) Secondary Hyperparathyroidism
: “An autonomous overproduction of PTH is seen.” MC cause of
asymptomatic hypercalcemia.
 : Adenoma (MC); Primary hyperplasia; Carcinoma
 : Elderly Female
It can be Sporadic (MC) or Familial
Genetic alterations with sporadic adenomas:
1) Cyclin D1 gene inversions leading to overexpression of cyclin D1
2) Mutations involving MEN1 tumor suppressor gene
Familial parathyroid adenomas; They are seen with MEN-1 & MEN-2A, caused by germline
mutations of MEN1 & RET.
Clinical Presentations: Asymptomatic or can exhibit the following symptomatic primary
hyperparathyroidism:
 Bone: Osteoporosis & osteitis fibrosa cystica leading to fractures.
 GIT: Constipation, nausea, peptic ulcers, & pancreatitis
 Renal: Nephrolithiasis & chronic renal insufficiency
 CNS: Depression, lethargy, & seizures
 Neuromuscular: Weakness & fatigue
 Cardiac: Aortic or mitral valve calcifications
Inv.: Elevated PTH levels, Hypercalcemia, hypophosphatemia, increased urinary excretion of both
calcium & phosphate.
: “Compensatory overactivity of the parathyroid glands due to
chronic hypocalcemia.”
➢ Causes: Renal failure (MC), inadequate intake of calcium, Vit. D deficiency, & steatorrhea.
➢ Clinical Presentations:
1) Features of chronic renal failure
2) Milder skeletal abnormalities
3) Ischemic damage to tissues (calciphylaxis)
Treat the cause {Ex: surgical excision of the hormone secreting tissue}.
 Orthopedic treatment – directed towards adequate protection of the softened bones from all
deforming stress and strain.
 Urology treatment – directed towards the removal of calculi and maintenance of renal functions.
---------------------------------------------------------------------------------------------------------------------------------------
13. Hirsutism [11, 07, 02]
Ans.
Hirsutism is the excessive growth of thick or dark hair in women in locations that are more typical of
male hair growth patterns (e.g., moustache, beard, central chest, shoulders, lower abdomen, back, inner thigh).
– it occurs due to androgen excess seen in –
Polycystic ovary syndrome (PCOS); Congenital adrenal hyperplasia; Ovarian and adrenal tumors
Acromegaly; ACTH-induced Cushing's disease
Drugs (minoxidil, anabolic steroids, diazoxide)

Increase in hair is seen on the chin, upper lip, abdomen, and chest.
Androgen excess also increases sebaceous gland activity, producing acne.
Menstrual irregularities, anovulation, and amenorrhea are common.
Defeminization (decrease in breast size, loss of feminine adipose tissue) and virilization (frontal
balding, muscularity, clitoromegaly, and deepening of the voice) occur if androgen excess is severe.
Hypertension is seen in Cushing's syndrome, adrenal 11-hydroxy lase deficiency, or cortisol
resistance syndrome.

Serum testosterone level is elevated.


17-hydroxyprogesterone level is elevated in congenital adrenal hyperplasia.
Ultrasound abdomen to assess adrenals and ovaries.
Adrenal CT.
:
Laparoscopic bilateral adrenalectomy - For Girls with congenital adrenal hyperplasia
Antiandrogen therapy – spironolactone, cyproterone acetate, finasteride, and flutamide
Local treatment – shaving, depilation, waxing, electrolysis, or bleaching.
▪ Eflornithine topical cream retards hair growth when applied twice daily
▪ Laser therapy is an effective treatment for facial hirsutism.
▪ Alopecia may be treated with minoxidil 2% solution applied twice daily to a dry scalp.
---------------------------------------------------------------------------------------------------------------------------------------
14. Azoospermia [10, 07] ➔ refer Gynae Notes pg. no. 36
----------------------------------------------------------------------------------------------------------------------------------------
15. Free T4 & TSH [10]
a. TSH (Thyroid stimulating Hormone) [11, 09]
Ans. The free hormones are the really active molecules
The free T4 constitutes only 0.03% of the total T4, whereas free T3 forms 0.3% of total T3.
Variations in binding proteins do not affect the free hormone levels & therefore more reliable in
diagnosing true hyper and hypofunction of thyroid
Serum TSH is the most useful single screening test for hyperthyroidism, because it is decreased even at
subclinical stage
Thyroid-stimulating hormone (TSH) secreted by anterior pituitary
TSH influences every stage of formation and release of thyroid hormones
In primary hypothyroidism, TSH level is elevated due to lack of feedback
But in secondary hypothyroidism, TSH, T3 and T4 levels are low; this could point to a pituitary or
hypothalamic cause
Hyperthyroidism due to pituitary cause is indicated by high TSH, T3 and T4 levels
----------------------------------------------------------------------------------------------------------------------------------------
16. Insulin resistance [09]
Ans.
Insulin Resistance: It is a state in which patient requires > 200 U of insulin per day.
Etiology: Idiopathic; Acute insulin resistance develops during stressful conditions like trauma,
infection, surgery, psychological stress, etc.
Pathogenesis: Resistance to the action of insulin in the liver and muscle leads to overproduction &
underutilization of glucose  Hyperglycemia Type 2 diabetes
Central obesity (esp. intraabdominal fat)  FFA  compete with glucose to be utilized by
peripheral tissues insulin resistance
Adipose tissue also releases many hormones (e.g., cortisol, adipokines) which may decrease the
sensitivity of insulin receptors
Clinical Presentation: Patient presents with central (visceral) obesity, hypertension and
dyslipidaemia (LDL cholesterol and triglycerides, low HDL cholesterol).
Coexistence of this cluster of conditions is called 'insulin resistance syndrome' or 'metabolic
syndrome'.
Metabolic syndrome predisposes to cardiovascular diseases.
Treatment:
Exercise allows noninsulin-dependent glucose uptake by muscles.
Reduction of obesity is the main therapeutic goal
Metformin is used to treat insulin resistance
Cessation of smoking
----------------------------------------------------------------------------------------------------------------------------------------
17. Acarbose [07]
Ans. ( voglibose) These drugs should be given just before food
▪ Inhibit α-Glucosidase in brush border of the small intestine ➔ reduce intestinal absorption
of carbohydrates ➔ reduce postprandial hyperglycaemia.
▪ used in obese patients with type-2 DM.
▪ are mainly on GI tract: flatulence, fullness and diarrhoea
----------------------------------------------------------------------------------------------------------------------------------------
18. Bromocriptine [05]
Ans. It is a relatively selective dopamine D2 agonist with prominent action on pituitary lactotropes
(inhibits pro-lactin release), in striatum (antiparkinsonian) and in CTZ (emetic)
Bromocriptine is infrequently used now; has been largely superseded by newer D2 agonists.
Conditions in which it can be used are: Hyperprolactinemia, Acromegaly, Parkinsonism & Diabetes
mellitus
Side effects:
Early: Nausea, vomiting, constipation, nasal block-age & Postural hypotension
Late: Behavioural alterations, mental confusion, hallucinations, psychosis, Abnormal movements,
livedo reticularis.
----------------------------------------------------------------------------------------------------------------------------------------
19. Cretinism & it’s prevention [05, 04] ➔ Pg. 71 in Paeds Notes
----------------------------------------------------------------------------------------------------------------------------------------
20. Carbimazole [04]
Ans. Carbimazole is a Thioamide – anti-thyroid drug.
MOA & Uses:
1. They inhibit thyroid peroxidase enzyme, which converts iodide to iodine.
2. They inhibit iodination of tyrosine residues in thyroglobulin
3. They inhibit coupling of iodotyrosines (MIT and DIT).
4. Carbimazole is More potent than Propylthiouracil.
5. Carbimazole/ methimazole is preferred for long-term treatment as it is long acting and has fewer
adverse effects than propylthiouracil
Pharmacokinetics: Well absorbed orally; converted to methimazole after absorption. widely
distributed but get accumulated in thyroid gland. Excreted in urine.
Adverse Effects:
➢ Skin rashes are most common.
➢ Others include: joint pain, fever, hepatitis, nephritis, etc.
➢ A dangerous but rare adverse effect is agranulocytosis. The drugs should be stopped at the first
sign of agranulocytosis, i.e., sore throat and/or fever.
➢ Hypothyroidism may occur but it is reversible
----------------------------------------------------------------------------------------------------------------------------------------
21. Glucose Tolerance Test [04]
Ans. should be done after an overnight fast, using a glucose load containing 75 gm of anhydrous
glucose dissolved in water
2-hour post load glucose levels of 200 mg/dL or greater establish the diagnosis of diabetes.
It is commonly done in the diagnosis of gestational diabetes mellitus.
----------------------------------------------------------------------------------------------------------------------------------------
22. Lactation Failure [03, 95] ➔ refer Obs Pg. No. 36
----------------------------------------------------------------------------------------------------------------------------------------
23. Homocystinuria [02]
Ans. Homocystinuria is an autosomal recessive disorder caused by loss-of-function mutations in the
CBS gene, which encodes cystathionine β-synthase.
The enzyme deficiency causes accumulation of homocysteine and methionine in the blood.
Clinical Features:
Eyes: ectopia lentis – displacement of the lens
CNS: intellectual disability, delayed developmental milestones, seizures, psychiatric disturbances
Skeleton – resembling Marfan’s syndrome, and also with generalised osteoporosis
Vascular system – thrombotic lesions of arteries and veins &
Skin – hypopigmentation.
Treatment:
Dietary, involving a methionine-restricted, cystine-supplemented diet
Large doses of pyridoxine.
----------------------------------------------------------------------------------------------------------------------------------------
24. Cryptorchidism [01]
Ans. Undescended testis occurs due to arrest of descent of the testis in some part along its pathway to
the scrotum.
Bilateral undescended testis is called cryptorchidism (means hidden testis).
Complications of undescended testis: Sterility; Torsion testis; indirect inguinal hernia; Epididymo-
orchitis; Testicular atrophy; Malignant transformation to seminoma
----------------------------------------------------------------------------------------------------------------------------------------

1) Nelson's Syndrome [22]


Ans.
Nelson’s Syndrome is ↑pigmentation because of high levels of ACTH associated with an enlarging
pituitary tumor post bilateral adrenalectomy.
It occurs in about 20% of cases after b/l adrenalectomy for Cushing’s disease.
Treatment: Pituitary surgery or radiotherapy
----------------------------------------------------------------------------------------------------------------------------------------
2) Sildenafil uses [22]
Ans:
• To treat erectile dysfunction (impotence) in men.
• To treat pulmonary arterial hypertension and improve exercise capacity in men and women.
----------------------------------------------------------------------------------------------------------------------------------------
3) Mention acute complications of DM [18]
Ans:
Diabetic ketoacidosis: Seen predominantly with type 1 diabetes ➔ Fatigue, nausea & vomiting,
severe abdominal pain, fruity odor and deep, labored breathing (Kussmaul breathing). Altered
consciousness & coma may be seen.
Hyperosmolar hyperosmotic syndrome: Seen with type 2 diabetics ➔ Severe dehydration &
impairment of mental status.
Hypoglycemia: if the patient took excess insulin / didn’t take sufficient die after insulin injection.
Lactic Acidosis
----------------------------------------------------------------------------------------------------------------------------------------
4) Serum Cortisol [07]
Ans. The plasma cortisol level is determined by ELISA, radioimmunoassay (RIA), or chemiluminescent
immunoassay (CLIA)
The normal range is 5–25 mg/dL at 9 am and 2–5 mg/dL at 10 pm.
A loss of diurnal rhythm may be an early indication of disease – Ex: levels in Cushing’s Syndrome
----------------------------------------------------------------------------------------------------------------------------------------
5) Normal human semen [06]
Ans.

----------------------------------------------------------------------------------------------------------------------------------------
6) Growth Hormone therapy – indications [03]
Ans.
1. (Hypo & Hyper) kalaemia – causes & treatment. Draw a diagram of potassium handling by nephron
[05, 02]
a. Name 4 causes of Hypokalaemia [May-22]
b. ECG in hypokalemia [21]
c. Treatment of Hyperkalemia [19, 17]
Ans.

Hypokalaemia is defined as existing when serum K+ falls below 3.5 mmol/L.


Causes of HYPOkalaemia
- Dietary deficiency
1. Reduced intake
- Potassium-free intravenous fluids
▪ Alkalosis
2. Redistribution ▪ Insulin & Catecholamines
into cells (flux of ▪ β- agonists
K+ into cells)
▪ Hypokalaemic periodic paralysis
↑ Activation of mineralocorticoid receptor – seen in - Conn’s syndrome
Cushing’s syndrome, Glucocorticoid excess & Carbenoxolone/liquorice
– iddle’s syndrome, itelman’s syndrome, artter’s
3. Increased urinary syndrome
excretion Renal tubular acidosis
Acetazolamide
Diuresis: Loop diuretics, Thiazides, Recovery from renal obstruction etc.
 Upper GIT – a/w metabolic alkalosis – Vomiting, Nasogastric aspiration
4. Increased GI loss  Lower GIT – a/w metabolic acidosis – Diarrhoea, Laxative abuse, Villous
adenoma, Bowel obstruction/fistula & Ureterosigmoidostomy
Clinical Features of HYPOkalaemia
▪ Muscular weakness and tiredness.
▪ Ventricular ectopic beats (arrhythmias)
▪ Electrocardiogram (ECG) changes:
▪ Functional bowel obstruction – occur due to paralytic ileus.
▪ Renal tubular damage – in chronic cases
Management
First determine the cause & then correct Hypokalaemia with Potassium chloride tablets or i.v.
Potassium chloride
If Hypokalemia is a/w Acidosis – then use Potassium bicarbonate
The rate of administration depends on the severity of hypokalaemia
Correct hypomagnesemia {since low cell magnesium can promote tubular potassium secretion into
urine}
, such as amiloride can be used especially when renal loss of potassium
is the underlying cause.
is defined as existing when serum K+ is > 5 mmol/L.
Causes of HYPERkalaemia
 Haemolysis during venepuncture or in vitro – Release of intracellular K+
1) Artefactual during sample collection
 Thrombocytosis/leucocytosis
- Dietary Potassium
2) Increased intake
- Potassium-containing intravenous fluids
▪ Acidosis
3) Redistribution ▪ Insulin deficiency & severe hyperglycaemia
From cells (flux of ▪ β – blockers
K+ into PLASMA) ▪ Hyperkalaemic periodic paralysis
▪ Rhabdomyolysis, Severe haemolysis & Tumour lysis syndrome
↓GFR – Acute kidney injury & chronic kidney disease
↓ Mineralocorticoid receptor activation – seen in:
Addison’s disease & Congenital adrenal hyperplasia
ACE inhibitors & ARBs {↓ Aldosterone levels}
Calcineurin inhibitors, Spironolactone & Eplerenone {block the
mineralocorticoid receptor}
4) Reduced urinary Heparin {inhibits aldosterone production}
excretion
↓ Renin production – can occur due to: NSAIDs & β-blockers
Tubulointerstitial diseases – Interstitial nephritis, Diabetic nephropathy &
Obstructive uropathy
Others:
▪ Amiloride – Blocks K+ exchange in distal tubule
▪ Gordon’s syndrome – ↓ K+ secretion in the renal tubules
Clinical features
⇨ Mild to moderate hyperkalaemia (< 6.5 mmol/L) is usually
asymptomatic
⇨ In severe cases muscular weakness & cardiac arrest occurs.
⇨ Electrocardiogram (ECG) changes:
Management
 Treatment of hyperkalaemia depends on its
severity and the rate of development
 Mild to moderate hyperkalaemia (< 6.5 mmol/L)
can be treated with a reduction of potassium
intake and correction of predisposing factors
 Treatment of severe hyperkalaemia

----------------------------------------------------------------------------------------------------------------------------------------

1) Hypocalcemia [16, 15, 06, 04, 03]


a. Tetany – C/F & Mx [11, 06, 04, 02, 97]
Ans.
: Hypocalcemia is an abnormal reduction in serum
ionized calcium concentration (<8.8 mg/dL).
Tetany is seen when calcium levels fall below 4.3 mg/dL.
Tetany is characterized by both sensory and motor features.
 Sensory symptoms: - circumoral numbness, paresthesias of the
hands and feet
 Motor symptoms are stiffness, myalgia, muscle spasms and
cramps. Hand muscle spasm leads to adduction of the thumb,
flexion of the MCP joints and wrists, and extension of the fingers.
Spasm of the respiratory muscles
and of the glottis can cause cyanosis.
 Autonomic manifestations include
diaphoresis, bronchospasm, and
biliary colic.
 Chvostek’s sign – tapping over facial
nerve causes facial contraction.
 Trousseau’s sign – inflation of BP cuff for 3 minutes causes carpopedal spasm

----------------------------------------------------------------------------------------------------------------------------------------
2) Hypercalcemia – Causes & Treatment [14, 03]
Ans.
Definition: blood calcium level is more than 11 mg/dL.

----------------------------------------------------------------------------------------------------------------------------------------
3) Lithium [04] ➔ refer Paper - 2 pg. no. 127
----------------------------------------------------------------------------------------------------------------------------------------
4) Hyponatremia [01]
Ans.

Treatment of hyponatremia depends on cause:


Water restriction, increased salt intake, furosemide and anti-ADH drugs
Rapid correction may increase mortality by neurological complications
----------------------------------------------------------------------------------------------------------------------------------------

1. Genetic Counselling [21] ➔ refer Paper-2 Pg.NO. 58


----------------------------------------------------------------------------------------------------------------------------------------
2. Hyperglycemic hyperosmolar state [15]
Ans.

----------------------------------------------------------------------------------------------------------------------------------------
3. Forced alkaline diuresis [09]
Ans.
Forced alkaline diuresis (using IV bicarbonate infusion and furosemide) is used to maintain a good flow
of less acidic fluid within the renal tubules.

----------------------------------------------------------------------------------------------------------------------------------------
4. Serum Magnesium [06]
Ans.
Normal serum level Mg+2 is 1.8-2.2 mg/dL

----------------------------------------------------------------------------------------------------------------------------------------
5. Fluorosis [05]
Ans. Fluorosis occurs due to excessive amounts of fluorine in drinking water
The toxic manifestation of fluorosis:
1) Dental fluorosis
▪ The teeth lose their shiny appearance and chalk-white patches develop on them.
▪ Later the white patches become yellow and sometimes brown or black.
▪ In severe cases, loss of enamel gives the teeth a corroded appearance.
▪ Mottling is best seen on the incisors of the upper jaw.
2) Skeletal fluorosis – crippling can ensue. It leads to permanent disability.
3) Genu valgum and osteoporosis of the lower limbs
Intervention
1. Changing the drinking water source with a low fluoride content (0.5 to 0.8 mg/L).
2. Chemical treatment: Nalgonda technique for defluoridation of water. It involves the addition of 2
chemicals (viz. lime and alum) in sequence followed by flocculation, sedimentation and filtration
3. Other measures: Fluoride supplements (Ex: fluoride toothpaste) should not be prescribed for children
who drink fluoridated water.
1. Sjogren's syndrome [19]
Ans.
Sjögren syndrome is an autoimmune destruction of lacrimal and salivary glands
Age & Sex: women in their late middle age
Clinical manifestations: Triad of dry eyes (keratoconjunctivitis sicca), dry mouth (xerostomia) &
recurrent dental caries in an older woman
• a/w SLE, PSS (progressive systemic sclerosis, scleroderma), polymyositis, or Hashimoto thyroiditis
• Sicca syndrome is a variant characterized by xerostomia and keratoconjunctivitis alone.
Laboratory findings:
 Polyclonal hypergammaglobulinemia
 ANAs, including the highly specific anti-SS-8
 Schirmer test-to detect lack of lacrimal secretion
 Autoantibody estimation – Salivary duct antibody, sialography; estimation of salivary flow;
 Slit lamp test of eyes
Treatment: It is conservative. Artificial tears; artificial saliva; frequent drinking of water; treat the
cause
----------------------------------------------------------------------------------------------------------------------------------------
2. Tuning fork tests [07]
Ans.
Type of TF test Principle Normal CHL SNHL
It compares patient’s AC with AC > BC BC > AC AC > BC
Rinne test
BC (Rinne +) (Rinne -) (Rinne +)
BC of both ears is compared Lateralized to Lateralized to
Weber Test Not lateralized
simultaneously. affected ear better ear
It compares the duration of
Absolute Bone patient’s BC with that of the Same as Same as
Reduced
Conduction Test examiner. examiner’s examiner’s
EAC is occluded.
BC of patient is compared
with that of the normal Same as Reduced
Schwabach’s Test Lengthened
hearing examiner but EAC is examiner’s (Shortened)
not occluded
It tests the effect of occlusion Negative
Bing Test Positive (No change in Positive
of EAC on BC hearing hearing)
It tests the effect of ↑ air Negative
Gelle’s Test pressure in EAC (via Siegel’s Positive (Ossicular chain is Positive
speculum) on the BC hearing fixed)
----------------------------------------------------------------------------------------------------------------------------------------
3. Hypertensive Retinopathy [05]
a. Ocular retinal changes in hypertension [06]
Ans.
Hypertensive retinopathy refers to fundus changes occurring in patients suffering from systemic HTN.
Pathogenesis: 3 factors -
1) Vasoconstriction of retinal & choroidal vessels ➔ choroidal & RPE ischaemia→ hypertensive
choroidopathy
 Vasoconstriction of peripapillary choroid → optic nerve head ischaemia → hypertensive optic
neuropathy
2) Arteriosclerotic changes in the vessels
3) ↑ vascular permeability results from
hypoxia & is responsible for
haemorrhages, exudates, macular
oedema etc.
Clinical Types
1) Simple hypertension without sclerosis

2) Hypertension with involutionary (senile) sclerosis

3) Chronic hypertension with compensatory arteriolar sclerosis

4) Malignant (acute) hypertensive retinopathy

Management
 Mild cases require BP control only.
 Moderate cases (with retinal haemorrhages, microaneurysms & cotton-wool spots): BP control + risk reduction
therapy (e.g., cholesterol lowering agents).
 Accelerated hypertensive retinopathy (with bilateral disk swelling & severe HTN) patients: stepwise control of
BP over a few hours to avoid a sudden ↓ in BP which may ↓ perfusion of optic nerve head and CNS
(causing stroke).
1. Ethanol induced liver disease – pathogenesis, C/F, Mx & Complications [22]
a. Write about alcoholic hepatitis [20] – VSQ
Ans.
Alcoholic liver disease (ALD) is defined as liver damage, caused by over consumption of alcohol,
leading to fat accumulation, liver inflammation, and liver scarring.
The threshold for the development of ALD is consumption of >80 gm/day of alcohol.
Patient usually present in their 30s.
:
In liver, Alcohol Acetaldehyde acetate + LOTTTTs of Hydrogen
 reduction of NAD to NADH inhibits fatty acid oxidation in liver
 fat accumulation
Alcohol also activates microsomal enzyme oxidation system 
oxidative damage to the liver
Alcohol-related undernutrition protective antioxidants (e.g.,
glutathione, vitamins A & E).
Acetaldehyde  bind to liver cell proteins form Neoantigens 
inflammation.
This inflammation leads to alcoholic hepatitis  it can progress to liver cell necrosis & apoptosis.
Subsequent attempts at regeneration by stellate cells result in fibrosis & nodules cirrhosis
: ALD is a spectrum of disease and comprises of 3 pathological forms of liver damage
Forms of ALD Clinical Picture Investigation findings
 Hepatomegaly
1. Alcoholic
 {Fat accumulates in liver parenchyma} Mild serum bilirubin & ALP
steatosis
 Reversible stage
▪ There is inflammation of liver parenchyma; a/w binge
Moderate bilirubin, ALP, &
alcohol intake
2. Alcoholic ▪ Tender hepatomegaly; Aminotransferases (AST>ALT);
{ALT is low because of dietary deficiency of
hepatitis ▪ fatigue, fever, jaundice, RUQ pain & sometimes hepatic bruit
vitamin B6}
▪ Reversible stage
▪ In severe cases, it can lead to liver failure Neutrophilia
3. Alcoholic bilirubin, aminotransferases
Features of liver cell failure & portal hypertension
cirrhosis & ALP; Hypoproteinemia

Abstinence from alcohol & Supportive care.


Corticosteroids or pentoxifylline for severe alcoholic hepatitis.
Rarely liver transplantation.
Hepatic coma; GIT hemorrhage; Hepatorenal syndrome;
Hepatocellular carcinoma
----------------------------------------------------------------------------------------------------------------------------------------
2. Cirrhosis of liver – causes, C/F, Lab inv. complications & Mx [21, 15, 13, 04]
a. Signs of Liver cell Failure / Signs of Hepatocellular failure/Insufficiency [22, 19, 14]
Ans.
refers to a late stage of progressive hepatic fibrosis characterized
by distortion of the hepatic architecture and the formation of regenerative
nodules. It is the final common pathway of many types of chronic liver
injury.
Causes of Cirrhosis
Drugs and toxins: Alcohol (Laennec’s cirrhosis), Amiodarone, Arsenicals,
Oral contraceptives, Pyrrolizidine alkaloids and antineoplastic agents (Veno-occlusive disease)
Infectious diseases: Hepatitis B, C, D, CMV, EBV, Schistosomiasis etc.
Genetic causes: α1 antitrypsin deficiency, Wilson’s disease, Hemochromatosis, Galactosemia,
Gaucher’s disease, Glycogen storage disease, Cystic fibrosis etc.
Biliary disorders: Primary biliary cirrhosis, Biliary atresia, Primary sclerosing cholangitis, Chronic
biliary obstruction, Progressive familial intrahepatic cholestasis
Cardio vascular causes: Chronic right heart failure (cardiac cirrhosis), Budd-Chiari syndrome, long
standing portal vein thrombosis
Others – Non-alcoholic fatty liver disease; Autoimmune hepatitis; Scleroderma; Cryptogenic etc.

Investigations
Investigations to Identify the Underlying Cause
Hepatitis serologies (HBsAg, anti-HCV, anti-HDV).
Iron, total iron-binding capacity and ferritin – If
hemochromatosis is suspected.
Antimitochondrial antibody (AMA) – If primary biliary
cirrhosis is suspected.
Antinuclear antibody, anti-smooth-muscle antibody – If autoimmune etiology is suspected.
Serum copper and ceruloplasmin levels if Wilson disease is suspected.
α1 – antitrypsin levels if deficiency is suspected.
Complete Blood Count: Anemia may be present due to blood loss, folate deficiency & hypersplenism.
Pancytopenia due to hypersplenism.
Liver Function Tests: Hypoalbuminemia and globulin levels (reversal of A:G ratio).
 Bilirubin level & amino transferases.
 ALP & PT may be prolonged.
Urea & Creatinine - are usually normal unless there is hepatorenal syndrome
Serum Electrolytes: hypoglycemia may be present – due to impaired gluconeogenesis by the liver.
Imaging
Abdominal ultrasound with Doppler – reveal nodular liver & splenomegaly and dilated portal vein.
CT or MRI is rarely required.
Liver Biopsy (Percutaneous, Transjugular, or Open): It shows regenerating nodules and fibrosis.
Complications of cirrhosis:
1) Portal hypertension with clinical consequences – Ascites Portosystemic shunts manifesting as
hemorrhoids, esophagogastric varices & caput medusae Congestive splenomegaly Hepatic
encephalopathy
2) Hepatorenal syndrome & Hepatopulmonary syndrome
3) Hepatocellular carcinoma
Treatment
Diet:
▪ protein intake if there is hepatic encephalopathy.
▪ Restrict sodium if there is fluid retention.
▪ Vitamin supplementation - especially Vit K.
For ascites:
 Albumin transfusions.
 Diuretics (spironolactone and frusemide)
 Paracentesis is indicated for tense ascites
 TIPSS in severe refractory ascites.
For Portal hypertension:
 Use non-selective ß-blockers such as propranolol or nadolol.
 Alternative: Nitrates can be used for patients in whom ß-blockers are contraindicated.
For Esophageal varices – endoscopic variceal ligation
Lactulose syrup is used daily (15 ml at night) to prevent hepatic encephalopathy.
All patients with cirrhosis should receive the HAV, HBV, and pneumococcal vaccines and a yearly
influenza vaccine
Liver transplantation can be considered in suitable patients.
----------------------------------------------------------------------------------------------------------------------------------------
3. Portal HTN – Classify, aetiopathogenesis, C/F, evaluation,
Complications & Mx [18, 16, 14, 12]
a. Caput medusae [05]
Ans.
Portal hypertension: It is sustained elevation of the portal venous
pressure more than 10 mm of Hg. {normal value is 5-10 mm of Hg}
Clinical Features:
 Triad of portal hypertension – Oesophageal varices,
splenomegaly & ascites
 Jaundice, recurrent infection, SBP & Coagulopathy.
 Features of encephalopathy – memory loss, altered behaviour and mental status, asterixis (flapping
tremor/liver flap), unconsciousness, foetor hepaticus
 Features of liver cell failure – gynaecomastia, palmar
erythema, leukonychia, testicular atrophy, spider
angioma etc.
 Kenawy's sign: Venous hum which is heard louder on
inspiration in epigastrium in portal HTN.
 Hepatorenal syndrome – Decreased urine output &
features of renal failure

Evaluation of Portal HTN: {refer 2nd LQ}


Caput medusae (so called because of its resemblance to
the head of the Greek god, Medusa) is one of the cardinal
features of portal hypertension.
Portal hypertension  blood from the portal venous system will be shunted through the
postnatally redundant periumbilical veins, which thereby
reopens them  they radiate from the umbilicus across
the abdomen to join systemic veins.
Blood from the portal venous system is shunted through
the umbilical veins into the abdominal wall veins, which
manifest as the caput medusae.
DDx: inferior vena cava syndrome and superior vena cava
syndrome.
These can be differentiated from each other by passing fingers
superiorly and inferiorly along the dilated veins to empty them of
blood; one side is allowed to refill and then the process is repeated
from the opposite direction.
In caput medusae, the blood flows in a cephalad and caudad
direction from the umbilicus.
In inferior vena cava syndrome, blood flows cephalad, whereas it flows caudad in superior vena cava syndrome.
No specific treatment is required for this condition – It is rarely encountered in clinical practice
nowadays due to earlier diagnosis and treatment of portal hypertension
----------------------------------------------------------------------------------------------------------------------------------------
4. Acute Viral Hepatitis – etiology, C/F, Complications & Mx [13, 04]
a. Liver function tests in Acute hepatitis [10]
Ans.
It is the MC consequence of all hepatotropic viruses.
:
Incubation period: Asymptomatic period
Pre-icteric phase – fatigue, anorexia, nausea, vomiting,
abdominal discomfort; headache; myalgia or arthralgia.
Icteric phase: Seen with jaundice, pruritus, tender
hepatomegaly, dark-colored urine.
Mild splenomegaly and cervical lymphadenopathy with EBV infection
Post-icteric phase: Clinical & biochemical recovery is seen
Serum bilirubin; Serum transaminases;  Alkaline phosphatase; PT
:
H V: Detection of serum IgM antibodies
H V: Detection of HBsAg or Ab to HBcAg; PCR for HBV DNA
H V: Anti-HCV antibodies; PCR for HCV RNA
H V: Detection of IgM & IgG antibodies; HDV RNA in serum
H V: Detection of IgM & IgG antibodies; PCR for HEV RNA
: Recovery; Fulminant hepatitis

Most individuals do not need hospital care; just out-patient supportive therapy; Antiviral are not used
Persistent nausea, vomiting & any mental confusion warrants hospitalization
General measures: Avoid sedatives, narcotics & alcohol (hepatotoxic drugs)
Liver transplantation – in cases of acute liver failure
----------------------------------------------------------------------------------------------------------------------------------------

1) Hepatic Encephalopathy – etiopath, C/F, DDx, inv. & Mx [22, 21, 14, 08, 02]
a. Signs in Hepatic Encephalopathy [11]
Ans. Hepatic encephalopathy (HE) is a reversible neuropsychiatric syndrome occurring in patients with
advanced liver failure.
Its severity ranges from inversion of sleep rhythm and mild intellectual impairment to coma.

'Neurotoxins' (ammonia, aminobutyric acid, amino acids, mercaptans and fatty acids) produced in
the gut, are normally metabolized by the healthy liver.
In the presence of liver failure and portosystemic shunting these nitrogenous substances enter
systemic circulation and brain  neurotoxic effects – esp. because of ammonia.
Disruption of the BBB is a feature of acute hepatic failure  cerebral edema and encephalopathy.

Inversion of sleep rhythm – daytime sleepiness and night time insomnia is an early symptom.
Clinical grades 👉🏻
Examination usually shows a flapping tremor (asterixis),
inability to perform simple mental arithmetic calculations and
draw objects such as a star (constructional apraxia).
Signs of liver cell failure such as fetor hepaticus (sweet musty
odor to the breath due to the presence of mercaptans),
jaundice, spider nevi, coagulation defect, disturbances, etc.
may be present.
Diagnosis is made by clinical features.
Serum ammonia levels.
EEG shows diffuse slowing of the normal α-waves with eventual
development of delta waves.
Serum electrolytes, urea, creatinine, glucose, etc. should be
done to rule out other causes of altered sensorium.
Brain imaging (CT or MRI) is required if stroke is suspected.

Oral Lactulose 30 ml three or four times daily. The dose should then be titrated. so that two or
three soft stools per day are produced – to ammonia levels
Rifaximin 400 mg three times daily controls ammonia producing intestinal bacteria. Alternative
antibiotics are metronidazole, neomycin and vancomycin.
Avoid Opioids & sedatives. Oxazepam can be given to control agitation as it is not metabolized by
the liver.
Dietary protein should be restricted.
Chronic or refractory hepatic encephalopathy requires liver transplantation.
----------------------------------------------------------------------------------------------------------------------------------------
2) Liver Function Tests & their interpretation [21, 19, 18, 14, 08, 03]
Ans.
Liver function tests are done for following purposes:
To Detect Liver dysfunction.
To Assess the severity of liver injury.
To Monitor the course of liver diseases and the response to
treatment.
Refining the diagnosis.
Interpretation of LFTs:
 Unconjugated bilirubin – seen in Prehepatic or hepatic
jaundice
Blood ammonia – seen in hepatic encephalopathy
ALT & AST – seen in acute hepatocyte injury
 Elevated ALT is more specific for liver injury. Because
AST is present in the heart, skeletal muscle, kidneys,
and pancreas
  AST/ALT ratio > 2 seen in alcoholic liver disease
ALP – it indicates cholestasis
GGT – seen in alcoholic liver disease & cholestasis
Albumin is produced solely by the liver. Hence albumin
levels can be decreased in liver dysfunction
Prothrombin time (PT) reflects the plasma concentrations
of factors VII, X, and V, prothrombin, and fibrinogen – all
these are synthesised in live; hence, prolonged PT occurs
in liver diseases

----------------------------------------------------------------------------------------------------------------------------------------
3) Hepatitis C [21, 15]
a. Extrahepatic complications of hepatitis C [17]
Ans.
Hepatitis C virus (HCV) is the common cause of post-transfusion hepatitis in developing countries. It
was first labelled as "non-A, non-B hepatitis virus”
It is spherical and enveloped; It contains a positive sense ssRNA

Parenteral route: blood transfusion & needle prick injuries.


Vertical (perinatal) transmission: during delivery (max risk)
Sexual mode – (rare)
Following an infection with HCV:
1. Acute hepatitis
2. Spontaneous clearance: About 5-15% of infections, the virus gets cleared spontaneously within 12
weeks
3. Chronic disease: About 75-85% directly develop chronic disease → chronic hepatitis → cirrhosis →
HCC.
4. Extrahepatic manifestations: Due to deposition of circulating immune complexes in extrahepatic
sites → Mixed cryoglobulinemia / Glomerulonephritis / Arthritis and joint pain.

 HCV Core Antigen Assay


 HCV Antibody Detection Assay – via ELISA
 Recombinant lmmunoblot Antibody Assay (RIBA): RIBA was used in past as a supplementary test
to confirm the ELISA result; not in use currently. It works based on the principle of western blot.
 Molecular Methods: Real time RT-PCR detecting HCV RNA has been the gold standard method.

Supportive care – to allow spontaneous clearance


Antivirals – Ex: protease inhibitors; interferon and ribavirin – used in chronic cases
Liver transplantation – in cases of fulminant hepatic failure and severe encephalopathy.

 Chronic hepatitis C & Fulminant hepatitis (massive hepatic necrosis).


 Pancreatitis
 Myocarditis & Atypical pneumonia
 Aplastic anemia
 Essential mixed cryoglobulinemia & B cell lymphoma.
 Transverse myelitis
 Peripheral neuropathy
 Immune complex disease (arthritis, cutaneous vasculitis, glomerulonephritis).
----------------------------------------------------------------------------------------------------------------------------------------
4) Complications of Pancreatitis [13]
a. Complications of Acute Pancreatitis [21, 17]
b. Causes and complications of Chronic Pancreatitis [16]
c. Management of acute pancreatitis [10]
Ans.
Acute pancreatitis – {refer Surgery Paper-II pg. no. 94}

Chronic Pancreatitis: Chronic pancreatitis is persistent


inflammation of the pancreas that results in permanent
structural damage characterised by a decline in exocrine and endocrine function (pancreatic
insufficiency).
Clinical features: Middle-aged alcoholic men with pain in epigastrium & features of malabsorption
& diabetes
Complications of Chronic Pancreatitis: Pseudocyst; Pancreatic ascites; Obstructive jaundice;
Duodenal stenosis; Portal or splenic vein thrombosis & Peptic ulcer
Management:
 Stop alcohol intake; Give Analgesics & pancreatic enzyme supplements
 Surgical or endoscopic pancreatic therapy – for severe cases
----------------------------------------------------------------------------------------------------------------------------------------
5) Chronic acute hepatitis [12, 04]
a. Causes of Chronic Hepatitis [15]
Ans.
: Chronic hepatitis is hepatitis that lasts >6 months
:
 Viruses – HBV; HCV(MC); HDV
 Alcoholic steatohepatitis & NASH {Nonalcoholic steatohepatitis}
 Metabolic causes (Wilson disease, hemochromatosis, alpha-1 antitrypsin deficiency, primary
biliary cirrhosis, sclerosing cholangitis).
 Autoimmune hepatitis & Cryptogenic hepatitis.
 Drugs (isoniazid, nitrofurantoin, amiodarone, methotrexate)
: Fatigue, loss of appetite or mild jaundice; Mild tender hepatomegaly or
splenomegaly.
:  serum transaminases; Prolonged PT; Hyperglobulinemia; Hyperbilirubinemia; 
alkaline phosphatase.
 HBV: Detection of HBsAg or antibody to HBcAg; PCR for HBV DNA
 HCV: Anti-HCV antibodies; PCR for HCV RNA
 HDV: Detection of IgM & IgG antibodies; HDV RNA in serum
– in untreated cases  Cirrhosis; Hepatocellular carcinoma

 Supportive care & Treatment of cause (e.g., corticosteroids for autoimmune hepatitis etc.)
----------------------------------------------------------------------------------------------------------------------------------------
6) Peritoneal fluid analysis In Cirrhosis of Liver [11]
a. Pathogenesis of ascites [10, 09]
Ans.

back pressure
Measurement of the protein concentration and the serum– ascites albumin gradient (SAAG) is done to
know the cause of hepatic or extrahepatic cause
Calculate the SAAG by subtracting the concentration of the ascites fluid albumin from the serum
albumin – A gradient of > 11 g/L (1.1 g/dL) suggest that ascites is due to portal hypertension {hepatic cause}
Cirrhotic patients typically develop ascites with a low protein concentration (‘transudate’)
----------------------------------------------------------------------------------------------------------------------------------------
7) Hepatitis B vaccines [09, 05]
a. Prevention and complication of Hepatitis B Virus [17, 03]
b. Complications of Hepatitis B [16]
Ans.
Complications of Hepatitis B:
Hepatic complications: Chronic hepatitis B; fulminant hepatitis; cirrhosis/ hepatocellular carcinoma
Extrahepatic complications: This is due to immune complex deposition.
▪ During the prodromal phase, a serum sickness-like syndrome characterized by arthritis, rash,
angioedema, and rarely, hematuria and proteinuria may develop.
▪ Glomerulonephritis with nephrotic syndrome.
▪ Polyarteritis nodosa-like systemic vasculitis.
Others: Atypical pneumonia. Aplastic anemia Transverse myelitis
Prevention of Hep-B: HBV can be destroyed by hypochlorite and heat (by autoclaving).
➢ Active Immunization (Hepatitis B Vaccine)
Route of administration: IM; over deltoid region (in infant-anterolateral thigh)
Schedule:
Under national immunization schedule: It is given at 6, 10, 14 weeks (along with DPT vaccine).
Recommended schedule for adults: Three doses are given at 0, 1 and 6 months
Minimum interval between the doses – 4 weeks.
Marker of protection: anti-HBsAg antibody titer > 10 mIU/mL
➢ Passive Immunization: (Hepatitis B Immunoglobulin or HBIG) - used in situations where an
immediate protection is needed (Ex: Neonates borne to hepatitis B carrier mothers)
----------------------------------------------------------------------------------------------------------------------------------------
8) Amoebic liver abscess & it’s Mx [07, 04] ➔ refer surgery paper-II pg. no. 77
----------------------------------------------------------------------------------------------------------------------------------------
9) Wilson's disease [04, 03, 99]
Ans.
Wilson disease is a rare autosomal recessive disorder of copper metabolism.
: Here, total body copper is increased, and excess copper is deposited in various
organs causing damage
▪ It is caused by a variety of mutations in the gene ATP7B on chromosome 13.
▪ In Wilson disease, there is failure of synthesis of ceruloplasmin & impairment in biliary excretion
accumulation of copper in the liver liver is progressively damaged

Excess copper accumulation  damage to organs like liver and nervous system.
Liver disease can manifest in many ways such as acute hepatitis, fulminant liver failure, chronic
hepatitis and cirrhosis.
Neurological damage causes basal ganglion syndromes and dementia.
are greenish-brown discoloration of the corneal margin
Other manifestations include renal tubular damage and osteoporosis.

Low serum ceruloplasmin and high serum copper level.


24-hour urinary copper excretion: >40 μg/day is highly suggestive of Wilson disease.
AST and ALT are usually elevated.
Liver biopsy: Copper content is high.
Genetic testing – for mutations.
Aim is to remove the tissue copper that has accumulated and then prevent re-accumulation.
For Copper removal – use chelators such as penicillamine, trientine & oral zinc
For prevention of re-accumulation – use lower dose of penicillamine or trientine or zinc & avoid
copper-rich foods such as liver, kidney, shellfish, nuts, beans, peas, chocolate, and mushrooms.
First degree relatives should be screened for Wilson disease
Liver transplantation is indicated for fulminant hepatic failure.
----------------------------------------------------------------------------------------------------------------------------------------
10) Indian Childhood cirrhosis [04, 02]
Ans.
Indian childhood cirrhosis (ICC) is a progressive liver failure in young children, associated with
marked hepatic copper overload
Etiology: excess dietary intake of copper from copper or brass household utensils in a genetically
susceptible individual.
Unlike Wilson disease, ICC is associated with normal or elevated ceruloplasmin level & affects only
the liver; Neurologic deficits and Kayser-Fleischer rings are not seen in ICC.
{Management is same as for copper overload Wilson’s}
----------------------------------------------------------------------------------------------------------------------------------------
11) Management of Neonatal Hyperbilirubinaemia [04] =➔ Refer Paeds notes pg. no. 23
a. Neonatal jaundice [01, 96]
----------------------------------------------------------------------------------------------------------------------------------------
12) Hydatid disease [04]➔ refer Surgery Paper-II Pg. No. 81
----------------------------------------------------------------------------------------------------------------------------------------
13) Kernicterus [03]
Ans.
Kernicterus refers to brain damage caused by unconjugated bilirubin deposition in basal ganglia
and brainstem nuclei
It occurs in neonates due to hyperbilirubinemia of various reasons.
Clinical features of kernicterus: Choreoathetoid cerebral palsy (chorea, ballismus, tremor),
sensorineural hearing loss, gaze abnormalities (especially limitation of upward gaze), dental-
enamel dysplasia. Cognitive function is usually spared.
Management: kernicterus will subside after the treatment of hyperbilirubinemia.
----------------------------------------------------------------------------------------------------------------------------------------
14) Hepatocellular carcinoma [2000]
Ans.
Hepatocellular Carcinoma: “Primary malignant tumor of liver.”
: M > F.
:
1) Viral infections - HBV & HCV
2) Toxins – Alcohol & Aflatoxin;
3) Metabolic diseases - Hereditary hemochromatosis, α1 antitrypsin deficiency
Hepatocellular adenoma, dysplastic nodule.
Activating β-catenin mutations & inactivating p53 mutations.
Vague upper abdominal pain, fatigue, weight loss, or an abdominal mass. Jaundice,
fever, or esophageal variceal bleeding may be seen.
↑ serum α-fetoprotein levels; Radiological (USG; CT; MRI).
Cachexia; GIT or esophageal variceal bleeding; Hepatic coma
: Lungs

 Local ablative therapies – Ex: Transcatheter arterial chemoembolization (TACE); Radiofrequency


ablation; Percutaneous ethanol injection etc.
 Surgical therapy – hepatic resection or Liver transplantation
 Prevention: Hepatitis B vaccination can prevent cancer
----------------------------------------------------------------------------------------------------------------------------------------
15) Gilbert's syndrome [2000]
Ans.
Gilbert’s syndrome is by far the most common inherited disorder of bilirubin metabolism.
:
It is an autosomal recessive trait caused by a mutation in the promoter region of the gene for
UDP-glucuronyl transferase enzyme (UGT1A1)   enzyme expression.
It can be inherited in a dominant fashion when there is a missense mutation in the gene.
 enzyme expression   conjugation of bilirubin  unconjugated bilirubin in the blood.
The levels of unconjugated bilirubin increase during fasting, as fasting reduces levels of UDP-glucuronyl transferase.
:
Typical presentation is with isolated bilirubin (Jaundice) in the setting of physical stress or illness.
 excretion of bilirubin {stercobilinogen & urobilinogen}  Dark stools & Dark urine.
In the presence of haemolysis, pallor and splenomegaly can be seen.
:
Plasma bilirubin
LFTs are otherwise normal.
Hepatic histology – it will be normal.
The condition is not associated with liver injury and thus has an excellent prognosis, needs no
treatment & is clinically important only because it may be mistaken for more serious liver disease.
----------------------------------------------------------------------------------------------------------------------------------------

1. Viral markers of Hepatitis B [19]


a. Viral markers in chronic Hepatitis [18]
Ans.
 Presence of HBsAg confirms the diagnosis of hepatitis B infection.
 Presence of lgM anti-HBc indicates acute infection.
 Presence of serum IgG anti-HBc indicates chronic hepatitis B infection.
 Presence of HBeAg is associated with high infectivity.
 Serum anti-HBsAg indicates immunity and found during recovery from hepatitis B and after
vaccination
----------------------------------------------------------------------------------------------------------------------------------------
2. Treatment of Acute Fulminant Hepatitis [18]
Ans.
Acute liver failure also known as fulminant liver failure is rapid deterioration of liver function
resulting in coagulopathy and encephalopathy in a previously healthy individual
:
:
Identify & treat the underlying cause
Supportive care:
▪ Adequate fluid, nutrition & electrolyte
balance;
▪ PPI – to control GI bleeding
▪ Manage encephalopathy via mannitol, lactulose & hyperventilation + Abx like neomycin or
rifaximin orally to sterilize the bowel and ammonia production by bacteria
▪ Avoid sedatives
▪ Hemodialysis for renal failure
▪ Replace clotting factors by transfusion of fresh frozen plasma (FFP) + Vit K injections
Liver transplantation is the definitive treatment in liver failure
----------------------------------------------------------------------------------------------------------------------------------------
3. Management of Cholelithiasis [17] ➔ refer Gallstones – Surgery Paper-II pg. no. 85
----------------------------------------------------------------------------------------------------------------------------------------
4. Pancreas Divisum [16]
Ans. It is congenital defect that occurs due to non-fusion of the duct of Wirsung and Santorini
This result in most of the pancreas drain through the duct of Santorini and the minor papilla.
This predispose to attacks of acute pancreatitis and the treatment for this acute pancreatitis is
minor papillary sphincterotomy.
MRCP is superior to ERCP in detecting pancreatic divisum
----------------------------------------------------------------------------------------------------------------------------------------
5. Indications for liver biopsy [16]
Ans.
Liver disease – Storage & Metabolic disorders {Ex: Glycogen storage disorders, Wilson’s disease};
Cirrhosis; Chronic Hepatitis; Hepatic tumours; Unexplained Hepatomegaly etc.
After Liver transplantation – to assess for rejection & to look for disease recurrence
P.U.O. when a/w Hepatomegaly – Ex: Miliary TB, Brucellosis; Cholangitis; Malignancy etc.
Staging of lymphoma
----------------------------------------------------------------------------------------------------------------------------------------
6. Causes and treatment of Spontaneous bacterial peritonitis (SBP) [14]
Ans.
 SBP is defined as an ascitic fluid infection without an evident intra-abdominal surgically treatable
source
 Causes: Escherichia coli {MC; enteric in origin}
 Treatment
 Broad-spectrum antibiotics, such as cefotaxime or piperacillin/tazobactam
 Prophylactic quinolones like norfloxacin or ciprofloxacin to Recurrence of SBP
----------------------------------------------------------------------------------------------------------------------------------------
7. Indications for Liver transplantation [12]
a. Liver transplantation [11]
Ans.
▪ Fulminant hepatic failure
▪ Metabolic diseases (hemochromatosis, galactosemia)
▪ Decompensated cirrhosis
▪ Hepatocellular carcinoma
▪ Hepatic trauma
----------------------------------------------------------------------------------------------------------------------------------------
16) Serum amylase [10, 06, 05]
Ans.
Amylase splits starch to maltose. It is produced by pancreas and salivary glands.
The serum amylase contains the P (pancreatic) and S (salivary) iso-enzymes. These two can be
distinguished by the inhibition test.
Normal serum value is 50–120 IU/L. The value is increased about 1000 times in acute pancreatitis.
Moderate increase in serum levels is seen in chronic pancreatitis, mumps (parotitis) and
obstruction of pancreatic duct.
Mild increase of serum amylase is seen in – cholecystitis, peptic ulcer, diseases of mesentery and
obstruction of intestine
Macroamylasemia is a condition characterized by persistent elevation of serum amylase activity
with no apparent clinical symptoms of pancreatic disease; here the amylase complexes with
immunoglobulins, which prevents renal excretion
----------------------------------------------------------------------------------------------------------------------------------------
8. Alkaline phosphatase [09, 05]
a. Serum alkaline phosphatase [07]
Ans.
Alkaline phosphatases can be found in many organs (liver, bile ducts, intestine, bone, kidney,
placenta, and leukocytes).
Serum alkaline phosphatase mainly comes from liver and bone.
ALP is seen in cholestatic hepatobiliary diseases.
ALP is also elevated in bone disorders (Pager's disease, osteomalacia, bone metastases), during
rapid bone growth in children, pregnancy, and chronic renal failure.
----------------------------------------------------------------------------------------------------------------------------------------
9. Urobilinogen [07]
Ans.
Urobilinogen – detected by Ehrlich test in urine
 Urobilinogen in urine indicate hemolytic (pre-hepatic) jaundice.
Urobilinogen is absent in urine, if there is obstruction to bile flow. The first indication of recovery
is the reappearance of urobilinogen in urine
----------------------------------------------------------------------------------------------------------------------------------------
10. Principles of ultra-sonography [07]
Ans.
Ultrasound is an oscillating pressure sound wave with a frequency (3 to 20 MHz) greater than the
human audible range
Diagnostic ultrasonography uses ultrasound produced by the transducer (probe) to image various
organs and parts
The basic principle of sonography is piezoelectricity which refers to a characteristic property of
certain elements, which on application of electronic energy vibrate and convert it into mechanical
energy (ultrasound) and vice versa.
The most commonly utilized piezoelectric element for sonography is compressed microcrystalline
lead titanate zirconate (PZT)
PZT is arranged on ultrasound probe or transducer
Depending on the geometry and structural properties of PZT, transducer produce different range
of sound frequencies, which is used for imaging of various organs located at different depth from
the skin surface
1. Nephrotic Syndrome – etiology, C/F, Lab inv. Dx, complications & Mx [21, 20, 17, 13, 12, 11, 08, 05]
a. Essentials for diagnosis of Nephrotic Syndrome [18]
Ans.
Nephrotic Syndrome is a glomerular disease
characterised by the classic tetrad of findings:
(> 3.5g/day or > 4mg/kg/hr in child),
,
: 👉🏻

 Peripheral edema – Initially it is noted in the dependent


areas such as the lower extremities, but later becomes
generalized. Early morning facial puffiness is seen in most
patients
 Dyspnea – due to pulmonary edema and pleural effusion.
 Ascites may be present.
 Patients are more prone to infection due to loss of
immunoglobulins and complements in the urine.
 Patients also have hypercoagulable state due to urinary
losses of antithrombin III, protein C, protein
S. and increased platelet activation.
Patients are prone to renal vein thrombosis
and other venous thrombo-emboli.
 Microcytic hypochromic anemia may result
from loss of transferrin in the urine.
 Vit D deficiency may result from loss of
cholecalciferol binding protein.
of Nephrotic Syndrome:
❖ Coronary atherosclerosis (due to
Hyperlipidemia)
❖ Infections: esp. staphylococci and
pneumococci due to loss of
immunoglobulins in the urine.
❖ Loss of Complements (esp. C3b) → ↑risk
of pneumococcal infections
❖ Loss of Transferrin → Fe Def. anemia
❖ Thrombotic complications: due to loss of endogenous anticoagulants (e.g., antithrombin III) in the
urine.

 Urine analysis – shows heavy proteinuria + hematuria


 Serum albumin – low (<3 g/dl)
 Serum anticoagulants and complement – decreased.
 Blood sugar and glycosylated hemoglobin – to r/o diabetes
 ANA & ANCA – to r/o collagen vascular disease & vasculitis
 Total cholesterol and LDL-cholesterol – Increased
 Urea & Creatinine – may be elevated if there is renal failure

Protein Loss – replace with  dietary intake; ACEI & ARBs


will also help to  proteinuria.
Edema – dietary salt + thiazide and loop diuretics.
Hyperlipidemia – Exercise + Dietary modification + Statins.
Hypercoagulable State – Start Anticoagulation therapy at least 3-6 months
Treatment of the Underlying Cause – Example:
 Minimal change disease responds to steroids.
 Membranous nephropathy responds to alternating monthly corticosteroids and monthly oral
chlorambucil over 6 months.
----------------------------------------------------------------------------------------------------------------------------------------
2. Acute Renal Failure – classify, etiology, C/F, DDx, inv. & Mx [21, 20, 18, 17, 15, 14, 11, 09, 02]
a. Urine microscopy in acute renal failure [10]
b. Acute Tubular Necrosis [03] ➔ aka AKI & ARF
Ans.
Acute kidney injury (AKI) is not a diagnosis, rather it describes
the situation where there is a sudden loss of renal function,
which develops over days or weeks and is accompanied by 
urine volume.
Types of AKI: based on Etiology {DDx}
‘Pre-renal’, when perfusion to the kidney is reduced
‘Renal’, when the primary insult affects the kidney itself
‘Post-renal’, when there is obstruction to urine flow at any
point from the tubule to the urethra
Clinical Features: Uremia can cause the following features-
Nausea, vomiting, malaise & altered sensorium.
Platelet dysfunction can lead to bleeding
CNS: Encephalopathy with flapping tremors and confusion.
seizures can occur.
Lung: crepitations due to volume overload.
Signs and symptoms of underlying disorders may be present.
In AKI clinical course may be divided into 3 phases –
1. OLIGURIC PHASE Oliguria or anuria
Lasts on an average Features of fluid overload – pedal edema, ascites, pleural & pericardial
of 10-14 days effusions
2. MAINTAINANCE  Low GFR; Low urine output
PHASE-  This phase lasts for days to weeks
Polyuria occurs due to defective urine concentrating ability of tubules
3. DIURETIC PHASE-
Dehydration, hyponatremia and hypokalemia may be seen
After diuretic phase kidney function usually recovers
Investigations:
» Urine analysis: {Urine microscopy in acute renal failure}
 Urine is acellular – Pre-renal AKI
 Pigmented "muddy brown" granular casts &
casts containing tubule epithelial cells are
characteristic of ATN {Renal AKI}
 RBC casts – Glomerular injury
 Abundant uric acid crystals – acute urate
nephropathy
 Hemoglobinuria or myoglobinuria – Renal
failure due to hemolysis & rhabdomyolysis.
 Eosinophiluria – Allergic interstitial nephritis
» Renal ultrasound – to detect hydronephrosis
» Coagulation profile
» Renal biopsy
» CT angiography
Management 👉🏻
----------------------------------------------------------------------------------------------------------------------------------------
3. Acute glomerulonephritis – etiology, C/F, complications and Mx [17, 97]
a. Treatment of Acute Proliferative Glomerulonephritis [19]
b. Treatment of Acute Post Streptococcal glomerulonephritis [14]
c. Clinical features of acute nephritis [13]
Ans.
Glomerulonephritis literally means 'inflammation of glomeruli'.
Here the glomeruli are damaged due to inflammation.

Patients present with Nephritic Syndrome-


Hematuria (gross or microscopic)
Oliguria
Mild to moderate Hypertension
Edema
Mild to moderate proteinuria
(Edema & proteinuria are common but not as severe as in nephrotic
syndrome)
-
Urinary analysis shows hematuria, moderate proteinuria
(usually <2 g/ d), RBC casts, and WBCs. Red cell casts are specific for glomerulonephritis.
 Serum Complement levels (C3, C4).
ASO titer, anti-DNase or anti-hyaluronidase in post-streptococcal glomerulonephritis + positive
Streptococcal culture
 Anti-GBM antibody levels are elevated.
ANCA, ANA if connective tissue disease is suspected.
Hepatitis serologies.
Renal ultrasound.
Renal biopsy if the cause is not clear.
-
• Treatment is mainly supportive – salt restriction, diuretics, anti-hypertensives & dialysis if
required.
High-dose steroids, cyclophosphamide, and rituximab.
Plasmapheresis can be used in Good-pastures disease as a temporary measure until chemotherapy
takes effect.
Treat the Underlying disease – Ex: Treatment of Acute Post Streptococcal glomerulonephritis
 Antibiotic choices are penicillins (ampicillin, amoxicillin) or cephalosporins or macrolides or
clindamycin. Oral penicillin V or amoxicillin are the 1st DOC and are given for 10 days.
 Prognosis is good and permanent renal failure is Uncommon
----------------------------------------------------------------------------------------------------------------------------------------
4. Chronic Renal Failure – causes, pathogenesis, C/F & complications [16, 04, 2000]
Ans.
Chronic kidney disease (CKD) (earlier known as chronic renal failure) is defined as either kidney
damage or a  GFR of le ss than 60 mL / min/ 1.73 m2 for > 3 months.
CKD can be divided into the following stages:

 CKD stages 1 and 2 cannot be diagnosed based on GFR alone as GFR can be normal in these stages.
 Stage 5 CKD is also known as end stage renal disease.
Investigations:
 RFT - urea & creatinine
 Urine analysis – reveal loss of urine concentrating ability of kidneys
 Anemia is seen which is usually normocytic normochromic.
 Serum electrolytes: Hyperkalemia, hypocalcemia, and hyperphosphatemia are seen
 Ultrasound abdomen reveals bilateral small sized kidneys.
 Chest X-ray: May show pulmonary edema and pericardial effusion
 ECG: May show signs of Hyperkalemia or cardiac disease.
 Renal artery Doppler: If renal artery stenosis is suspected
 ANA if connective tissue disease is suspected
 ANCA if vasculitis is suspected
 Renal biopsy to establish the diagnosis
MANAGEMENT:
1. TREATMENT OF UNDERLYING CAUSE OF CKD: Ex: control of diabetes, hypertension, etc.
2. SLOWING THE PROGRESSION OF CKD: ACE inhibitors, ARBs & Restriction of dietary protein have
been shown to slow the progression of CKD in diabetics
3. TREATMENT OF THE COMPLICATIONS OF RENAL FAILURE

4. RENAL REPLACEMENT THERAPY –


 If conservative measures are inadequate, hemodialysis must be planned;
 Renal transplantation can be considered if suitable donor is found
----------------------------------------------------------------------------------------------------------------------------------------
5. Urinary tract infection – etiology, C/F, Dx & Tt. What
factors perpetuate UTI in the female? [10, 03]
a. Complicated urinary tract infection [22]
b. What is the spectrum of infection in the urinary tract? Give the clinical
features, investigations and treatment of (a) Pyelonephritis (b) Cystitis
(c) Urethritis. [18, 11, 05]
c. Urinary Tract infection in males [15, 10]
d. Mention recent drugs used in the treatment of UTI [13]
Ans.
Uncomplicated UTI is cystitis or pyelonephritis that occurs in
healthy, nonpregnant women without any structural
abnormality of the urinary tract or comorbid illness that can
increase the risk of complications.
Complicated UTI is cystitis or pyelonephritis which occurs in
patients with structural or functional urinary tract
abnormality and obstruction of urine flow; patients with
comorbid illness (e.g., diabetes); pregnant ladies and
children; or after instrumentation of the urinary tract.

 frequency of micturition.
Pain in the urethra during micturition (dysuria).
Suprapubic pain during and after voiding (in cystitis).
Urgency.
Intense desire to pass more urine after micturition due to spasm
of the inflamed bladder wall (strangury).
Passing cloudy urine with unpleasant odor and occasionally
hematuria.
Systemic symptoms such as fever and chills may occur.

intake to maintain good urine output.


–based on C & S. Since the most
common organism is E. coli, it is susceptible to 
cotrimoxazole, amoxicillin, ampicillin, cephalosporins,
quinolones and nitrofurantoin.
Complicated UTI requires for 7-14
days.
should be treated if the Patient
is pregnant or immunocompromised.
Any predisposing to UTI should be corrected – BPH, cystocele, stones etc.
with potassium citrate may help if there is severe dysuria.
----------------------------------------------------------------------------------------------------------------------------------------
6. Draw a diagram of nephron. List the
diuretics used in clinical practice &
their anatomical sites of Action. [09]
Ans.

----------------------------------------------------------------------------------------------------------------------------------------
1) Renal Tubular Acidosis (incl types) [22, 21]
Ans.
Renal tubular acidosis (RTA) refers to the development of
metabolic acidosis due to a defect in the kidney to reabsorb
bicarbonate or to excrete hydrogen ions.
ABG shows normal anion gap (hyperchloremic)
metabolic acidosis in all types of RTA. {Due to inability to
reabsorb bicarbonate, there will be compensatory increase in chloride; hence
anion gap will be normal}

▪ Occurs due to carbonic


anhydrase II deficiency in
both proximal and distal
tubules
▪ Has features of both type
1 and type 2 RTA
▪ Treatment is same as that
for type 1 and type 2
RTA.
----------------------------------------------------------------------------------------------------------------------------------------
2) Hemodialysis & it’s indications [19, 15, 14]
Ans.
Haemodialysis is the most common form of dialysis employed in ESRD and is also used in AKI
Hemodialysis is done for 3-4 hours 3-4 times a week.
Besides low GFR, hemodialysis can also be done for
the following indications:

----------------------------------------------------------------------------------------------------------------------------------------
3) Renal Function Tests (RFT) [19]
a. Unilateral renal function test [06]
b. Glomerular Filtration Rate [05]
c. Creatinine Clearance [04]
Ans.
Glomerular filtration rate (GFR) is the
. It is expressed in mL/min.
Normal GFR in young, healthy adults is about 120 to 125
mL/min and declines with age.
GFR decreases due to disease or damage to kidneys.

INULIN CLEARANCE – Best method


Inulin is neither absorbed nor secreted by the renal tubule
and, therefore, it is the ideal for measuring GFR.
Estimated GFR (eGFR) can be calculated by using serum
creatinine or cystatin C levels.
The estimated glomerular filtration rate (eGFR) is used to
screen for and detect CKD, and to monitor kidney status.

Clearance is defined as the volume of blood or plasma


completely cleared of a substance per unit time
Creatinine Clearance is used as a GFR marker; it has both
the advantages & disadvantages👉🏻👉🏻👉🏻👉🏻👉🏻👉🏻
Creatinine level more than 1.5 mg/dL indicates impairment
of renal function. Creatinine is quantitated by Jaffe’s test
(alkaline
picrate).
:
𝑈𝑥𝑉
𝑃
is the urine creatinine concentration; is the plasma creatinine concentration and is the
urine flow in mL/min
----------------------------------------------------------------------------------------------------------------------------------------
4) Lupus nephritis [16]
Ans.
Kidney may be involved in about 50% of SLE patients
Pathogenesis of Glomerulonephritis: Immune complexes composed of DNA and anti-DNA antibodies
get deposited in the glomeruli  inflammation  proliferation of cells (endothelial, mesangial and/or
epithelial).
Morphologic classification of lupus nephritis: 6 patterns
1) Class I: immune complex deposition in the mesangium.
2) Class II: + ↑ in mesangial cells & matrix.
3) Class III: + fibrinoid necrosis & leukocyte infiltration.
4) Class IV: + homogeneous thickening of the capillary wall, which on light microscopy appear as a “wire-loop” lesion.
5) Class V: is characterized by diffuse thickening of the capillary walls.
6) Class VI: It shows sclerosis of more than 90% glomeruli.
Clinical Features:
• It predominantly affects young women, with female-to-male ratio of 9:1.
• Typical presentation: Butterfly rash over the face, fever, pain without deformity in one or more
peripheral joints, pleuritic chest pain and photosensitivity.
• SLE patients are susceptible to infections, because of immune dysfunction and treatment with
immunosuppressive drugs.
Management
High-dose glucocorticoids + cyclophosphamide or mycophenolate mofetil.
Many patients with SLE who develop ESRD go into remission, possibly because of
immunosuppression related to the ESRD.
Patients with ESRD caused by SLE are good candidates for dialysis and transplantation.
----------------------------------------------------------------------------------------------------------------------------------------
5) IgA Nephropathy [15]
Ans. IgA nephropathy is the MC type of primary glomerulonephritis.
:
IgA nephropathy is an autoimmune disease, causing antibody-mediated destruction of the
glomerular basement membrane.
Infection  triggers abnormal immune response  IgA immune complexes deposit in glomeruli 
Damage to basement membranes glomerulonephritis

More common in young males


Patient presents with gross painless hematuria, proteinuria and hypertension within 1-2 days of a
febrile mucosal illness (respiratory tract, GI tract).
There may be acute exacerbations of hematuria in association with respiratory tract infections.

Urinalysis.
Renal biopsy shows mesangial deposition of IgA & complement (C3) on immunofluorescent staining

Patients with mild disease – monitor at 6 to 12-month intervals to assess for disease progression.
For Patients with progressive disease – start on IV methylprednisolone 1 gm daily for 3 days
For Patients with persistent proteinuria – give ACE inhibitors & Omega-3 fatty acids (fish oil)
----------------------------------------------------------------------------------------------------------------------------------------
6) Renovascular Hypertension [15]
Ans.
Renal artery stenosis refers to narrowing of one or both renal arteries
: Atherosclerosis (MCC); Fibromuscular dysplasia & Vasculitis
(Takayasu’s, PAN).
:
Hypertension is present if RAS is unilateral. Hypertension is due to
activation of the RAAS in response to renal ischemia.
Renal failure if RAS is bilateral.
Evidence of vascular disease elsewhere especially in the legs.
Deterioration of renal function with ACE inhibitors & ARBs (since they block efferent arteriolar
vasoconstriction which maintains glomerular filtration pressure in ischemic kidney).
Repeated flash pulmonary edema.

Ultrasound abdomen: It shows asymmetry in kidney size in unilateral RAS and bilaterally shrunken
kidneys in bilateral RAS.
Renal artery Doppler – to identify renal artery stenosis.
Renal isotope scanning –  uptake of isotope &  excretion by the affected kidney.
Renal arteriography is the definitive test, but is invasive – risk of contrast nephropathy.
MR angiography and spiral CT angiography are non-invasive & are being increasingly used.

Medical management with antihypertensives, low-dose aspirin and lipid-lowering drugs.


Angioplasty, with placement of stents.
Surgical resection of the stenosed segment and re-anastomosis.
----------------------------------------------------------------------------------------------------------------------------------------
7) Polycystic kidney disease – etiopath, C/F & inv. [14, 12]
Ans.
Polycystic kidney disease is a common disorder, occurring in approximately 1 in every 1000 live births.
It is characterized by presence of extensive cysts scattered throughout both kidneys.
➢ There are 2 types of PKD. Infantile polycystic kidney disease and adult polycystic kidney disease.
Infantile PKD is AR and usually fatal. Adult PKD is common and is inherited as AD trait.

Small cysts lined by tubular epithelium develop from infancy or childhood and enlarge slowly
compressing the normal kidney tissue leading to renal failure.

Patients are usually asymptomatic until later life.


After the age of 20 there is often insidious onset of hypertension.
One or both kidneys may be palpable and the surface may be nodular.
Family history is usually positive.
Gradual reduction in renal function.
Some patients may have hepatic cysts, and berry aneurysms in the brain.
Mitral and aortic regurgitation are also frequent in these patients.

Ultrasound abdomen shows multiple cysts in both kidneys.


Urea and creatinine are elevated.
The following criteria exist for an ultrasound diagnosis of PKD in patients with a family
history but unknown genotype:
15–39 years of age: at least 3 unilateral or bilateral kidney cysts.
40–59 years of age: at least two cysts in each kidney.
60 years or older: at least four cysts in each kidney.
There is no treatment to alter the rate of progression of renal failure.
Control of hypertension.
Dialysis if required.
Kidney transplantation in end stage renal disease
----------------------------------------------------------------------------------------------------------------------------------------
8) Microscopic haematuria [10]
Ans.
 Haematuria is defined as three or more RBCs per high-power field (HPF).
 Haematuria may be grossly visible (macroscopic haematuria) or detectable only on urine
examination (called microscopic haematuria).
 Red urine does not always indicate haematuria.
 Red urine without haematuria is seen in haemoglobinuria, myoglobinuria, beeturia (excess
beetroot ingestion), medications (phenazo-pyridine, methyldopa, senna) and porphyria.

----------------------------------------------------------------------------------------------------------------------------------------
9) Renal handling of uric acid [07]
a. Serum Uric acid [09]
Ans.
Serum Uric acid:
 Normal blood level: and
Uric acid is almost completely reabsorbed in the PCT, by both active and passive carrier mediated
processes.
There is also active secretion of uric acid into the tubules in DCT.
The daily excretion varies from 500–700 mg
The drug, probenecid is secreted by the tubule, and competes with uric acid for reabsorption.
Since probenecid increases uric acid excretion, it is uricosuric.
----------------------------------------------------------------------------------------------------------------------------------------
10) Microproteinuria [07]
a. Microalbuminuria [16]
Ans.
Microalbuminuria is the presence of albumin in urine above the normal level but below the
detectable range of conventional methods.
It is defined as the persistent elevation of the urinary albumin excretion of 20-200 mg/L (or 20-
200 micrograms/min) in an early morning urine sample.
It indicates early and possibly reversible glomerular damage.
Causes of microalbuminuria: Diabetes mellitus & Essential hypertension
----------------------------------------------------------------------------------------------------------------------------------------
11) Kidney biopsy [2000]
Ans.

Renal biopsy is used to establish the diagnosis and


severity of renal disease in order to judge the
prognosis and need for treatment.

----------------------------------------------------------------------------------------------------------------------------------------

1. Contraindications to Renal Transplantation [22]


Ans.
Absolute Contraindications Relative Contraindications
 Active malignancy.
Very young children (<1 year) or older people
 Active vasculitis or anti-GBM disease.
(>75 years).
 Severe ischemic heart disease
High risk of disease recurrence in the
 Severe occlusive aorto-iliac disease.
transplant kidney.
 Persistent substance abuse.
Severe bladder or urethral abnormalities.
 Severe mental retardation & psychiatric
Significant comorbidity.
disease.
----------------------------------------------------------------------------------------------------------------------------------------
2. Name 2 DPP-4 inhibitors [22]
Ans.

----------------------------------------------------------------------------------------------------------------------------------------
3. Four causes of sterile pyuria [22]
Ans.
Sterile pyuria is the persistent finding of white cells in the urine in the absence of bacteria.
:
Infectious Causes – Chlamydia trachomatis; Mycoplasma hominis; Renal TB
Non-infectious Causes –
▪ After effect of radiotherapy
▪ Inflammation in pelvis – appendicitis
▪ Physiological change in body – Pregnancy or Post-menopausal changes
Due to Underlying Diseases – Lupus; DM; Malignant hypertension; Kawasaki disease
----------------------------------------------------------------------------------------------------------------------------------------
4. Causes of Hematuria [19, 18, 04]
Ans.

----------------------------------------------------------------------------------------------------------------------------------------
5. Name 4 Nephrotoxic drugs [19, 12]
a. Drug induced nephropathy [13]
Ans.
DRUG CLASS/DRUG PATHOPHYSIOLOGICAL MECHANISM
OF RENAL INJURY
ANALGESICS-
Acetaminophen, Aspirin Chronic interstitial nephritis
ANTIDEPRESSANTS/MOOD STABILIZERS-
Amitriptyline ,fluoxetine Rhabdomyolysis

Lithium Chronic interstitial nephritis,


glomerulonephritis, rhabdomyolysis
ANTIRETROVIRALS
Tenofovir Tubular cell toxicity

Indinavir Crystal Nephropathy


ANTIMICROBIALS-
Aminoglycosides, Amphotericin B Tubular cell toxicity

Quinoles, Rifampin , Sulphonamides and Acute Interstitial Nephritis


Vancomycin

----------------------------------------------------------------------------------------------------------------------------------------
6. Significant Bacteriuria [15]
Ans.
Significant Bacteriuria is defined as a urine sample containing > 105 CFU/ml of urine.
It indicates Urinary Tract Infection.
Kass concept of significant bacteriuria: This is based on the fact that, though the normal urine is
sterile it may get contaminated during voiding, with normal urethral flora. However, the bacterial
count in contaminated urine would be lower than that caused by an infection
----------------------------------------------------------------------------------------------------------------------------------------
7. Mention 4 causes of transient incontinence of urine in old age [14]
Ans.

----------------------------------------------------------------------------------------------------------------------------------------
8. Endocrine functions of the kidney [07]
Ans.
– stimulates production of RBCs especially in anemia
– from JG cells – converts Angiotensinogen to Angiotensin-I
Converts 25-hydroxyvitamin D3 to 1,25-dihydroxyvitamin D3 – to  calcium in blood.
– stimulates thrombopoiesis.
– vasodilators and anti-hypertensive action
– potent vasodilator and anti-hypertensive
Kininogen Bradykinin
Renal Kallikrein
etc.
----------------------------------------------------------------------------------------------------------------------------------------
9. Massive Proteinuria [06]
Ans.
Massive proteinuria is characterized by daily loss of > 3g/day of protein (less in children) in the urine
Seen in nephrotic syndrome.
Proteinuria is due to permeability of glomerular capillary wall to plasma proteins.
Major protein lost in the urine is albumin (low-molecular-weight proteins), and rarely globulins
(high-molecular-weight proteins).
----------------------------------------------------------------------------------------------------------------------------------------
10. Retinal vein thrombosis [06]
Ans.
Retinal vein occlusions are more common than the artery
occlusions
Predisposing factors: HTN, DM, Hyperviscosity of blood, ↑IOP,
orbital cellulitis, orbital tumors etc.
Site of Occlusion: It is just behind the lamina cribrosa
Symptom: There is Painless sudden onset of impaired vision
Signs – splashed tomato appearance of Retina in ischemic cases
Treatment:
1) Observation & monitoring in patients with mild to moderate visual loss (since CRVO resolves with almost
normal vision.)
2) For patients with marked visual loss: Intravitreal anti-VEGF drugs: e.g., Bevacizumab &
Intravitreal steroids – ex: triamcinolone acetonide – useful for associated CME &
Neovascularisation
3) Treatment of Predisposing factors – Ex: HTN, DM
----------------------------------------------------------------------------------------------------------------------------------------
11. Selective proteinuria [05]
Ans.
 In Selective proteinuria, only intermediate-sized
(<100 kDa) proteins (such as albumin, transferrin)
leaks through the glomerulus.
 Seen in Minimal Change Disease

You might also like